Download as pdf or txt
Download as pdf or txt
You are on page 1of 93

MODUL • Fizik TINGKATAN 4

UNIT

2 DAYA DAN GERAKAN


FORCES AND MOTION

Objektif pembelajaran / Learning objective

• Menganalisis gerakan linear. / Analysing linear motion.


• Menganalisis graf gerakan. / Analysing motion graphs.
• Memahami inersia. / Understanding inertia.
• Menganalisis momentum. / Analysing momentum.
• Memahami kesan daya. / Understanding the effect of force.
• Menganalisis impuls dan daya impuls. / Analysing impulse and impulsive force.
• Menyedari kepentingan ciri-ciri keselamatan kenderaan. / Realising the importance of vehicle safety features.
• Memahami daya graviti. / Understanding gravitational force.
• Menganalisis keseimbangan daya. / Analysing forces in equilibrium.
• Memahami kerja, tenaga, kuasa dan kecekapan. / Understanding work, energy, power and efficiency.
U • Memahami kekenyalan. / Understanding elasticity.
N
I
T

2 GERAKAN LINEAR
2.1 LINEAR MOTION

Jumlah panjang laluan yang dilalui dari satu lokasi ke satu lokasi yang lain
Terangkan
The total path length travelled from one location to another
jarak
Explain Kuantiti / Quantity: Skalar / Scalar
distance
Unit S.I / S.I unit: meter (m)

Sesaran ialah / Displacement is


(a) jarak dalam arah tertentu. / the distance in a specific direction.
(b) jarak antara dua lokasi yang diukur sepanjang laluan yang paling pendek yang menghubungkannya
Terangkan dalam arah tertentu. / the distance between two locations measured along the shortest path connecting
sesaran them in a specific direction.
Explain (c) jarak kedudukan akhir dari kedudukan awal dalam arah tertentu.
displacement the distance of its final position from its initial position in a specified direction.

Kuantiti / Quantity: Vektor / Vector

Unit S.I / S.I unit: meter (m)

Sekolah
Rajah di sebelah kanan menunjukkan beza antara jarak dan sesaran. School
Apabila Ah Chong berjalan kaki dari rumah A ke sekolah B, Jalan Baik
The diagram on the right shows the difference between distance and displacement. B
When Ah Chong walked from House A to School B,
Jarak / Distance = Panjang laluan di sepanjang Jalan Baik
Length of the road along Jalan Baik
Sesaran / Displacement = Panjang garis lurus AB
Length of the straight line AB A
Rumah
House

© Nilam Publication Sdn. Bhd. 24


MODUL • Fizik TINGKATAN 4

Contoh / Examples
Warung Pak Din Rumah Rahim
Pak Din’s stall Rahim’s house
Rahim berjalan dari rumahnya ke simpang sejauh 1.5 km.
Kemudian dia berpatah balik dan berhenti di warung Pak Din
yang sejauh 0.5 km dari rumahnya.
B O
Rahim walked from his house to the junction which is 1.5 km. Then he 0.5 km 1.5 km
turned back and stopped at Pak Din’s stall which is 0.5 km from his
house. Utara/North
(a) Berapakah sesaran Rahim dari rumahnya
What is Rahim’s displacement from his house
(i) apabila dia sampai di simpang? / when he reached the junction?
1.5 km ke timur
1.5 km to the east
(ii) apabila dia berada di warung Pak Din? / when he was at Pak Din’s stall?
0.5 km ke barat
0.5 km to the west

(b) Selepas bersarapan pagi, Rahim berjalan pulang ke rumahnya. Apabila dia sampai di rumahnya, U
After breakfast, Rahim walked back to his house. When he reached home, N
I
(i) berapakah jumlah jarak yang dilalui oleh Rahim? T
what was the total distance travelled by Rahim?
1.5 km + 1.5 km + 0.5 km + 0.5 km = 4 km
2
(ii) berapakah jumlah sesaran Rahim dari rumahnya?
what was Rahim’s total displacement from his house?
0 km

Laju dan Halaju


Speed and Velocity

1 Laju ialah kadar perubahan jarak


Speed is the rate of change of distance

Jarak dilalui Speed, v =


Distance travelled
Laju, v = Time taken
Masa yang diambil

Terangkan Kuantiti / Quantity: Skalar / Scalar


laju Unit S.I / S.I unit: m s–1
Explain
speed 2 Laju purata / Average speed
Jumlah jarak dilalui, s Total distance travelled, s
v= v =
Jumlah masa yang diambil, t Total time taken, t

Laju sekata Laju yang magnitudnya kekal sama tanpa mempertimbangkan


arahnya.
Uniform speed Speed that remains the same in magnitude regardless of its direction.

25 © Nilam Publication Sdn. Bhd.


MODUL • Fizik TINGKATAN 4

1 Halaju ialah kadar perubahan sesaran


Velocity is the rate of change of displacement

Sesaran Velocity, v =
Displacement
Halaju, v = Time taken
Masa yang diambil

Kuantiti / Quantity: Vektor / Vector


Unit S.I / S.I unit: m s–1

2 Halaju purata / Average velocity


Terangkan
halaju Sesaran, s Displacement, s
v= v =
Explain Jumlah masa yang diambil, t Total time taken, t
velocity
Halaju sekata Halaju yang magnitud dan arahnya kekal sama.
Uniform velocity Velocity that remains the same in magnitude and direction .

3 Suatu objek mempunyai halaju tak sekata jika:


An object has a non-uniform velocity if:
U
(a) arah gerakan berubah atau gerakan tidak linear.
N
I
T the direction of motion changes or the motion is not linear.
2 (b) magnitud halaju berubah.
the magnitude of its velocity changes.

Contoh / Examples
400 km j–1
A B
1 Sebuah kapal terbang menuju ke utara selama 1 jam dengan halaju
300 km j–1. Kemudian, kapal terbang itu menuju ke timur selama 1 jam
dengan halaju 400 km j–1. 300 km j–1
An aeroplane flies towards the north for 1 hour with a velocity of 300 km h–1.
Then, the aeroplane flies towards the east for 1 hour with a velocity of 400 km h–1.
(a) Berapakah laju purata kapal terbang itu?
What is the average speed of the aeroplane? O

Jumlah jarak Total distance


Laju purata = Average speed = Total time
Jumlah masa
Jarak OA = LajuOA × MasaOA Distance OA = SpeedOA × TimeOA
= 300 km j–1 × 1 jam = 300 km h–1 × 1 hour
= 300 km = 300 km
Jarak AB = LajuAB × MasaAB Distance AB = SpeedAB × TimeAB
= 400 km j–1 × 1 jam = 400 km h–1 × 1 hour
= 400 km = 400 km
(300 km + 400 km) (300 km + 400 km)
∴ Laju purata = ∴ Average speed = 2 hours
2 jam
700 km 700 km
= = 350 km j–1 = 2 hours = 350 km h–1
2 jam

© Nilam Publication Sdn. Bhd. 26


MODUL • Fizik TINGKATAN 4

(b) Berapakah halaju purata kapal terbang itu?


What is the average velocity of the aeroplane?
Dari (a); From (a);
JarakOA = 300 km DistanceOA = 300 km
JarakAB = 400 km DistanceAB = 400 km
∴ SesaranOB = (300 km)2 + (400 km)2 ∴ DisplacementOB = (300 km)2 + (400 km)2
= 500 km = 500 km
Sesaran Displacement
∴ Halaju purata = ∴ Average velocity = Time
Masa
500 km 500 km
= = 2h
2j
= 250 km h–1

= 250 km j–1

U
N
I
2 Bacaan meter laju bagi sebuah kereta yang bergerak ke arah utara menunjukkan 80 km j–1. Sebuah kereta yang T
lain bergerak pada 80 km j–1 menuju ke selatan. Adakah kelajuan kedua-dua kereta itu sama? Adakah halaju
kedua-dua kereta itu sama? Terangkan jawapan anda.
2
–1 –1
The speedometer reading for a car travelling towards north shows 80 km h . Another car is travelling at 80 km h
towards the south. Is the speed of both cars the same? Is the velocity of both cars the same? Explain your answer.
Kelajuan kedua-dua kereta itu adalah sama iaitu 80 km j–1 tetapi halaju adalah tidak sama kerana arah
kedua-dua kereta itu berbeza.
The speed of both cars is the same, that is, 80 km h–1 but the velocity is not the same because the cars are in different
directions.

Definisi pecutan dan nyahpecutan


Definition of acceleration and deceleration

1 Pecutan, a, didefinisikan sebagai kadar perubahan halaju.


Acceleration, a, is defined as the rate of change of velocity.

2 Formula dan unit S.I:


Formula and S.I unit:

Perubahan halaju Change in velocity


Pecutan, a = Acceleration, a =
Masa yang diambil Time taken
Halaju akhir, v – Halaju awal, u Final velocity, v – Initial velocity, u
= =
Masa yang diambil, t Time taken, t
v–u v–u
= =
t t

Unit S.I / S.I unit: m s–2

27 © Nilam Publication Sdn. Bhd.


MODUL • Fizik TINGKATAN 4

‘a positif ’ (pecutan): Halaju suatu objek bertambah dari halaju awal, u, kepada halaju akhir, v yang lebih tinggi.
‘positive a’ (acceleration): The velocity of an object increases from an initial velocity, u, to a higher final velocity, v.
‘a negatif ’ (nyahpecutan): Halaju suatu objek berkurang dari halaju awal, u, kepada halaju akhir, v yang lebih
rendah.
‘negative a’ (deceleration): The velocity of an object decreases from an initial velocity, u, to a lower final velocity, v.

Pecutan sifar bermaksud suatu objek berada dalam keadaan pegun atau bergerak pada halaju malar, a = 0
Zero acceleration means an object is at rest or is moving at a constant velocity, a = 0

Contoh / Examples
1 Seorang penunggang basikal bermula dari keadaan rehat dan menambahkan halajunya pada kadar seragam
sehingga dia mencapai halaju 4.0 m s–1 dalam 5.0 s. Berapakah purata pecutannya?
A cyclist starts from rest and increases his velocity at a constant rate until he reaches a velocity of 4.0 m s–1 in 5.0 s.
U What is his average acceleration?
N v–u v–u
I Halaju awal / Initial velocity = 0 pecutan, a = / acceleration, a = t
T
t
Halaju akhir / Final velocity = 4.0 m s–1
2 Masa yang diambil / Time taken = 5.0 s
a= (4.0 – 0) m s–1
5.0 s
= 0.8 m s–2

2 Perhatikan gerakan sebuah kereta dalam Rajah (a) dan Rajah (b) yang merupakan gambar stroboskop.
Observe the motion of the car in Diagram (a) and Diagram (b) which are stroboscopic pictures.

Arah gerakan
Direction of motion

Rajah (a) / Diagram (a)


(a)

Arah gerakan Arah gerakan


Direction of motion Direction of motion

Rajah (b)
(b) / Diagram (b)
(i) Huraikan perubahan halaju sebuah kereta dalam Rajah (a). Adakah kereta memecut atau menyahpecut?
Describe the changes in velocity of the car in Diagram (a). Is the car accelerating or decelerating?
Jarak antara dua gambar berturut-turut bertambah. Halaju kereta itu bertambah. Kereta itu memecut.

The distance between two consecutive images increases. Velocity of the car is increasing. The car accelerates.

(ii) Huraikan perubahan halaju sebuah kereta dalam Rajah (b). Adakah kereta memecut atau menyahpecut?
Describe the changes in velocity of the car in Diagram (b). Is the car accelerating or decelerating?
Jarak antara dua gambar berturut-turut berkurang. Halaju kereta itu berkurang. Kereta itu mengalami

nyahpecutan.

The distance between two consecutive images decreases. Velocity of the car is decreasing. The car decelerates.

© Nilam Publication Sdn. Bhd. 28


MODUL • Fizik TINGKATAN 4

3 Isikan tempat kosong: / Fill in the blanks:


(a) Laju malar 10 m s–1 Jarak sejauh 10 m dilalui setiap saat

A constant speed of 10 m s–1 A distance of 10 m travelled every second

(b) Halaju malar –10 m s–1 Sesaran sejauh 10 m yang dilalui setiap saat dalam arah bertentangan
A constant velocity of –10 m s–1
A displacement of 10 m travelled every second in the opposite direction

(c) Pecutan malar 4 m s–2 Halaju meningkat dengan 4 m s–1 setiap saat

A constant acceleration of 4 m s–2 Velocity increases by 4 m s–1 every second

(d) Nyahpecutan malar 4 m s–2 Halaju berkurang dengan 4 m s–1 setiap saat

A constant deceleration of 4 m s–2 Velocity decreases by 4 m s–1 every second

Hubung kait Sesaran, Halaju, Pecutan dan Masa


Relating Displacement, Velocity, Acceleration and Time
U
N
Jangka masa detik: / Ticker timer: I
• Ia disambung ke bekalan kuasa arus ulang-alik 50 Hz. T

Apabila dihidupkan, bilah pengetuk akan bergetar 50 2


kali sesaat. / It is connected to an alternating current supply 6 – 12 V a.c.
of 50 Hz. When it is switched on, the iron strip will vibrate
50 times per second.
• Masa diambil untuk membuat 50 titik pada pita detik
ialah 1 saat. Jadi, selang masa antara dua titik yang
1 Jangka masa detik
berturutan ialah s = 0.02 s. / The time taken to make
50 Troli dinamik Ticker timer
50 dots on the ticker tape is 1 second. Hence, the time interval Dynamic trolley
1
between 2 consecutive dots is s = 0.02 s.
50
• 1 detik didefinisikan sebagai selang masa antara 2 titik.
1 tick is defined as the time interval between 2 dots.

Aktiviti 1: Kaedah pengiraan


Activity 1: Method of calculation

Menyiasat gerakan dalam makmal untuk menentukan jarak / sesaran, kelajuan / halaju, masa dan pecutan /
nyahpecutan / To investigate motion in laboratory to determine distance / displacement, speed / velocity, time and
acceleration / deceleration

10 detik / ticks

Arah
gerakan
Direction of
motion A 8.0 cm B

29 © Nilam Publication Sdn. Bhd.


MODUL • Fizik TINGKATAN 4

LANGKAH 1: Menentukan masa diambil untuk 1 detik.


STEP 1: Determine the time taken for 1 tick.

(a) Masa yang diambil untuk 50 detik = 1 saat (a) Time taken for 50 ticks = 1 second
(b) Masa yang diambil untuk 1 detik = 0.02 s (b) Time taken for 1 tick = 0.02 s
(c) Masa yang diambil dari A ke B = 10 detik (c) Time taken from A to B = 10 ticks
= 0.2 s = 0.2 s

LANGKAH 2: Menentukan sesaran


STEP 2: Determine the displacement
Sesaran suatu objek ditentukan dengan mengukur panjang pita detik yang ditarik melalui jangka masa detik
The displacement of the object is determined by measuring the length of the ticker tape that is pulled through the ticker timer
Sesaran A ke B = 8.0 cm
Displacement from A to B = 8.0 cm
LANGKAH 3: Menentukan halaju
STEP 3: Determine the velocity
U
N Sesaran Velocity, v =
Displacement
I Halaju, v = Time
T Masa

2 =
8.0 cm
0.2 s
= 40.0 cm s–1 =
8.0 cm
0.2 s
= 40.0 cm s–1

LANGKAH 4: Menentukan pecutan


STEP 4: Determine the acceleration

1.5 cm 3.5 cm 5.5 cm 7.5 cm

Arah gerakan
Direction of
A u B C D ν E
motion

Dari jalur pertama: Halaju awal, u, pada AB


From the first strip: Initial velocity, u, at AB
1.5 cm
uAB = = 7.5 cm s–1
0.2 s

Dari jalur terakhir: Halaju akhir, v, pada DE


From the final strip: Final velocity, v, at DE
7.5 cm
vAB = = 37.5 cm s–1
0.2 s

Selang masa, bagi perubahan halaju, t


The time interval, t, for the change in the velocity
t = (4 – 1) × 0.2 s = 0.6 s

Pecutan / Acceleration,
(37.5 – 7.5) cm s–1
a= = 50.0 cm s–2
0.6 s

© Nilam Publication Sdn. Bhd. 30


MODUL • Fizik TINGKATAN 4

Latihan / Exercises

1 Arah pergerakan 2 Arah pergerakan


Direction of motion Direction of motion
A 2.0 cm B 4.0 cm C 6.0 cm D 8.0 cm E

0.2 cm 1.4 cm
Rajah di atas menunjukkan suatu pita detik yang
mengandungi 5 detik untuk setiap selang AB-BC-CD dan
Berdasarkan rajah di atas, hitungkan pecutan objek DE. Hitungkan pecutan objek itu.
itu. The diagram above shows a ticker tape contains 5 ticks for
Based on the diagram above, calculate the acceleration every interval AB-BC-CD and DE. Calculate the acceleration of
of the object. the object.
Penyelesaian / Solution Penyelesaian / Solution
0.2 cm 8.0 cm
u= = 10 cm s–1 u= = 80.0 cm s–1 t = 5 detik / ticks × 0.02
0.02 s 0.1 s
1.4 cm = 0.1 s
v= = 70 cm s–1 2.0 cm
0.02 s v= = 20.0 cm s–1
0.1 s
U
t = (5 – 1) × 0.02 s = 0.08 s t = (4 – 1) × 0.1 s = 0.3 s N
I
(70 – 10) cm s–1 (20.0 – 80.0) T
a= = 750 cm s–2 = 7.5 m s–2 a= = –200 cm s–2 = –2.0 m s–2
0.08 s 0.3 s
2
3 Rajah di sebelah menunjukkan carta pita detik bagi sebuah
troli yang bergerak. Frekuensi bagi jangka masa detik ialah
50 Hz. Setiap jalur pita mempunyai panjang 10 detik. Panjang 10 detik / cm
The diagram on the right shows a ticker tape chart for a moving 10-tick length / cm
trolley. The frequency of the ticker-timer used is 50 Hz. Each strip
12.0
of the tape is a 10-ticks length.
(a) Berapakah selang masa antara dua titik? Jalur pita
Strip of the tape
What is the time interval between two dots? 10.0
0.02 s

(b) Berapakah selang masa untuk satu jalur? 8.0


What is the time interval for one strip?
0.02 × 10 = 0.2 s 6.0

(c) Berapakah halaju awal?


What is the initial velocity? 4.0
2.0 cm
u= = 10.0 cm s–1
0.2 s
2.0
(d) Berapakah halaju akhir? / What is the final velocity?
12.0 cm
v= = 60.0 cm s–1
0.2 s 0
Detik / Ticks
(e) Berapakah selang masa yang diambil untuk berubah dari halaju awal kepada halaju akhir?
What is the time interval to change from its initial velocity to its final velocity?
t = (11 – 1) × 0.2 s = 2.0 s

(f) Berapakah pecutan objek itu? / What is the acceleration of the object?
(60.0 – 10.0) cm s–1
a= = 25.0 cm s–2
2.0 s

31 © Nilam Publication Sdn. Bhd.


MODUL • Fizik TINGKATAN 4

Aktiviti 2: Untuk mengenal pasti jenis gerakan


Activity 2: To identify the types of motion

PITA DETIK DAN CARTA


TICKER TAPE AND CHARTS
(i)
Arah
gerakan
Direction Jenis gerakan: Laju malar
Panjang / Length (cm)
of motion Constant speed
Type of motion:

Jarak dilalui antara dua titik berturutan: Sama


Distance between two consecutive dots: Equal

Detik / Ticks

(ii) Arah • Jarak antara dua titik berturutan bertambah


gerakan
Panjang / Length (cm)
secara seragam.
U Direction
N 4.0 of motion The distance between two consecutive dots increases
I
T 3.3 uniformly.
• Halaju objek itu bertambah secara seragam.
2 2.6
1.9 The velocity of the object increases uniformly.
1.2 • Objek itu bergerak pada pecutan seragam.
0.5 acceleration
The object moves at a uniform .
Detik / Ticks
1 2 3 4 5 6

• Jarak antara dua titik berturutan berkurang


(iii) Arah secara seragam.
gerakan
Panjang / Length (cm) Direction The distance between two consecutive dots decreases
of motion
uniformly.
• Halaju objek itu berkurang secara seragam.
The velocity of the object decreases uniformly.
• Objek itu bergerak pada nyahpecutan seragam.
The object moves at a uniform deceleration .
Detik / Ticks

Menyelesaikan masalah gerakan linear dengan pecutan seragam


Solve problems on linear motion with uniform acceleration

(1) v = u + at (2) s = u + v t
 
2
1 2
(3) s = ut + at (4) v2 = u2 + 2as
2
di mana / where: Maklumat tambahan:
s: sesaran / displacement v: halaju akhir / final velocity Additional information:
u: halaju awal / initial velocity a: pecutan / acceleration
t: masa / time

© Nilam Publication Sdn. Bhd. 32


MODUL • Fizik TINGKATAN 4

Latihan / Exercises

1 Sebuah kereta memecut dari keadaan rehat ke 25 m s–1 dalam 4 s. Cari pecutan kereta itu.
A car accelerates from rest to 25 m s–1 in 4 s. Find the acceleration of the car.
Penyelesaian / Solution
u = 0, v = 25 m s–1, t = 4 s, a = ?
v – u 25 m s–1 – 0
v = u + at a= = = 6.25 m s–2
t 4s

0 m s–1
–2
2 Sebuah kereta memecut dari keadaan rehat pada 3 m s a = 3 m s–2
sepanjang suatu jalan lurus. Berapakah sesaran yang
dilalui oleh kereta itu selepas 4 s?
A car accelerates from rest at 3 m s–2 along a straight road. Sesaran / Displacement
How far has the car travelled after 4 s?
Penyelesaian / Solution
u = 0, a = 3 m s–2, t = 4 s, sesaran / displacement = ? t=4s
1
s = ut + at2 U
2 N
I
1
= 0 + (3 m s–2)(4 s)2 = 24 m T

2
2
20 m s–1 v

3 Sebuah kereta bergerak dengan halaju 20 m s–1 sepanjang a = –3 m s–2


jalan lurus. Pemandu itu menekan brek selama 5 s. Ia
menyebabkan nyahpecutan 3 m s–2, berapakah halaju
akhir kereta itu?
A car is travelling at 20 m s–1 along a straight road. The driver
brakes for 5 s. This causes a deceleration of 3 m s–2. What is
the final velocity of the car?
t=5s
Penyelesaian / Solution
u = 20 m s–1, t = 5 s, a = –3 m s–2, v = ?
v = u + at
= 20 m s–1 + (–3 m s–2)(5 s)
= 5 m s–1

4 Sebuah kereta bergerak dengan halaju malar 40 m s–1. Pemandu ternampak suatu penghalang di
hadapannya dan dia segera menekan brek. Dia dapat memberhentikan keretanya dalam masa 8 s. Jarak
antara penghalang itu dari kereta apabila pemandu ternampak penghalang itu ialah 180 m. Berapakah jarak
penghalang itu dari kereta selepas ia berhenti?
A car was moving at a constant velocity of 40 m s–1. The driver saw an obstacle in front and he immediately stepped on
the brake pedal. He managed to stop the car in 8 s. The distance of the obstacle from the car when the driver spotted
it was 180 m. How far was the obstacle from the car after it stopped?
Penyelesaian / Solution
u = 40 m s–1, v = 0, t = 8 s, s = ?
u+v (40 m s–1 + 0)(8 s)
s=  2 
t=
2
= 160 m
Jarak penghalang dari kereta itu selepas berhenti / The distance of the obstacle from the car after it stopped
= 180 m – 160 m
= 20 m

33 © Nilam Publication Sdn. Bhd.


MODUL • Fizik TINGKATAN 4

GRAF GERAKAN
2.2 MOTION GRAPH

Daripada graf gerakan, anda akan memahami:


From motion graphs, you will understand:
(i) berapa jauh yang telah dilalui oleh sesuatu objek – (jarak, sesaran)
how far an object has travelled – (distance, displacement)
(ii) berapa cepat ia bergerak – (laju, halaju)
how fast it is moving – (speed, velocity)
(iii) perubahan halaju terhadap masa – (pecutan / nyahpecutan)
change of velocity with time – (acceleration / deceleration)

Lakar dan tafsir graf sesaran-masa


Plot and interpret displacement-time graph

U Rajah di sebelah kanan menunjukkan gerakan t=0s 1s 2s 3s 4s 5s


N sebuah kereta pada masa yang berlainan.
I
T The diagram on the right shows the motion of
Kedudukan: 0 m 10 m 20 m 30 m 40 m 50 m
2
a car with respect to time. Position
(a) Lakarkan graf sesaran-masa kereta itu.
Plot a displacement-time graph for the car. Graf sesaran-masa
(b) Hitungkan kecerunan graf. Displacement-time graph
Calculate the gradient of the graph.
(50 – 0) m 50
Kedudukan / Position (m)

Kecerunan / Gradient = = 10 m s–1


5s 40
30
20
(c) Apakah unit bagi kecerunan ini?
What is the unit for this gradient? 10
–1
ms 0
0 1 2 3 4 5
(d) Apakah kuantiti fizik yang ditunjukkan oleh Masa / Time (s)
unit ini?
What is the physical quantity shown by this unit? Kecerunan graf = Halaju
The gradient = Velocity of the graph
Halaju / Velocity

Membuat kesimpulan daripada bentuk graf sesaran-masa


Deduction from the shape of a displacement-time graph

Sesaran / Displacement,
s/m Objek berada dalam keadaan rehat kerana ia berada pada
kedudukan yang sama pada bila-bila masa.
Objek pada The object is at rest because it is at the same position at any
keadaan rehat time.
Object at rest
Halaju = kecerunan graf
Velocity = gradient of the graph
0 t/s = 0 m s–1

© Nilam Publication Sdn. Bhd. 34


MODUL • Fizik TINGKATAN 4

Sesaran / Displacement,
s/m • Objek bergerak pada halaju malar kerana sesaran
bertambah secara seragam dalam setiap saat.
Objek bergerak Object travels at constant velocity because displacement
20
dengan halaju increases constantly in every second.
malar • Garis lurus graf mempunyai kecerunan yang tetap.
Object moving at The straight line of the graph has a constant gradient.
constant velocity • Halaju = kecerunan
t/s
Velocity = gradient
0 5 = 4 m s–1

Sesaran / Displacement, • Objek bergerak dengan pecutan kerana kadar perubahan


s/m sesaran bertambah.
Object moves with acceleration because the rate of change of
Objek bergerak displacement is increasing.
dengan pecutan • Kecerunan lengkungan bertambah dan ini menunjukkan
Object moving with halaju bertambah.
acceleration The gradient of the curve is increasing showing that velocity is
increasing.
0 t/s • Objek mengalami pecutan. U
The object experiences acceleration. N
I
T

2
Menentukan jarak, sesaran dan halaju dari graf sesaran-masa
Determine distance, displacement and velocity from the displacement-time graph
Rajah di bawah menunjukkan graf sesaran-masa bagi suatu objek. Berdasarkan graf,
The diagram below shows a displacement-time graph of an object. Based on the graph,
(a) Hitungkan halaju objek antara
Sesaran / m
Calculate the velocity of the object between Displacement / m
(i) A dan / and B (ii) B dan / and C (iii) C dan / and D
v = 20 m v=0 v = – 20 m
B C
20
10 s 5s
= 2 m s–1 = –4 m s–1

A D Masa / s
0 10 30 35 Time / s

(b) Gambarkan gerakan objek itu antara


Describe the motion of the object between
(i) A dan B: halaju malar bergerak ke hadapan
A and B: constant velocity

(ii) B dan C: dalam keadaan rehat


at rest
B and C:
(iii) C dan D: halaju malar tetapi objek bergerak atau bergerak semula ke belakang
C and D: constant velocity but the object moves in opposite direction or going backward
(c) Cari / Find
(i) jumlah jarak / total distance (ii) jumlah sesaran / total displacement
= (20 + 0 + 20) m = 40 m = (20 + 0 – 20) m = 0

35 © Nilam Publication Sdn. Bhd.


MODUL • Fizik TINGKATAN 4

(d) Hitungkan / Calculate


(i) laju purata (ii) halaju purata gerakan zarah itu.
the average speed the average velocity of the moving particle.
jumlah jarak total distance sesaran displacement
= = = =
masa time masa time
40 m 40 m
= = =0 = 0
35 s 35 s
= 1.14 m s–1
= 1.14 m s–1

Lakar dan tafsir graf halaju-masa


Plot and interpret velocity-time graph

Rajah menunjukkan kereta bergerak dari t=0s 1s 2s 3s 4s 5s


U rehat dalam suatu garis lurus.
N
I The diagram shows the car move from rest in
T a straight line.
2
Halaju: 0 m s–1 2 m s–1 4 m s–1 6 m s–1 8 m s–1 10 m s–1
(a) Lakarkan graf halaju-masa bagi Velocity
kereta ini.
Plot the velocity-time graph for the car.
Graf halaju-masa
(b) Hitungkan kecerunan graf. Velocity-time graph
Calculate the gradient of the graph. Halaju / Velocity
(m s–1)
Kecerunan = (10 – 0) m s–1 = 2 m s–2 10
Gradient (5 – 0) s
8

(c) Apakah unit bagi kecerunan ini? 6


What is the unit of this gradient? 4
m s–2 2

(d) Apakah kuantiti fizik yang ditunjukkan oleh 0


0 1 2 3 4 5
unit ini? Masa / Time
What is the physical quantity shown by this unit? (s)
Pecutan
Acceleration

Graf halaju-masa
Velocity-time graph
• Kecerunan graf = pecutan atau nyahpecutan
The gradient of graph = acceleration or deceleration
• Luas di bawah graf = sesaran
The area under the graph = displacement

© Nilam Publication Sdn. Bhd. 36


MODUL • Fizik TINGKATAN 4

Membuat kesimpulan daripada bentuk graf halaju-masa


Deduce from the shape of velocity-time graph

Objek berada dalam keadaan rehat 0


Object at rest • Kecerunan = pecutan =
Gradient = acceleration = 0
v / m s-¹

• Luas di bawah graf = sesaran = 0

Area under the graph = displacement = 0

• Objek berada dalam keadaan rehat .


Object is at rest .
0 t/s

Objek bergerak dengan • Kecerunan / Gradient = pecutan / acceleration = 0


halaju malar
• Luas di bawah graf = sesaran = 10 m s–1 × 2 s = 20 m
Object moves with constant velocity
v / m s-¹ Area under the graph = displacement = 10 m s–1 × 2 s = 20 m

• Sesaran dilalui = 20 m
10
Displacement travelled = 20 m U
N
• Objek bergerak dengan halaju malar I
0 t/s
2 T
constant
Object moves with velocity
2
–1
Objek bergerak dengan pecutan malar 18 m s
• Kecerunan / Gradient = malar / constant = = 6 m s–2
Object moving with constant 3s
acceleration –2
• Pecutan / Acceleration = 6 m s
v / m s-¹
• Luas di bawah graf / Area under the graph
18 1
(18 m s–1)(3 s) = 27 m
= 2
• Sesaran dilalui / Displacement travelled = 27 m
0 t/s • Objek bergerak dengan pecutan malar
3
Object moves with constant acceleration

Objek bergerak dengan


• Kecerunan graf adalah malar dan negatif
nyahpecutan malar
Object moves with constant deceleration The gradient of the graph is constant and negative
v / m s-¹

• Objek itu bergerak dengan nyahpecutan seragam


The object is moving with uniform deceleration

0 t/s

Nota / Notes:
• Dalam analisis gerakan linear, jika halaju suatu zarah adalah sifar, ini bermakna zarah itu berhenti bergerak.
In linear motion analysis, if the velocity of a particle is zero, it means that the particle has stopped moving.
• Jika halaju zarah itu menjadi negatif, maka zarah itu bergerak dalam arah bertentangan dengan arah gerakan awalnya.
If the velocity of the particle becomes negative, then the particle is moving opposite to its earlier direction of motion.

37 © Nilam Publication Sdn. Bhd.


MODUL • Fizik TINGKATAN 4

Menentukan jarak, sesaran, halaju dan pecutan daripada graf halaju-masa


Determine distance, displacement, velocity and acceleration from a velocity-time graph

Berikut menunjukkan graf halaju-masa bagi sebuah kereta. Berdasarkan graf,


The following shows the velocity-time graph of a car. Based on the graph,
(a) hitungkan pecutan kereta itu bagi / calculate the acceleration of the car for
(i) JK
–1
a = 20 m s = 2 m s–2
Halaju / Velocity / m s-1
10 s
K
(ii) KL 20
–1
a = – (20 – 10) m s = –1 m s–2
(20 – 10) s L M
10
(iii) LM
a = 0 N Q
J Masa / s
(iv) MN 0 10 20 30 35 40 45 50 Time / s
–1
a = – 10 m s = –2 m s–2
5s –10
P
U
N
I (b) Nyatakan jenis gerakan kereta itu bagi
T State the types of motion of the car for
2 (i) JK : pecutan malar (ii) KL : nyahpecutan malar (tetap)
constant acceleration
constant deceleration

(iii) LM : halaju tetap / malar / pecutan sifar (iv) MN : nyahpecutan malar (tetap)
constant velocity / zero acceleration
constant deceleration

(c) Hitungkan jumlah sesaran yang dilalui oleh kereta itu semasa
Calculate the total displacement travelled by the car during
(i) gerakan bagi 10 s yang pertama (ii) kereta bergerak dengan halaju seragam
the first 10 s of motion the car moves with uniform velocity
Sesaran = 1 (20 m s–1)(10 s) Sesaran (LM) = 10 m s–1 × 10 s = 100 m
Displacement 2 Displacement
= 100 m
(iii) gerakan bagi 10 s yang terakhir
the last 10 s of motion
Sesaran / Displacement = 1 (10 m s–1)(10 s) = 50 m (pada arah bertentangan / in opposite direction)
2
(d) Hitungkan / Calculate
(i) jumlah jarak bagi keseluruhan perjalanan / the total distance for the whole journey
Jarak / Distance
= 1 (10)(20) + 1 (10 + 20)10 + (10)(10) + 1 (5)(10) + 1 (10)(10)
2 2 2 2
= (100 + 150 + 100 + 25 + 50) m
= 425 m

(ii) jumlah sesaran bagi keseluruhan perjalanan.


the total displacement for the whole journey.
Sesaran / Displacement

2
[
= 1 (10)(20) + 1 (10 + 20)10 + (10)(10) + 1 (5)(10) – 1 (10)(10)
2 2 2
]
= (100 + 150 + 100 + 25) m – 50 m
= 325 m

© Nilam Publication Sdn. Bhd. 38


MODUL • Fizik TINGKATAN 4

(e) Hitungkan / Calculate


(i) laju purata / the average speed (ii) halaju purata / the average velocity
Laju purata / Average speed Halaju purata / Average velocity
425 m 325 m
= = 9.44 m s–1 = = 7.22 m s–1
45 s 45 s

Menyelesaikan masalah gerakan linear dengan pecutan seragam


Solve problems on linear motion with uniform acceleration

Sesaran / Displacement, 1 Hitungkan / Calculate


s/m
(i) halaju bagi OP, QR dan RS
velocity of OP, QR and RS
P Q (ii) sesaran / displacement
20

Penyelesaian / Solution:
10
(i) OP: halaju / velocity = 20 m = 10 m s–1 U
2s N
I
QR: halaju / velocity = – 20 m = –10 m s–1
O R
T

2
0 1 2 3 4 5 6 7 8 t/s 2s
RS: halaju / velocity = – 10 m = –10 m s–1
–10 1s
S
(ii) s = (20 + 0 – 20 – 10)m = –10 m

2 Hitungkan / Calculate
Halaju / m s-¹
(i) pecutan bagi OA dan BC
Velocity / m s-¹ acceleration of OA and BC
(ii) jumlah sesaran / total displacement

10
A B Penyelesaian / Solution:
–1
(i) OA: pecutan = 10 m s = 1 m s–2
acceleration 10 s
–1
= – 10 m s = –2 m s–2
5
BC: pecutan
acceleration 5s
C
O t/s (ii) Jumlah sesaran = 1 (25 + 10)s (10 m s–1)
0 5 10 15 20 25 Total displacement 2
= 175 m

Kesimpulan / Conclusion
1 Kecerunan graf s melawan t memberikan halaju suatu objek.
Gradient of the graph s against t gives the velocity of an object.
2 Kecerunan graf v melawan t memberikan pecutan suatu objek.
Gradient of the graph v against t gives the acceleration of an object.
3 Luas di bawah graf v melawan t memberikan sesaran yang dilalui oleh objek.
Area under the graph v against t gives the displacement travelled by the object.

39 © Nilam Publication Sdn. Bhd.


MODUL • Fizik TINGKATAN 4

Ringkasan bagi bentuk graf gerakan


Summary of shapes of motion graphs

Graf s melawan t v melawan t a melawan t


Graph s against t v against t a against t

s v

Halaju sifar
Zero velocity

0 t 0 t

s v

Halaju negatif
0 t
Negative velocity
0 t
U
N
I
T s v a

2 Halaju seragam
Uniform velocity

0 t 0 t 0 t

s v a

Pecutan seragam
Uniform acceleration

0 t 0 t 0 t

s v a

Nyahpecutan seragam t
0
Uniform deceleration
0 t 0 t

Maklumat tambahan:
Additional information:

© Nilam Publication Sdn. Bhd. 40


MODUL • Fizik TINGKATAN 4

MEMAHAMI INERSIA
2.3 UNDERSTANDING INERTIA

Inersia suatu objek ialah kecenderungan objek untuk kekal dalam keadaan rehat atau terus bergerak dalam
keadaan gerakannya.
The inertia of an object is the tendency of the object to remain at rest or if moving to continue its motion.

• Suatu objek berada dalam keadaan rehat akan cenderung kekal dalam keadaan rehat .
An object in a state of rest tends to remain at rest .

• Suatu objek yang berada dalam keadaan bergerak cenderung untuk kekal dalam keadaan gerakan .
An object in a state of motion tends to stay in motion .

Hukum Newton pertama / Newton's first law :


Suatu objek akan terus berada dalam keadaan rehat atau keadaan gerakannya dengan halaju seragam kecuali
ia dikenakan daya luar
An object continues in its state of rest or of uniform motion unless it is acted upon by an external force .
U
N
I
T

Hubung kait jisim dengan inersia 2


Relationship between mass and inertia

Semakin besar jisim, semakin besar inersia.


The larger the mass, the larger the inertia. Tali
Ropes
• Dua baldi kosong digantung dengan tali dari siling.
Two empty buckets are hung with rope from the ceiling. Pasir
Sand
• Sebuah baldi diisi dengan pasir manakala sebuah baldi yang lain adalah kosong.
One bucket is filled with sand while the other bucket is empty.

• Kemudian, kedua-dua baldi ditolak. Baldi


Then, both buckets are pushed. Buckets

• Didapati baldi kosong itu senang ditolak berbanding dengan baldi yang diisi dengan pasir.
It is found that the empty bucket is easy to push compared to the bucket with sand.

• Baldi yang diisi dengan pasir adalah lebih susah untuk digerakkan.
The bucket filled with sand is more difficult to move.

• Apabila kedua-dua baldi diayun dan cuba diberhentikan, baldi yang diisi dengan pasir lebih susah untuk
diberhentikan. / When both buckets are oscillating and an attempt is made to stop them, it is more difficult to stop the
bucket filled with sand.

• Ini menunjukkan baldi dengan jisim yang lebih besar menghasilkan rintangan yang lebih untuk berubah dari
keadaan rehat atau dari keadaan gerakan. / This shows that the bucket with a bigger mass offers a greater resistance
to change from its state of rest or from its state of motion.

• Oleh itu, suatu objek dengan jisim yang besar mempunyai inersia yang lebih besar.
So, an object with a larger mass has a larger inertia.

41 © Nilam Publication Sdn. Bhd.


MODUL • Fizik TINGKATAN 4

Aktiviti yang melibatkan inersia


Activities involving inertia

Apabila sekeping duit syiling 20 sen dijentik ke arah timbunan duit


syiling 20 sen pada permukaan yang licin, duit syiling di bawah dihentam
Syiling keluar tanpa menggerakkan duit syiling yang lain. Ini menunjukkan
Coin
bahawa inersia bagi timbunan duit syiling di atas cenderung untuk
kekal dalam keadaan rehat dan menentang gerakan.
When a 20 cent coin is flicked towards a stack of 20 cent coins on a smooth
surface, the bottom coin is knocked off without moving the rest of coins. This
shows that the inertia of the stack of coins above tends to remain at rest and
resists motion.

Apabila kadbod ditarik keluar dengan cepat, duit syiling itu terus jatuh
Syiling
ke dalam gelas.
Coin Inersia duit syiling itu mengekalkannya dalam keadaan rehat
walaupun kadbod itu ditarik keluar.
When the cardboard is quickly pulled away, the coin drops straight into the
U Kadbod glass.
N Cardboard The inertia of the coin maintains it in rest even when the cardboard is
I
T withdrawn.
2
Letakkan segelas air di atas sekeping kertas A4. Seterusnya, dengan
cepat tarik keluar kertas itu secara mendatar. Apakah yang akan berlaku
Gelas berisi air kepada gelas air itu? / Place a glass of water on a piece of A4 paper. Later,
Kertas Glass filled with water
Paper quickly pull the paper horizontally. What happens to the glass of water?

Gelas air itu kekal dalam keadaan rehat.


Inersia gelas yang berisi air itu cenderung mengekalkan gelas air dalam
keadaan rehat.
The glass of water remains at rest.
The inertia of the glass of water tends to remain at rest.

Sebuah blok kayu diletakkan di atas sebuah troli yang bergerak menuruni
Troli
Trolley landasan. Apabila gerakan troli itu dihalang oleh suatu penghalang,
Blok kayu blok kayu itu akan kekal dalam keadaan gerakan dan ia menggelongsor
Wooden block Penghalang ke hadapan. Inersia blok kayu itu cenderung untuk mengekalkan
Obstacle
keadaan gerakannya. / A wooden block is placed on top of a moving trolley
down a runway. When the motion of the trolley is stopped by an obstacle, the
wooden block will continue its state of motion and slide forward. The inertia
of the wooden block tends to keep its state of motion.

Sebuah buku ditarik keluar dari kedudukan tengahnya. Buku di atasnya


Buku
Books
akan jatuh ke bawah secara terus. Inersia cuba menentang perubahannya
dari keadaan rehat, iaitu, apabila buku ditarik keluar, buku-buku di atas
tidak akan bergerak bersamanya.
A book is pulled out from its central position. The books on top will drop
straight downwards. Inertia tries to resist the change from rest, that is, when
the book is pulled out, the books on top not follow suit.

© Nilam Publication Sdn. Bhd. 42


MODUL • Fizik TINGKATAN 4

Contoh-contoh situasi yang melibatkan inersia


Examples of situations involving inertia

Penumpang di dalam bas akan terhumban ke belakang apabila


bas yang pegun memecut ke hadapan. Mengapa?
Passengers in a bus will be thrown backwards when a stationary bus
starts to accelerate forwards. Why?

Sebab / Reason:
Apabila bas itu bergerak ke hadapan secara tiba-tiba dari keadaan
Bas yang pegun / The bus is stationary rehat, inersia badan penumpang cenderung untuk
kekal dalam keadaan rehat. Ini menyebabkan badan penumpang
terhumban ke belakang .
When the bus suddenly moves forward from rest, the inertia
of the passenger's body tends to keep them at rest. This causes their
body to be thrown backwards .

Bas bergerak ke hadapan secara tiba-tiba U


The bus moves suddenly forward N
I
T
Penumpang dalam bas yang bergerak terhumban ke hadapan
apabila bas itu berhenti secara tiba-tiba. Mengapa? 2
Passengers in a moving bus will be thrown forward when the bus
suddenly stops. Why?

Sebab / Reason:
Penumpang berada dalam keadaan gerakan apabila
Bas sedang bergerak / The bus is moving bas itu sedang bergerak. Apabila bas itu berhenti secara tiba-
tiba, inersia badan penumpang cenderung untuk terus bergerak
ke hadapan . Ini menyebabkan badan penumpang
terhumban ke hadapan .
The passengers are in a state of motion when
the bus is moving. When the bus suddenly stops,
the inertia of the passengers tends to continue in its
Bas berhenti secara tiba-tiba forward motion. This causes their body to be thrown
The bus suddenly stops forward .

Video

Hukum Newton
Newton's Law
https://goo.gl/exoLmo

43 © Nilam Publication Sdn. Bhd.


MODUL • Fizik TINGKATAN 4

Contoh-contoh situasi yang melibatkan inersia


Examples of situations involving inertia

Gerakan ke bawah Sos cili dalam botol boleh dituang keluar dengan senang jika botol digerakkan
yang cepat
Fast downward motion ke bawah dengan cepat dan berhenti secara tiba-tiba. Hal ini kerana:
Chili sauce in the bottle can be easily poured out if the bottle is moved downward fast
with a sudden stop. This is because:
• Sos dalam botol bergerak bersama-sama dengan botol semasa pergerakan ke
bawah.
Sos The sauce in the bottle moves with the bottle during the downward movement.
Sauce • Apabila botol itu berhenti secara tiba-tiba, inersia sos menyebabkan ia
terus bergerak ke bawah dan mengakibatkan sos dituang keluar dari botol
itu.
When the bottle is stopped suddenly, the inertia of the sauce causes it to continue in
its downward movement. Thus, the sauce is poured out of the bottle.

Gerakan ke bawah
yang cepat Kepala tukul dicantum dengan ketat kepada pemegangnya dengan mengetuk
Fast downward motion penghujung pemegangnya secara menegak di atas permukaan yang keras.
The head of hammer is secured tightly to its handle by knocking one end of the handle,
U
N held vertically, on a hard surface.
I • Ini menyebabkan kepala tukul meneruskan gerakan ke bawah apabila gerakan
T
pemegang itu diberhentikan. Dengan ini, hujung atas pemegang itu akan
2 dimasukkan lebih dalam ke dalam kepala tukul.
Tukul This causes the hammer head to continue on its downward motion when the motion
Hammer of the handle is stopped. So that the top end of the handle is slotted deeper into the
hammer head.

Titisan air pada payung yang basah akan jatuh apabila budak perempuan itu
memusingkan payung itu.
The water droplets on a wet umbrella will fall when the girl rotates the umbrella.
• Ini adalah disebabkan titisan air pada permukaan payung itu bergerak secara
serentak apabila payung itu dipusingkan.
This is because the water droplets on the surface of the umbrella move
simultaneously as the umbrella is rotated.
• Apabila payung itu berhenti berpusing, inersia titisan air akan terus
mengekalkan pergerakannya.
When the umbrella stops rotating, the inertia of the water droplets will continue in
its original motion.

Seorang budak melarikan diri dari lembu dalam gerakan zig-zag. Mengapa?
A boy runs away from a cow in a zig-zag motion. Why?

Sebab / Reason:
Lembu itu mempunyai jisim yang lebih besar, maka inersianya juga lebih
Lembu besar. Jadi, lembu itu sukar untuk menukar arah gerakannya.
Cow
A cow has a larger mass, so it has a larger inertia. So the cow has difficulty to
change its direction of motion.

© Nilam Publication Sdn. Bhd. 44


MODUL • Fizik TINGKATAN 4

Contoh-contoh situasi yang melibatkan inersia


Examples of situations involving inertia
Sebuah kapal minyak yang besar mengambil masa yang lebih panjang untuk memecut kepada laju maksimumnya
dan ia mengambil beberapa kilometer untuk berhenti walaupun propelernya telah diterbalikkan. Mengapa?
A massive oil tanker (a very big ship) takes a long time to accelerate to its full speed and a few kilometers to come to a stop
even though the engine has reversed its propeller to slow it down. Why?
Kapal minyak yang besar mempunyai jisim yang lebih besar, jadi inersianya juga lebih besar. Oleh itu,
ia adalah lebih sukar untuk memberhentikan kapal minyak.
The massive oil tanker has larger mass, so it has a larger inertia. So it is more difficult to stop the oil tanker.

Peta Pemikiran / Thinking Maps


Inersia (Punca dan Kesan)
Inertia (Cause and Effect)
U
N
Inersia menjadi besar I
1 Inertia becomes large T

Jisim objek besar


Larger mass of object
1 2
Objek yang berada dalam
keadaan rehat akan cenderung
Inersia 2
Inertia
kekal dalam keadaan rehat
2 Object at rest will tend to
Hukum Gerakan remain at rest
Newton Pertama
Newton's first Law 2
of Motion
Objek yang berada dalam
keadaan gerakan cenderung untuk
kekal dalam keadaan gerakannya
An object in motion tends to
remain in motion

Cadangan untuk mengurangkan kesan negatif inersia


Suggestions to reduce the negative effects of inertia

1 Keselamatan dalam kereta:


Safety in a car:
(a) Tali pinggang keledar mengekalkan pemandu pada tempat duduknya. Apabila kereta berhenti secara mendadak,
tali pinggang itu mengelakkan pemandu daripada terhumban ke hadapan .
A safety belt secures a driver to his seat. When the car stops suddenly, the seat belt prevents the driver from being
thrown forward .
(b) Alas kepala mencegah kecederaan leher semasa perlanggaran dari belakang. Inersia kepala cenderung untuk
mengekalkan keadaan rehat apabila badan digerakkan secara tiba-tiba ke depan.
A headrest prevents injuries to the neck during rear-end collisions. The inertia of the head tends to keep it in its state
of rest when the body is moved forward suddenly.

45 © Nilam Publication Sdn. Bhd.


MODUL • Fizik TINGKATAN 4

(c) Beg udara dipasang di dalam stereng. Ia membekalkan kusyen untuk mengelakkan pemandu
daripada terhentam pada stereng atau papan pemuka kereta semasa perlanggaran.
An air bag is fitted inside the steering wheel. It provides a cushion to prevent the driver from hitting the steering wheel
or dashboard during a collision.

2 Perabot yang dibawa oleh lori biasanya perlu diikat dengan tali kepada bahagian-bahagian lori yang tertentu
supaya apabila lori bergerak atau berhenti dengan tiba-tiba, perabot itu tidak akan jatuh atau tidak akan terhumban
ke hadapan. / Furniture carried by a lorry is normally tied by ropes to certain fixed parts of the lorry so that when the
lorry moves or stops suddenly, the furniture will not fall or will not be thrown forward.

3 Lori tangki mempunyai empat tangki kecil di mana jisim muatan dibahagi antara tangki-tangki tersebut akan
mempunyai inersia yang lebih kecil. Ini akan mengurangkan impak pada setiap tangki yang disebabkan oleh
inersia jika lori tangki itu berhenti dengan tiba-tiba. / Four small tanks with distributed mass will have smaller inertia.
This will greatly reduce the inertial impact on each tank if the tanker stops suddenly.

Beg udara
Airbag Tali Maklumat tambahan:
Rope Additional information:

U
N Treler dengan
I 4 tangki kecil
T Trailer with

2
4 small tanks
Kepala lori
Tractor Lori tangki
Tanker

Eksperimen Inersia dan Jisim


Experiment Inertia and Mass

Inersia bergantung kepada jisim objek / Sukar untuk menggerakkan atau memberhentikan
Inferens sesuatu jisim yang lebih besar
Inference
The inertia depends on mass of object / It is more difficult to move or stop a larger mass

Hipotesis Apabila jisim objek bertambah, inersianya juga bertambah.


Hypothesis When the mass of an object increases, its inertia also increases.

Tujuan Untuk mengkaji hubungan antara jisim dengan inersia (tempoh ayunan)
Aim
To study the relationship between mass and inertia (period of oscillation)

Radas Bilah gergaji, pengapit-G, jam randik dan plastisin.


Apparatus Hacksaw blade, G-clamp, stopwatch and plasticine.
Pengapit-G / G-clamp
Pemboleh ubah dimanipulasikan / Manipulated variable:
Jisim plastisin / Mass of plasticine Plastisin / Plasticine
Pemboleh ubah bergerak balas / Responding variable: Bilah Hacksaw / Hacksaw blade
Pemboleh ubah Tempoh ayunan / Period of oscillation
Variables
Pemboleh ubah dimalarkan / Constant variable:
Panjang bilah Hacksaw / amplitud ayunan / kekerasan bilah Hacksaw
Length of the Hacksaw blade / amplitude of oscillation / stiffness of Hacksaw blade

© Nilam Publication Sdn. Bhd. 46


MODUL • Fizik TINGKATAN 4

1 Letakkan sejumlah plastisin (berbentuk sfera) dengan jisim 30 g pada hujung bilah Hacksaw.
Place a lump of plasticine (sphere-shaped) with a mass of 30 g at the free end of the Hacksaw blade.
2 Sesarkan sedikit bilah Hacksaw dan lepaskannya supaya ia berayun secara mengufuk.
Displace the Hacksaw blade slightly and release it so that it oscillates horizontally.
3 Tentukan dan rekodkan masa yang diambil untuk 10 ayunan lengkap, t saat.
Prosedur Determine and record the time taken for 10 complete oscillations, t seconds.
Procedure t t
4 Hitungkan tempoh ayunan, T = saat. / Calculate period of oscillation, T = seconds.
10 10
5 Ulangi langkah 1 – 4 eksperimen dengan jisim 40 g, 50 g, 60 g dan 70 g.
Repeat steps 1 – 4 of the experiment with mass of 40 g, 50 g, 60 g and 70 g.
6 Lakarkan graf tempoh ayunan melawan jisim.
Plot the graph of period of oscillation against mass.

Masa untuk 10 ayunan, t / s


Jisim / g Time for 10 oscillation, t / s t
Mass / g T= s
t1 t2 tmin 10

30
Keputusan
Results 40
U
50 N
I
T
60
70 2
Lakarkan graf T melawan jisim, m.
Plot the graph T against mass, m.
T/s

Analisis
Analysis

0 m/g

1 Nyatakan kuantiti yang digunakan untuk mewakili inersia dalam aktiviti ini.
State the quantity used to represent inertia in this activity.
Tempoh ayunan / Period of oscillation

2 Apakah hubungan antara tempoh ayunan suatu objek dengan inersianya?


What is the relationship between the period of oscillation of an object and its inertia?
Semakin panjang tempoh ayunan, semakin besar inersia.
The longer the period of oscillation, the larger the inertia.
Perbincangan
Discussion 3 Daripada graf, nyatakan hubungan antara / From the graph, state the relationship between
(a) tempoh ayunan dengan jisim objek. / period of oscillation and mass of object.
Semakin besar jisim, semakin panjang tempoh ayunan.
The larger the mass, the longer the period of oscillation.
(b) inersia suatu objek dan jisimnya. / inertia of an object and its mass.
Semakin besar jisim objek, semakin besar inersianya.
The larger the mass of the object, the larger its inertia.

47 © Nilam Publication Sdn. Bhd.


MODUL • Fizik TINGKATAN 4

MENGANALISIS MOMENTUM
2.4 ANALYSING MOMENTUM

Definisi momentum sebagai hasil darab jisim dan halaju


Define momentum as the product of mass and velocity

Momentum = jisim × halaju Momentum = mass × velocity

Unit S.I: kg m s–1 atau N s (Newton saat) / S.I unit: kg m s–1 or N s (Newton second)

Momentum adalah suatu kuantiti vektor. Arah momentum mengikut arah halaju.
Momentum is a vector quantity. The direction of the momentum follows the direction of the velocity.

Contoh / Examples 1
Dalam permainan ragbi, seorang pemain berjisim 70 kg bergerak
dengan halaju 4 m s-1 dan seorang pemain yang lain yang berjisim
75 kg bergerak dengan 3 m s-1 menghala antara satu sama lain
U seperti yang ditunjukkan. Hitungkan momentum kedua-dua
N
I pemain itu masing-masing.
T
In a rugby game, a player of mass 70 kg is moving with velocity of 4 m s-1
2 and the other player of mass 75 kg is moving with 3 m s-1 towards each
other as shown. Calculate the momentum of the two players respectively. Pemain I Pemain II
Player I Player II
Penyelesaian / Solution

Momentum pemain I / Momentum player I = m1v1 = (70 kg)(4 m s–1) = 280 kg m s-1
Momentum pemain II / Momentum player II = m2v2 = (75 kg)(–3 m s–1) = –225 kg m s-1

Contoh / Examples 2
Seorang nenek (m = 80 kg) meluncur mengelilingi gelanggang dengan
halaju 6 m s–1. Tiba-tiba dia berlanggar dengan Bobby (m = 40 kg)
yang berada dalam keadaan rehat. Hitungkan momentum nenek dan
Bobby masing-masing.
A granny (m = 80 kg) skating around the ring with a velocity of 6 m s–1.
Suddenly she collides with Bobby (m = 40 kg) who is at rest. Calculate the
momentum of granny and Bobby respectively. Nenek
Penyelesaian / Solution Granny Bobby

Momentum nenek / granny = m1v1 = (80 kg)(6 m s–1) = 480 kg m s–1


Momentum Bobby = m2v2 = (40 kg) × (0 m s–1) = 0 kg m s–1 (dalam keadaan rehat / at rest)

Nyatakan prinsip keabadian momentum


State the principle of conservation of momentum

Tanpa kehadiran daya luar, jumlah momentum dalam suatu sistem kekal tidak berubah.
In the absence of an external force, the total momentum of a system remains unchanged.
Jumlah momentum sebelum perlanggaran / letupan = Jumlah momentum selepas perlanggaran / letupan
Total momentum before collision / explosion = Total momentum after collision / explosion

© Nilam Publication Sdn. Bhd. 48


MODUL • Fizik TINGKATAN 4

Aktiviti / Activity 1
(a) Rajah di sebelah menunjukkan dua orang adik-beradik yang sedang meluncur.
Abang bergerak dan berlanggar dengan adiknya yang berada dalam keadaan rehat.
Apakah gerakan mereka selepas perlanggaran?
The diagram on the right shows two brothers skating. The elder brother moves and collides
with his younger brother who is at rest. What is their movement after the collision?
Selepas perlanggaran, / After collison,
Laju abang berkurang. / The speed of the elder brother decreases.
Laju adik bertambah. / The speed of the younger brother increases.
Momentum abang berkurang. / Momentum of the elder brother decreases.
Momentum adik bertambah. / Momentum of the younger brother increases.
(b) Adakah jumlah momentum sebelum perlanggaran sama dengan jumlah momentum selepas perlanggaran?
Is the total momentum before collision equal to the total momentum after collision?
Ya / Yes

Aktiviti / Activity 2 Aktiviti / Activity 3


U
Menjentik sekeping duit syiling 20 sen, A secara terus Menjentik sekeping duit syiling 20 sen, A secara terus N
kepada sekeping duit syiling 20 sen, B yang lain. kepada dua keping duit syiling 20, sen B dan C. I
T
Flick a 20 cent coin, A, directly to another 20 cent coin, B. Flick a 20 cent coin A, directly to two 20 cent coins, B and C.
2

A B A B C

(a) Apakah yang berlaku kepada gerakan kedua-dua (a) Gambarkan gerakan semua duit syiling selepas
duit syiling tersebut selepas perlanggaran? perlanggaran.
What happens to the motion of both coins after collision? Describe the motion of all the coins after collision.
Duit syiling A berhenti, duit syiling B bergerak. Duit syiling A / Coin A:
Coin A stops, coin B moves. Berhenti / Stop
Duit syiling B / Coin B:
Rehat / At rest
Duit syiling C / Coin C:
Bergerak ke kanan / Moves to the right

(b) Apakah yang berlaku kepada momentum duit syiling (b) Apakah yang berlaku kepada momentum duit syiling
A selepas perlanggaran? A selepas perlanggaran?
What happens to the momentum of coin A after collision? What happens to the momentum of coin A after collision?
Momentum duit syiling A dipindahkan kepada Momentum duit syiling A dipindahkan ke duit
duit syiling B selepas perlanggaran. syiling B dan duit syiling C.
Momentum of coin A is transferred to coin B after Momentum of coin A is transferred to coin B and

collision. coin C.

49 © Nilam Publication Sdn. Bhd.


MODUL • Fizik TINGKATAN 4

Aktiviti / Activity 4
Rajah di sebelah menunjukkan sebiji bola keluli, A ditarik dan dilepaskan.
The diagram on the right shows a steel ball, A is pulled and released.
(a) Bola itu akan berlanggar dengan empat biji bola yang lain. Ini akan
menyebabkan bola terakhir, E bergerak ke ketinggian yang sama dengan
ketinggian bola A dilepaskan.
E D C B A
The ball will collide with the other four balls. This will cause the last ball, E to move to
the same height as ball A released.
Adakah momentum diabadikan?
Is the momentum conserved?
Ya / Yes
(b) Apakah yang akan berlaku jika kedua-dua bola A dan B ditarik dan kemudian dilepaskan?
What will happen if two balls A and B are pulled and then released?
Bola E dan D akan bergerak ke ketinggian yang sama dengan bola A dan B masing-masing. Bola C akan
berada dalam keadaan rehat.
Balls E and D move rise to the same heights of balls A and B respectively. Ball C is at rest.
U
N
I
T
Aktiviti / Activity 5
2 Seorang budak perempuan berdiri dalam keadaan rehat di atas papan luncur. Dia
membaling bola ke hadapan. Bola itu bergerak ke kiri. Budak perempuan bergerak
ke kanan.
A girl is standing at rest on the skateboard. She throws the ball forward. The ball moves to
the left. The girl moves to the right.
• Momentum bola sebelum balingan = 0
Momentum of the ball before the throw = 0
• Momentum budak perempuan sebelum balingan / Momentum of the girl before the throw = 0
• Jumlah momentum selepas balingan sama dengan jumlah momentum sebelum balingan = 0
Total momentum after the throw is equal to total momentum before the throw = 0
• Jumlah momentum selepas balingan = momentum bola + momentum budak perempuan = 0
Total momentum after the throw = momentum of the ball + momentum of the girl = 0

Oleh itu, selepas balingan, magnitud momentum budak perempuan adalah sama dengan magnitud
momentum bola tetapi dalam arah bertentangan .

Therefore, after the throw, the magnitude of the momentum of the girl is equal to the magnitude of the momentum of the
ball but in the opposite direction.

Video Video

Perlanggaran Kenyal Perlanggaran tak Kenyal


Elastic Collision Inelastic Collision
https://goo.gl/bqjBDV https://goo.gl/FD9Euv

© Nilam Publication Sdn. Bhd. 50


MODUL • Fizik TINGKATAN 4

Perlanggaran kenyal / Elastic collision Perlanggaran tak kenyal / Inelastic collision


u1 u2 v1 v2 u1 u2 v
m1 + m2

m1 m2 m1 m2 m1 m2 m1 m2

Sebelum perlanggaran Selepas perlanggaran Sebelum perlanggaran Selepas perlanggaran


Before collision After collision Before collision After collision

• Kedua-dua objek bergerak secara berasingan • Kedua-dua objek bergabung dan bergerak bersama
dengan halaju masing-masing selepas perlanggaran. dengan satu halaju sepunya selepas perlanggaran.
Both objects move separately at their respective The two objects combine and move together with a
velocities after the collision. common velocity after the collision.
• Jumlah momentum diabadikan .
• Jumlah momentum diabadikan .
Total momentum is conserved .
Total momentum is conserved .
• Jumlah tenaga diabadikan .
• Jumlah tenaga diabadikan .
Total energy is conserved .
Total energy is conserved .
• Tenaga kinetik diabadikan .
U
• Tenaga kinetik tidak diabadikan . N
I
Kinetic energy is conserved . T
Kinetic energy is not conserved .
2
u1 u2 v1 v2 u1 u2 v

m1 m2 m1 m2 m1 m2 m1 m2
Sebelum perlanggaran Selepas perlanggaran Sebelum perlanggaran Selepas perlanggaran
Before collision After collision Before collision After collision

Tuliskan persamaan yang menghubungkaitkan jumlah Tuliskan persamaan yang menghubungkaitkan jumlah
momentum sebelum perlanggaran dengan jumlah momentum sebelum perlanggaran dengan jumlah
momentum selepas perlanggaran: momentum selepas perlanggaran:
Write equation which relates the total momentum before Write equation which relates the total momentum before
collision with the total momentum after collision: collision with the total momentum after collision:
m1u1 + m2u2 = m1v1 + m2v2 m1u1 + m2u2 = (m1 + m2 )v

Letupan / Explosion
Troli pegun
Stationary trolleys
Sebelum letupan, kedua-dua objek bercantum bersama dan berada dalam
keadaan rehat. Selepas letupan, kedua-dua objek bergerak pada arah yang
Pin bertentangan.
Before explosion, both of the objects stick together and are at rest. After explosion,
both objects move at opposite directions.
m2 m1

Jumlah momentum sebelum Jumlah momentum selepas letupan


Sebelum letupan / Before explosion
letupan adalah sifar. Total momentum after explosion
v2 v1 The total momentum before explosion = m1v1 + m2v2
is zero.
m2 m1

Selepas letupan / After explosion

51 © Nilam Publication Sdn. Bhd.


MODUL • Fizik TINGKATAN 4

(Catatan: v2 bernilai negatif) Daripada prinsip keabadian momentum:


(Remarks: v2 has a negative value) From the principle of conservation of momentum:

Apakah yang dimaksudkan dengan Jumlah momentum sebelum Jumlah momentum selepas
nilai negatif bagi v2? perlanggaran = perlanggaran
What is mean with negative value of v2? Total momentum before collision Total momentum after collision
Arah bertentangan
Terbitkan persamaan untuk letupan: / Derive an equation for explosion:
Opposite direction
0 = m1v1 + m2v2
m1v1 = –m2v2

Huraikan aplikasi prinsip keabadian momentum


Describe applications of the principle of conservation of momentum
m1 + m2
Pegun / Stationary • Apabila selaras senapang ditembak, sebutir peluru yang berjisim m2
u=0 bergerak dengan halaju tinggi, v2. Ini menghasilkan suatu momentum
ke arah hadapan .
(a) Sebelum letupan When a rifle is fired, the bullet of mass m2 moves with a high velocity, v2.
Before explosion Peluru forward
Bullet This creates a momentum in the direction.
U v1
N m1 • Daripada prinsip keabadian momentum, suatu momentum yang sama
I
T v2 tetapi bertentangan arah dihasilkan supaya senapang itu tersentak ke

2 m2
belakang .
From the principle of conservation of momentum, an equal but opposite
(b) Selepas letupan
After explosion momentum is produced to recoil the rifle backward .
Catatan / Remarks:
Jisim senapang / Mass of rifle = m1
Jisim peluru / Mass of bullet = m2
Selepas letupan / After explosion:
v1 = Halaju senapang / Velocity of rifle
v2 = Halaju peluru / Velocity of bullet

Pelancaran roket.
The launching of rocket.
• Jumlah momentum sebelum letupan ialah sifar .
Bahan api cecair The total momentum before explosion is zero .
Liquid fuels oksigen
• Bahan api bercampur dengan .
Pengoksida cecair Fuel mixed with oxygen .
Liquid oxidiser
• Campuran itu terbakar dengan letupan di dalam kebuk pembakaran.
Pam
The mixture is burn explosively in the combustion chamber.
Pump • Gas ekzos dipancutkan ke arah bawah dengan halaju yang tinggi.
The exhaust gas is expelled downward with high velocity.
Kebuk pembakaran • Momentum yang besar ke arah bawah dihasilkan.
Combustion chamber A large downward momentum is produced.
• Momentum yang besar ke arah atas dengan magnitud yang sama
Gas ekzos dihasilkan.
Exhaust gas
A large upward momentum with the same magnitude is produced.
• Jumlah momentum terabadi .
Total momentum is conserved .

© Nilam Publication Sdn. Bhd. 52


MODUL • Fizik TINGKATAN 4

Huraikan aplikasi prinsip keabadian momentum linear


Describe applications of the principle of conservation of linear momentum
Gas panas Aplikasi dalam enjin jet: / Application in the jet engine:
Hot gas
• Suatu gas panas yang berkelajuan tinggi dipancut keluar dari belakang
dengan momentum tinggi .
A high-speed hot gas is ejected from the back with high momentum .
• Ini menghasilkan mementum yang sama tetapi bertentangan
arah untuk menolak jet bergerak ke hadapan. / This produces an
equal but opposite momentum to propel the jet plane forward.
Jet

Dalam kawasan paya, suatu bot berkipas digunakan.


Gerakan udara ke belakang In a swamp area, a fan boat is used.
Movement of air backwards
• Kipas itu menghasilkan gerakan udara berkelajuan tinggi ke
belakang . Ini menghasilkan suatu momentum yang besar ke
belakang. / The fan produces a high speed movement of air backwards .
This produces a large momentum backwards. U
• Dengan keabadian momentum, suatu momentum yang sama N
I
tetapi bertentangan arah dihasilkan dan ditindakkan ke atas bot itu. T

Jadi, bot itu akan bergerak ke hadapan . 2


Bot berkipas By conservation of momentum, an equal
but opposite momentum is
Fan boat
produced and acts on the boat. So the boat will move forward .

Sotong bergerak ke hadapan Seekor sotong bergerak dengan mengeluarkan cecair pada halaju
Squid move forwards
yang tinggi. Air masuk melalui pembukaan yang besar dan keluar melalui
tiub yang kecil. Air dipaksa keluar pada kelajuan tinggi ke belakang.
Magnitud momentum air dan sotong adalah sama tetapi pada arah
yang bertentangan. Ini menyebabkan sotong itu bergerak ke hadapan .
A squid propels by expelling a liquid at high velocity . Water enters through
a large opening and exits through a small tube. The water is forced out at a high
Cecair bergerak ke belakang speed backward. The magnitude of the momentum of water and squid are
Liquid move backwards
equal but opposite in direction. This causes the squid to move forward .
Sotong / Squid

Pelancaran roket air. / The launching of water rocket.


• Udara di dalam botol dimampatkan
Muncung
Nozzle Botol (dipam dengan pam angin).
Udara
plastik
Air inside the bottle is compressed (it is pumped with air pump).
Plastic
dimampatkan
bottle • Tekanan di dalam botol menjadi lebih tinggi.
Compressed air
Rintangan Pressure
Air udara inside the bottle becomes higher.
Water Air ditolak halaju
Sayap resistance
• Air di dalam botol keluar dengan yang tinggi.
Wing Water inside the bottle is pushed out with high velocity .
• Momentum ke arah bawah dihasilkan.
Air dipaksa keluar
Water forced out Downward momentum is produced.
• Momentum ke arah atas dengan magnitud yang sama dihasilkan.
Berat Upward momentum with the same magnitude is produced.
Weight
• Jumlah momentum terabadi . / Total momentum is conserved .

53 © Nilam Publication Sdn. Bhd.


MODUL • Fizik TINGKATAN 4

Menyelesaikan masalah melibatkan momentum linear


Solve problems involving linear momentum

Contoh / Examples 1
Kereta A yang berjisim 1 000 kg bergerak pada 20 m s–1 berlanggar dengan kereta B yang berjisim 1 200 kg
dan bergerak pada 10 m s–1 dalam arah yang sama. Akibatnya, kereta B, bergerak ke hadapan pada 15 m s–1.
Berapakah halaju, v, bagi kereta A sebaik sahaja selepas perlanggaran?
Car A of mass 1 000 kg moving at 20 m s–1 collides with car B of mass 1 200 kg moving at 10 m s-1 in the same direction.
The car B is shunted forwards at 15 m s–1 by the impact. What is the velocity, v, of car A immediately after the collision?

uA = 20 m s-1 uB = 10 m s-1

A B

m1 = 1 000 kg m2 = 1 200 kg

Penyelesaian / Solution
Jumlah momentum sebelum perlanggaran = Jumlah momentum selepas perlanggaran
Total momentum before collision = Total momentum after collision
U
N (1 000 kg)(20 m s–1) + (1 200 kg)(10 m s–1) = (1 000 kg)v + (1 200 kg)(15 m s–1)
I
T 20 000 kg m s–1 + 12 000 kg m s–1 = (1 000 kg)(v) + 18 000 kg m s–1
2 (1 000 kg)(v)
∴v
=
=
14 000 kg m s–1
14 m s–1

Contoh / Examples 2

Sebiji bola yang berjisim 5 kg dibaling pada halaju 20 km j–1 kepada Lily yang berjisim 60 kg pada keadaan rehat
di atas ais. Lily menangkap bola itu dan kemudian menggelongsor dengan bola di atas ais. Tentukan halaju Lily
dengan bola selepas perlanggaran.
A 5 kg ball is thrown at a velocity of 20 km h–1 towards Lily whose mass is 60 kg at rest on ice. Lily catches the ball and
subsequently slides with the ball across the ice. Determine the velocity of Lily and the ball together after the collision.
m1 = 5 kg m2 = 60 kg
u1 = 20 km j-¹ u2 = 0 km j-¹ m1 = 5 kg m2 = 60 kg

v=?

Penyelesaian / Solution
Jumlah momentum sebelum perlanggaran = Jumlah momentum selepas perlanggaran
Total momentum before collision = Total momentum after collision
(5 kg)(20 km j–1) + (60 kg)(0 km j–1) = (5 + 60) kg × v
(100 + 0) kg km j–1 = (65 kg)v
∴ v = 1.54 km j–1

© Nilam Publication Sdn. Bhd. 54


MODUL • Fizik TINGKATAN 4

Contoh / Examples 3
Sebuah trak yang berjisim 1 200 kg bergerak pada 30 m s–1 berlanggar dengan sebuah kereta yang berjisim 1 000 kg
yang bergerak dalam arah bertentangan pada 20 m s–1. Selepas perlanggaran, kedua-dua kenderaan itu bergerak
bersama. Berapakah halaju kedua-dua kenderaan itu sebaik sahaja selepas perlanggaran?
A truck of mass 1 200 kg moving at 30 m s–1 collides with a car of mass 1 000 kg which is traveling in the opposite direction
at 20 m s–1. After the collision, the two vehicles move together. What is the velocity of both vehicles immediately after collision?
30 m s-¹ 20 m s-¹ v

(a) Sebelum perlanggaran (b) Selepas perlanggaran


Before collision After collision

Penyelesaian / Solution
Jumlah momentum sebelum perlanggaran = Jumlah momentum selepas perlanggaran
Total momentum before collision = Total momentum after collision
(1 200 kg)(30 m s–1) + (1 000 kg)(–20 m s–1) = (1 200 + 1 000)kg × v
(36 000 – 20 000) kg m s–1 = (2 200 kg)v U
N
(2 200 kg)v = 16 000 kg m s–1 I
∴v = 7.27 m s–1 T

Contoh / Examples 4

Seorang lelaki menembak sepucuk pistol yang berjisim 1.5 kg. Jika peluru itu berjisim 10 g dan mempunyai halaju
300 m s–1 selepas tembakan, berapakah halaju sentakan pistol itu? / A man fires a pistol which has a mass of 1.5 kg.
If the mass of the bullet is 10 g and it has a velocity of 300 m s–1 after shooting, what is the recoil velocity of the pistol?
v 300 m s-¹

1.5 kg 10 g
Pegun / Stationary

(a) Sebelum tembakan (b) Selepas tembakan


Before shooting After shooting

Penyelesaian / Solution
Jumlah momentum sebelum tembakan = Jumlah momentum selepas tembakan
Total momentum before explosion = Total momentum after explosion
0 kg m s–1 = (1.5 kg)(v) + (0.010 kg)(300 m s–1)
(1.5 kg) (v) = –3.0 kg m s–1
∴ v = –2.0 m s–1

55 © Nilam Publication Sdn. Bhd.


MODUL • Fizik TINGKATAN 4

Contoh / Examples 5 Contoh / Examples 6


Bayangkan anda berlegar di sebelah kapal angkasa Seekor ikan yang besar yang berjisim 3m
pada orbit bumi dan rakan anda yang sama bergerak dengan 2 m s–1 bertemu seekor ikan kecil
jisim bergerak dengan 4 km j–1 (dengan merujuk yang berjisim m dalam keadaan rehat. Ikan besar itu
kepada kapal angkasa) melanggar anda. Jika dia menelan ikan kecil dan meneruskan gerakan dengan
memegang anda, berapakah kelajuan anda bergerak kelajuan yang berkurang. Jika jisim ikan besar
(dengan merujuk kepada kapal angkasa itu)? adalah tiga kali ganda jisim ikan kecil, berapakah
Imagine that you are hovering next to a space shuttle in halaju ikan besar selepas menelan ikan kecil itu?
earth orbit and your buddy of equal mass who is moving A large fish of mass 3 m is in motion at 2 m s–1 when
at 4 km h–1 (with respect to the ship) bumps into you. If it encounters a smaller fish of mass m which is at rest.
she holds onto you, how fast do you move (with respect to The large fish swallows the smaller fish and continues
the ship)? in motion at a reduced speed. If the large fish has three
Sebelum perlanggaran Selepas perlanggaran times the mass of the smaller fish, then what is the
Before collison After collison speed of the large fish after swallowing the smaller fish?
m m m
m Sebelum perlanggaran Selepas perlanggaran
Before collison After collison
3m m

U Dalam gerakan Dalam keadaan Dalam gerakan bersama


N
I In motion rehat pada laju yang sama
T u1 = 4 km j–1 At rest In motion together Dalam gerakan / In motion Rehat

2
u2 = 0 km j–1 at the same speed u1 = 2 m s–1 At rest

Penyelesaian / Solution Penyelesaian / Solution


Jumlah momentum Jumlah momentum Jumlah momentum Jumlah momentum
sebelum perlanggaran = selepas perlanggaran sebelum perlanggaran = selepas perlanggaran
Total momentum before Total momentum after Total momentum before Total momentum after
collision collision collision collision
(m kg)(4 km j–1) + (m kg)(0 km j–1) = (m + m) kg × v [(3m) kg × (2 m s–1)] + 0 = (3m + m) kg × v
(4m) kg km j–1 + 0 = (2m) kg × v (6m) kg m s–1 = (4m) kg × v
(4m) kg km j–1 ∴ 4v = 6 m s–1
∴v= (2m) kg = 2 km j–1
v = 1.5 m s–1

Huraikan apa yang dilakukan oleh penjaga gol sebelum dia menendang bola itu.
Describe what the goalkeeper does before kicking the ball.
• Penjaga gol itu akan mengambil beberapa langkah ke belakang dan kemudian
berlari ke hadapan untuk menendang bola itu.
The goalkeeper takes a few steps backwards and then runs forward to kick the ball.

• Bola itu akan bergerak lebih jauh / lebih cepat apabila ditendang semasa berlari
berbanding dengan tendangan dari kedudukan pegun.
The ball goes further / faster when kicked while running compared to kicking from a
standing position.

• Ini adalah disebabkan seorang pemain bola sepak yang berlari mempunyai momentum yang besar dan
momentumnya dipindahkan kepada bola.
This is because a running football player has a large momentum and his momentum is transferred to the ball.

© Nilam Publication Sdn. Bhd. 56


MODUL • Fizik TINGKATAN 4

Eksperimen Keabadian Momentum


Experiment Conservation of Momentum

Untuk menunjukkan jumlah momentum bagi suatu sistem tertutup adalah malar dalam perlanggaran
Tujuan
Aim tak kenyal.
To show that the total momentum of a closed system is constant in an inelastic collision.

Radas Jangka masa detik, pita detik, plastisin, pita selofan, troli, landasan, bekalan kuasa a.u. 12 V.
Apparatus Ticker timer, ticker tape, plasticine, cellophane tape, trolleys, runway, 12 V ac power supply.

1 Dirikan satu landasan dengan mengubah suai kecerunannya supaya landasan terpampas geseran
di mana troli boleh bergerak turun landasan dengan halaju malar.
Set up a runway and adjust the slope to compensate for friction where the trolley moves down the
runway with constant velocity.
2 Letakkan plastisin pada troli P dan Q supaya mereka akan melekat antara satu sama lain
semasa perlanggaran.
Fix plasticine on trolleys, P and Q so that they can stick together upon collision.
Jangka masa detik
Pita detik Troli P Landasan
Ticker timer Plastisin
Ticker tape Trolley P terpampas
Plasticine Troli Q
Trolley Q geseran U
Friction- N
Bekalan kuasa compensated I
Power supply runway T

Blok kayu/Wooden block 2


3 Pita detik diletakkan melalui jangka masa detik dan dilekatkan pada troli P.
A ticker tape is passed through the ticker timer and is attached to trolley P.
4 Mulakan jangka masa detik dan tolak troli P supaya ia bergerak menuruni landasan dan
berlanggar dengan troli Q, yang berada dalam keadaan rehat.
Start the ticker timer and give trolley P a push so that it will move down the runway and collide with
trolley Q, which is at rest.
Prosedur
Procedure
5 Daripada pita detik yang diperoleh, tentukan dan ukurkan halaju berikut.
From the ticker tape obtained, determine and measure the following velocities.
(a) Halaju troli P sebelum perlanggaran, uP / Velocity of trolley P before collision, uP
(b) Halaju troli Q sebelum perlanggaran, uQ / Velocity of trolley Q before collision, uQ
(c) Halaju troli (P + Q) selepas perlanggaran, v
Velocity of trolley (P + Q) after collision, v
6 Langkah-langkah 2 – 5 diulangi dengan jisim P dan jisim Q yang berbeza seperti ditunjukkan
dalam jadual di bawah.
Steps 2 – 5 are repeated for different masses of P and Q as shown in the table below.

Sebelum perlanggaran Selepas perlanggaran


Before collision After collision

Jisim troli Q Jumlah Jumlah


Jisim troli P Halaju P Halaju sepunya
Mass of momentum momentum
Mass of trolley P Velocity of P Common velocity
trolley Q Total momentum Total momentum
kg m s–1 m s–1
kg kg m s –1
kg m s–1
1 1
2 1
1 2
2 2
3 2

57 © Nilam Publication Sdn. Bhd.


MODUL • Fizik TINGKATAN 4

Contoh: Perlanggaran tak kenyal antara dua troli


Example: Inelastic collision between two trolleys
Perlanggaran berlaku di sini
Arah gerakan / Direction of motion
Collision
18.4 cm occurs here 9.2 cm

Sebelum perlanggaran Selepas perlanggaran


Before collision After collision

Kuantiti fizik Sebelum perlanggaran Selepas perlanggaran


Physical quantity Before collision After collision

Panjang 10 detik
18.4 cm 9.2 cm
Keputusan 10-tick length
Result
Masa diambil untuk 10 detik
0.2 s 0.2 s
Time taken for 10 ticks

Halaju 18.4 cm 9.2 cm


Velocity
= 0.92 m s–1 = 0.46 m s–1
0.2 s 0.2 s
U
N
I Jisim troli
T (jisim 1 troli = 1 kg) 1 kg 2 kg
2 Mass of trolley
(mass of 1 trolley = 1 kg)

Momentum (1 kg)(0.92 m s–1) (2 kg)(0.46 m s–1)


Momentum = 0.92 kg m s–1 = 0.92 kg m s–1

1 Bandingkan jumlah momentum sebelum perlanggaran dan selepas perlanggaran.


Compare the total momentum before collision and after collision.
Jumlah momentum sebelum dan selepas perlanggaran adalah sama.
The total momentum before collision and after collision are equal.

2 Nyatakan satu kesimpulan. / State a conclusion.


Tanpa kehadiran daya luar, jumlah momentum sebelum perlanggaran adalah sama
Perbincangan dengan jumlah momentum selepas perlanggaran.
Discussion
In the absence of any external force, the total momentum before collision is equal to total
momentum after collision.

3 Apakah tujuan utama mengubah suai landasan supaya landasan terpampas geseran?
What is the main purpose of adjusting the runway so that it is friction-compensated?
Troli bergerak dengan halaju malar.
The trolley moves with constant velocity.

Maklumat tambahan:
Additional information:

© Nilam Publication Sdn. Bhd. 58


MODUL • Fizik TINGKATAN 4

MEMAHAMI KESAN DAYA


2.5 UNDERSTANDING THE EFFECTS OF A FORCE

Huraikan kesan daya seimbang yang bertindak ke atas objek


Describe the effect of balanced forces acting on an object
Daya seimbang / Balanced force
Apabila daya-daya yang bertindak ke atas suatu objek dalam keadaan seimbang, ia akan membatalkan antara satu
sama lain. Daya bersih adalah sifar .
When the forces acting on an object are balanced, they canceling each other. The net force is zero .

Kesan / Effect:
Objek berada dalam keadaan rehat [halaju = 0] atau bergerak pada halaju malar [pecutan = 0]
The object is at rest [velocity = 0] or moves at constant velocity [acceleration = 0]

Daya dikenakan oleh meja ke atas cawan Daya angkat, U / Lift, U


Force exerted by table on the cup

U
Tujahan, F N
Seretan, G I
Drag, G Thrust, F T

2
Berat
Weight Berat, W
Weight, W
Cawan itu berada dalam keadaan rehat. Daya bersih Kapal terbang bergerak dengan halaju malar. Daya bersih
yang bertindak ke atasnya adalah sifar. yang bertindak ke atasnya adalah sifar.
The cup stays at rest. The net force acting on it is zero.
The plane moves with constant velocity. The net force acting
W = R, di mana / where on it is zero.
W : Berat / Weight W = U, di mana / where F = G, di mana / where
R : Tindak balas normal / Normal reaction W : Berat / Weight F : Tujahan / Thrust
U : Daya angkat / Lift G : Seretan / Drag

Kesan daya tidak seimbang yang bertindak ke atas suatu objek


The effects of unbalanced forces acting on an object

Apabila daya yang bertindak ke atas objek tidak seimbang, terdapat daya bersih yang bertindak ke atasnya.
When the forces acting on an object are not balanced, there must be a net force acting on it.
Daya bersih dikenali sebagai daya paduan yang bertindak ke atasnya.
The net force is known as the resultant force acting on it.

Daya tidak seimbang = daya bersih = daya yang dikenakan – daya geseran
The unbalanced force = net force = force applied – frictional force

Kesan: Boleh menyebabkan suatu jasad / Effect: Can cause a body to


• bertukar keadaan rehatnya (objek itu akan memecut) / Change its rest state (an object will accelerate)
• bertukar keadaan gerakannya (suatu objek yang bergerak akan memecut / nyahpecut atau menukar arahnya)
change its state of motion (a moving object will accelerate / decelerate or change its direction)

59 © Nilam Publication Sdn. Bhd.


MODUL • Fizik TINGKATAN 4

Menentukan hubungan antara daya, jisim dan pecutan (F = ma)


Determine the relationship between force, mass and acceleration (F = ma)
Hukum Gerakan
Newton Kedua Pecutan yang dihasilkan oleh daya ke atas suatu objek adalah berkadar langsung
Newton’s Second dengan magnitud daya bersih yang dikenakan dan berkadar songsang dengan jisim
Law of Motion objek itu.
The acceleration produced by a force on an object is directly proportional to the magnitude
of the net force applied and is inversely proportional to the mass of the object.
F=5N
Daya = Jisim × Pecutan Force = Mass × Acceleration
F = ma F = ma
m = 25 kg

aαF a
Hubungan antara a dan F Pecutan, a, berkadar langsung dengan daya yang
Relationship between dikenakan, F
a and F The acceleration, a, is directly proportional to the applied force, F
0 F

U 1 a

N
Hubungan antara a dan m m
I
T Relationship between Pecutan, a, bagi suatu objek berkadar songsang dengan
2 a and m jisimnya, m / The acceleration, a, of an object is inversely
proportional to its mass, m 0
1
m

Eksperimen Mencari hubungan antara daya, jisim dan pecutan


Experiment Find the relationship between force, mass and acceleration

Hubungan antara 1
aαF aα
Relationship between m

A A

B B

Situasi
Situation
Dua orang pemuda menolak jisim yang sama Dua orang pemuda mengenakan daya yang
tetapi pemuda A menolak dengan daya yang sama. Tetapi pemuda B bergerak dengan lebih
lebih besar. Jadi dia bergerak dengan lebih cepat daripada pemuda A.
cepat. Two men exerted the same force. But man B moves
Two men pushing the same mass but man A pushes faster than man A.
with a greater force. So, he moves faster.

Inferens Pecutan, a, bergantung kepada daya yang Pecutan, a, bergantung kepada jisim objek,
Inference dikenakan, F m
The acceleration, a, depends on applied force, F The acceleration, a, depends on mass of the object, m

Semakin besar daya, semakin besar pecutan. Semakin besar jisim, semakin kecil pecutan.
Hipotesis
Hypothesis The larger the force, the greater the acceleration. The greater the mass, the smaller the acceleration.

Untuk mengkaji hubungan antara pecutan, a Untuk mengkaji hubungan antara pecutan, a
Tujuan dan daya yang dikenakan, F. dan jisim objek, m.
Aim To investigate the relationship between acceleration, a To investigate the relationship between acceleration, a
and applied force, F. and object mass, m.

© Nilam Publication Sdn. Bhd. 60


MODUL • Fizik TINGKATAN 4

Pemboleh ubah Daya yang dikenakan ke atas objek


dimanipulasikan Jisim / Mass
Manipulated variable Force applied on the object

Pemboleh ubah
bergerak balas Pecutan / Acceleration Pecutan / Acceleration
Responding variable

Pemboleh ubah Daya yang dikenakan ke atas objek


dimalarkan Jisim / Mass
Constant variable Force applied on the object

Jangka masa detik dan pita detik, bekalan kuasa, landasan terpampas geseran,
Bahan dan radas pembaris, troli, takal licin (dengan pengapit), tali tak kenyal, pemberat berslot
Materials and
apparatus Ticker timer and ticker tape, power supply, friction-compensated runaway, ruler, trolley,
smooth pulley (with clamp), inelastic string, slotted weights

Jangka masa detik


Pita detik Ticker timer
Ticker tape Troli P
Trolley P Tali tak kenyal U
Inelastic string N
Bekalan kuasa a.u. Takal licin I
a.c. power supply Smooth pulley T

2
Rajah Landasan terpampas
Diagram Blok kayu Pemberat
Wooden block geseran
Friction-compensated berslot
runway Slotted
weight

Susunan radas untuk mengkaji hubungan antara Arrangement of apparatus to


(1) daya dan pecutan investigate the relationship between
(2) jisim dan pecutan (1) force and acceleration
(2) mass and acceleration

1 Radas disusun seperti ditunjukkan dalam 1 Radas disusun seperti ditunjukkan dalam
rajah di atas. rajah di atas.
The apparatus is set up as shown in the diagram The apparatus is set up as shown in the diagram
above. above.
2 Sebuah troli berjisim 1.0 kg (jisim malar) 2 Sebuah troli dengan jisim , m = 1.0 kg
diletakkan di atas landasan. Pita detik diletakkan di atas landasan. Pita detik
dilekatkan pada troli itu. dilekatkan pada troli itu.
A trolley of mass 1.0 kg (constant mass) is A trolley of mass, m = 1.0 kg is placed on the
placed on the runway. A length of ticker tape runway. A length of ticker-tape is attached to
is attached to the trolley. the trolley.

Prosedur 3 Jangka masa detik dihidupkan dan troli 3 Jangka masa detik dihidupkan dan troli itu
Procedure itu ditarik oleh pemberat yang mempunyai ditarik oleh pemberat (daya malar pemberat
daya, F = 10.0 N. ini ialah 10 N)
The ticker timer is switched on and the trolley The ticker timer is switched on and the trolley
is pulled by a weight of force, is pulled by a weight of constant force, 10 N
F = 10.0 N. 4 Dari pita detik yang diperoleh, pecutan troli
4 Dari pita detik yang diperoleh, pecutan troli dihitung dengan menggunakan formula,
dihitung dengan menggunakan formula, (v – u)
a= .
(v – u) t
a= From the ticker tape obtained, the acceleration
t
From the ticker tape obtained, the acceleration of the trolley is calculated by using the formula,
(v – u)
of the trolley is calculated by using the formula, a= .
(v – u) t
a= .
t

61 © Nilam Publication Sdn. Bhd.


MODUL • Fizik TINGKATAN 4

5 Langkah-langkah 2 – 4 diulangi dengan 5 Eksperimen diulangi dengan dua troli yang


menambahkan pemberat berslot supaya F bertindan dan seterusnya tiga dan empat
= 15.0 N, 20.0 N, 25.0 N dan 30.0 N. troli yang bertindan. / The experiment is
Steps 2 – 4 are repeated by adding slotted repeated with two trolleys that are stacked
weights to pull the trolley so that F = 15.0 N, together and then three and four trolleys that
20.0 N, 25.0 N and 30.0 N. are stacked together.

Daya, F / N Pecutan, a / cm s–2 Bilangan Jisim songsang, Pecutan,


1
Force, F / N Acceleration, a / cm s-2 troli m / kg
–1
a / cm s–2
10.0 Number Inverse of mass, Acceleration,
1
of trolleys m / kg
–1 a / cm s–2
15.0
Merekodkan data
Recording data 20.0 1
25.0 2
30.0 3
4

Pecutan, a / cm s–2 Pecutan, a / cm s–2


U Acceleration, a / cm s–2 Acceleration, a / cm s–2
N
I
T
Menganalisis data
2 Analysing data
1 1 –1
jisim , m / kg
0 Daya, F / N 0 1 1 –1
Force, F / N mass , m / kg

Menyelesaikan masalah menggunakan F = ma / Solve problems using F = ma


1 Hitungkan pecutan bagi blok di bawah: Nota / Note:
1. Perhatikan arah gerakan objek, F / Observe the direction of moving objek, F
Calculate the acceleration of the block: 2. Daya, F ke hadapan (kanan) tolak daya, F ke belakang (kiri)
The forward (right) force, F subtract the backward (left) force, F.
3. Daya yang berlainan arah perlu ditolak, daya yang sama arah ditambah / The different
direction of the force should be subtract, the same direction of the force should be multiply.

(a) m = 2 kg (c) m = 10 kg
F = 8.0 N F=2N F = 18 N

F = ma F = ma
8 N = 2 kg(a) (18 – 2) N = 10 kg(a)
8.0 N (18 – 2) N 16 N
a = a= =
2 kg 10 kg 10 kg
= 4 m s-2 / 4 N kg-1 = 1.6 m s-2 / 1.6 N kg-1

(b) m = 8 kg (d) m = 12 kg
F=6N F = 14 N F=5N F = 10 N
R=5N

F = ma F = ma
(14 + 6) N = 8 kg(a) 10 N – (5 + 5) N = 12 kg(a)
(14 + 6) N 20 N (10 – 5 – 5) N
a= = a= =0
8 kg 8 kg 12 kg
= 2.5 m s / 2.5 N kg-1
-2 = 0 m s-2

© Nilam Publication Sdn. Bhd. 62


MODUL • Fizik TINGKATAN 4

Menyelesaikan masalah menggunakan F = ma / Solve problems using F = ma


2 Seorang lelaki menolak troli yang berisi kotak (jumlah jisim 5 kg) di atas permukaan
yang licin. Jika dia menggunakan daya 30 N untuk menolak troli itu, apakah magnitud
dan arah pecutan troli itu?
A man pushes a trolley with a box (total mass 5 kg) on a smooth surface. If he uses a force of F = 30 N
30 N to move the trolley, what is the magnitude and direction of the acceleration of the trolley?
Penyelesaian / Solution:
5 kg
F = ma
30 N
a= = 6 m s-2 ke kanan / to the right.
5 kg

3 Sebuah objek yang berjisim 2 kg ditarik di atas tanah dengan daya 5 N dan halaju malar.
An object of mass 2 kg is pulled on the floor by a force of 5 N and has a constant velocity.
(a) Berapakah daya geseran antara objek dan tanah?
What is the frictional force between the object and the floor?
(b) Hitungkan pecutan objek itu jika objek itu ditarik dengan daya 17 N. R F1 = 5 N U
Calculate the acceleration of the object if the object is pulled by a 17 N force. N
I
Penyelesaian / Solution: T
R ialah daya geseran / R is the frictional force
(a)
F1 – R = ma
2
∴ R = F1 – ma
Oleh kerana halaju malar / Because the velocity is constant,
a = 0
∴ R = F1 – 0
= F1
= 5 N Maklumat tambahan:
(b) F2 – R = ma Additional information:

17 N – 5 N = (2 kg) (a)
a = 12 N = 6 m s–2
2 kg

4 Sebuah bas berjisim 2 000 kg bergerak dengan halaju seragam 40 m s-1 sejauh 2 500 m sebelum berehat.
Hitungkan
A bus of mass 2 000 kg travels at a uniform velocity 40 m s-1 for a distance of 2 500 m before it comes to rest. Calculate
(a) purata nyahpecutan bas itu. / the average deceleration of the bus.
(b) purata daya yang dikenakan oleh brek itu untuk membolehkan bas itu berhenti bergerak.
the average force applied by the brakes to bring the bus to stop.
Penyelesaian / Solution:
(a) v2 = u2 + 2as (b) F = ma
0 = (40 m s–1)2 + 2a(2 500 m) = (2 000 kg)(–0.32 m s–2)
∴ 5 000a = –1 600 m s–2 = –640 N
a = –0.32 m s-2 (Negatif bermaksud daya untuk menentang gerakan
Negative means force to resist the motion)

63 © Nilam Publication Sdn. Bhd.


MODUL • Fizik TINGKATAN 4

MENGANALISIS IMPULS DAN DAYA IMPULS


2.6 ANALYSING IMPULSE AND IMPULSIVE FORCE

Menerangkan daya impuls


Explain what an impulsive force is

Daya yang besar yang bertindak dalam tempoh masa yang singkat semasa perlanggaran atau letupan dikenali
sebagai daya impuls.
A large force that acts over a short period of time during a collision or explosion is known as an impulsive force.
Daripada hubungan antara daya, jisim dan pecutan: / From the relationship between force, mass and acceleration:
v–u
F = ma = m ( )
t
mv – mu perubahan momentum
F= = Unit = N = kg m s–2
t t
m = jisim / mass u = halaju awal / initial velocity t = masa / time v = halaju akhir / final velocity
Daya impuls ialah kadar perubahan momentum dalam perlanggaran atau letupan.
An impulsive force is the rate of change of momentum in a collision or explosion.
U
N
I
T

2
Peta Pemikiran / Thinking Maps
Impuls (Menerangkan / Mencirikan)
Impulse (Explaining / Characterising)

Perubahan momentum Formula/Formula


Change in momentum Ft = mv – mu

Impuls
Impulse

Unit:/Unit: Hasil darab daya dan


N s atau / or masa, Ft
kg m s–1 The product of force and
time, Ft

Kesan peningkatan dan pengurangan masa perlanggaran


The effects of increasing and decreasing the time of collision

Daripada formula, F = Perubahan momentum /From the formula, F =


Change of momentum
.
Masa Time

Daya impuls berkadar songsang dengan masa sentuhan atau tindakan atau perlanggaran.
Impulsive force is inversely proportional to the time of contact or impact or collision.

Tempoh masa yang panjang / Longer period of time Daya impuls kecil / Impulsive force is small
Tempoh masa yang pendek / Shorter period of time Daya impuls besar / Impulsive force is large

© Nilam Publication Sdn. Bhd. 64


MODUL • Fizik TINGKATAN 4

Melompat ke atas Memakai sarung tangan


tilam yang tebal untuk menangkap bola
Jumps on the Wearing gloves
thick mattress to catch a ball

Contoh-contoh
Situasi Di Mana
Impuls Perlu
Dikurangkan
Examples Situation
Where an Impulsive
Membengkokkan kaki Force Needs to Be Menangkap bola mengikut
U
semasa mendarat Reduced arah pergerakan bola N
Bend the legs Catch the ball in the direction of I
T
upon landing the motion of the ball
2

Bahan yang mudah pecah seperti telur, kaca dan perkakasan elektrik mestilah dibungkus dalam bahan yang lembut
dan boleh dimampatkan. Mengapa?
Items that are fragile such as eggs, glass and electrical appliances must be packed in materials that are soft and compressible.
Why?
Bahan yang lembut dan mudah dimampatkan menghasilkan masa perlanggaran yang panjang. Ia menyerap
hentakan. Jadi ia mengurangkan daya impuls.
Soft and compressible material provides longer time of impact. It absorbs the shock. So it can reduce the impulsive force.

Perbincangan
Explanations

Masa tindakan yang panjang menyebabkan daya impuls menjadi kecil. Ini akan mengelakkan
kecederaan yang serius.
The longer the time of impact will cause the impulsive force become smaller. This can prevent serious injury.

65 © Nilam Publication Sdn. Bhd.


MODUL • Fizik TINGKATAN 4

Terangkan situasi di mana daya impuls mendatangkan faedah


Explain situations where an impulsive force is beneficial

Situasi Penjelasan
Situations Explanation

Peserta karate yang mahir boleh memecahkan kayu yang tebal dengan menggunakan
sisi tangan yang bergerak dengan kelajuan yang sangat tinggi.
A karate expert can break a thick wooden slab with his bare hand that moves at a very
fast speed.

Tangan tersebut digerakkan pada halaju yang tinggi. Masa tindakan adalah singkat.
Menghasilkan daya impuls yang besar .
The hand is moved at high velocity. Time of impact is shorter. Produce large
impulsive force.

Kepala penukul yang besar bergerak pada kelajuan yang tinggi untuk memukul
U
N paku.
I A massive hammer head moving at a fast speed is brought to rest upon hitting the nail.
T

2 Masa tindakan adalah pendek apabila kepala penukul memukul pada halaju yang
tinggi. Ia akan menghasilkan daya impuls yang besar .
Time of impact is shorter when the hammer head is hit at high velocity. It produces
large impulsive force.

Sebiji bola sepak mestilah mempunyai tekanan udara yang cukup tinggi.
A football must have an air pressure that is high enough.

Sebiji bola sepak yang mempunyai tekanan udara yang cukup tinggi akan mempunyai
masa tindakan yang pendek. Maka daya impuls adalah besar . Bola
akan bergerak lebih jauh.
Daya impuls A football which has a high-enough air pressure will have a short time of impact. So, the
Impulsive force
impulsive force is large . The ball will move further.

Antan dan lesung diperbuat daripada batu.


Antan A pestle and mortar are made of stone.
Pestle
Daya impuls
Impulsive force Masa tindakan adalah kecil. Maka daya impuls adalah besar .
Time of impact is shorter. So the impulsive force is large .
Lesung/Mortar

Maklumat tambahan:
Additional information:

© Nilam Publication Sdn. Bhd. 66


MODUL • Fizik TINGKATAN 4

Latihan / Exercises
1 Rosli yang berjisim 60 kg melompat dari tingkat 2 Rooney menyepak bola dengan kekuatan daya
pertama sebuah rumah yang terbakar. Halajunya 1 500 N. Masa tindakan di antara kasut dan bola
sejurus sebelum mendarat ialah 6 m s-1. ialah 0.01 s. Berapakah impuls yang dikenakan
Rosli, with a mass of 60 kg, jumps from the first floor kepada bola itu? Jika jisim bola itu ialah 0.5 kg,
of a burning house. His velocity just before landing on berapakah halaju bola tersebut?
the ground is 6 m s-1. Rooney kicks a ball with a force of 1 500 N. The time of
(a) Kirakan impuls apabila kakinya mencecah contact of his boot with the ball is 0.01 s. What is the
impulse delivered to the ball? If the mass of the ball is
tanah. 0.5 kg, what is the velocity of the ball?
Calculate the impulse when his legs hit the ground.
Penyelesaian / Solution
(b) Berapakah daya impuls yang bertindak ke
atas kaki Rosli jika dia membengkokkan kaki Impuls/Impulse = Ft = 1 500 N × 0.01 s
ketika mendarat dan mengambil masa 0.5 s = 15.0 N s
untuk berhenti? = 15.0 kg m s-1
What is the impulsive force on Rosli’s legs if he
bends upon landing and takes 0.5 s to stop? mv – mu = 15 kg m s-1 u=0
(0.5 kg)v = 15 kg m s-1
(c) Berapakah daya impuls yang bertindak ke v = 30 m s-1
atas kaki Rosli jika dia tidak membengkokkan U
kaki dan berhenti dalam 0.05 s? 3 Dalam satu perlawanan tenis, pemain memukul N
What is the impulsive force on Rosli’s legs if he I
bola yang mempunyai jisim 0.2 kg yang menuju ke T
does not bend and stops in 0.05 s? arahnya dengan halaju 20 m s-1. Bola itu memantul
(d) Apakah kebaikan membengkokkan kaki dengan halaju 40 m s-1. Masa yang diambil semasa 2
semasa mendarat? perlanggaran antara bola dan raket tenis ialah
What is the advantage of bending his legs upon 0.01 s.
In a tennis match, a player hits an on-coming ball
landing?
with mass of 0.2 kg and velocity of 20 m s-1. The ball
Penyelesaian / Solution rebounds with a velocity of 40 m s-1. The time taken in
the collision between the ball and the tennis racket is
(a) mv – mu = m (v – u)
0.01 s.
= (60 kg) (0 – 6) m s–1
= –360 kg m s–1 u = –20 m s-1
Impuls adalah –360 kg m s–1 kerana
momentumnya dikurangkan sehingga sifar.
The impulse is –360 kg m s–1 because its v = 40 m s-1
momentum is reduced to zero.
mv – mu –360 kg m s–1 0.2 kg
(b) F = = t = 0.01 s
t 0.5 s
= –720 N
(a) Berapakah impuls yang dialami oleh bola itu?
mv – mu –360 kg m s–1
(c) F = = What is the impulse experienced by the ball?
t 0.05 s (b) Berapakah daya impuls yang dikenakan ke
= –7 200 N atas bola tenis?
(b) dan (c) : Tanda negatif bagi daya bermakna What is the impulsive force exerted on the tennis
daya ini telah menyebabkan kehilangan ball?
momentum
(b) and (c) : The negative sign to the force means that Penyelesaian / Solution
the force has caused a loss of momentum (a) Impuls = m (v – u)
(d) Dengan membengkokkan kaki semasa Impulse = (0.2 kg)[(40 – (–20)] m s–1
mendarat, dia akan meningkatkan masa
tindakan dan mengurangkan daya impuls. Jadi = 12.0 kg m s-1
ia dapat mengurangkan kecederaan. = 12.0 N s
By bending his legs upon landing, he will increase 12 N s
(b) F = = 1 200 N
the time of impact and reduce the impulsive force. 0.01 s
So it will minimise the injuries.

67 © Nilam Publication Sdn. Bhd.


MODUL • Fizik TINGKATAN 4

KESELAMATAN KENDERAAN
2.7 VEHICLE SAFETY

Beg udara / Air bag


Menyerap hentakan dengan menambahkan masa
perlanggaran apabila kepala pemandu terhentak ke
stereng. Oleh itu, daya impuls dikurangkan..
Absorbing impact by increasing the collision time when the
driver’s head is thrown towards the steering. So, the impulsive
force is reduced.

Cermin hadapan kereta / Windscreen


Kaca tahan pecah yang tidak akan mudah pecah
U
N kepada serpihan yang kecil dengan mudah semasa
I
T perlanggaran. Mengurangkan kecederaan disebabkan Stering
2 oleh serpihan kaca yang berselerak. Steering

Shatterproof glass that will not break into small pieces


easily during collision. This will reduce injuries caused by Zon remuk
depan
scattered glass. Front crumple
zone

Bumper depan / Front bumper


Menyerap hentakan akibat daripada kemalangan.
Papan pemuka
Diperbuat daripada keluli, aluminium, Dashboard
plastik, getah dan fiber komposit.
Absorb the shock from the accident. Made from steel,
aluminium, plastic, rubber and composite fibres. Semasa perlanggaran, papan D
pemuka kemek. Ini akan da
menyerap kesan hentaman w
Sistem brek anti kunci dengan meningkatkan masa in
Anti-lock braking system (ABS) perlanggaran antara kepala tim
Membolehkan pemandu memberhentikan kereta dengan pemandu dan stereng. Jadi dr
segera tanpa menyebabkan roda terkunci apabila brek ia mengurangkan daya Th
ditekan secara tiba-tiba. Mengelakkan kereta daripada impuls. fo
menggelongsor.
Enables drivers to quickly stop the car without the wheels
locking when the brake is applied suddenly.
Prevents the car from skidding.

© Nilam Publication Sdn. Bhd. 68


MODUL • Fizik TINGKATAN 4

Penyandar kepala / Headrest


Untuk mengurangkan kesan inersia terhadap kepala
pemandu. Mengurangkan kecederaan leher apabila
kereta dilanggar daripada belakang.
To reduce the inertia effect on the driver’s head. Reduce
neck injury when the car is hit from behind.

Zon remuk belakang / Rear crumple zone


U
Boleh dimampatkan ketika kemalangan. Jadi ia akan N
I
meningkatkan masa yang diperlukan kereta untuk T

berhenti sepenuhnya. Maka ia akan mengurangkan 2


daya impuls.
Can be compressed during an accident. So it can increase
the time taken by the car for a complete stop. So it
can reduce the impulsive force.

muka Tali pinggang keselamatan / Safety belt


Untuk mengurangkan kesan inersia dengan
mengelakkan pemandu daripada tercampak ke
hadapan.
an During collision, the To reduce the inertia effect by preventing the driver from
dashboard collapses. This
will absorb the impact by being thrown forward.
a increasing the
a time of collision between the
i driver's head and the steering. Bar hentaman sisi / Side impact bar
This will reduce the impulsive Menyerap hentakan dengan menambahkan masa
force. perlanggaran apabila kepala pemandu terhentak ke
stereng. Oleh itu, daya impuls dikurangkan..
Absorbing impact by increasing the collision time when the
driver’s head is thrown towards the steering. So, the impulsive
force is reduced.

69 © Nilam Publication Sdn. Bhd.


MODUL • Fizik TINGKATAN 4

MEMAHAMI GRAVITI
2.8 UNDERSTANDING GRAVITY

Kekuatan medan graviti / Gravitational field strength


Objek
• Maksud medan graviti : terapung
1 kg
Lingkungan kawasan di mana suatu objek berada dan mengalami pegun
daya tarikan graviti. 1 kg
0N Stationary
floating
Meaning of gravitational field : 10 N object
Scope of area where an object be and experiences gravitational force.
Medan
• Objek berjisim 1 kg yang diletakkan di dalam medan graviti graviti
mengalami daya tarikan graviti sebanyak 10 N menghala ke arah Gravitational
pusat Bumi. / An object of mass 1 kg placed in gravitational field field
experiences gravitational force 10 N towards the centre of the Earth.
• Kekuatan medan graviti di Bumi : 10 N kg–1
The strength of gravitational field in Earth : 10 N kg–1 .
• Maksud kekuatan medan graviti : Daya tarikan graviti yang bertindak
U
ke atas 1 kg objek yang diletakkan di dalam medan graviti.
N Meaning of strength of gravitational field : Gravitational force acting on 1 kg object placed in gravitational field.
I
T daya tarikan graviti
• Kekuatan medan graviti =
2 jisim
gravitational force
Gravitational field strength =
mass

• Objek yang berada di luar lingkungan kawasan medan graviti, daya tarikan graviti ialah sifar .
An object placed at outside the scope of gravitational field, gravitational force is zero .

Aktiviti 1: Pecutan disebabkan graviti


Activity 1: Acceleration due to gravity

Rajah di sebelah menunjukkan gambar foto stroboskop bagi bola yang jatuh bebas dan Bola
graf halaju lawan masa bagi gerakannya. / The diagram on the right shows a stroboscopic Ball
photograph of a free falling ball and its velocity-time graph.
(a) Perhatikan gambar foto dan terangkan halaju bola.
Observe the photograph and describe the velocity of the ball.
Halaju bola itu meningkat dengan seragam .
The velocity of the ball increases uniformly . Lantai
Floor
(b) Apakah yang boleh anda simpulkan daripada graf kecerunan v – t?
What can you deduce from the gradient of the v – t graph?
V
Kecerunan ialah pecutan bola itu .
The gradient is the acceleration of the ball .
(c) Terangkan gerakan bola tersebut.
Describe the motion of the ball. 0 t

Bola tersebut bergerak dengan pecutan seragam .


The ball moves with constant acceleration .

© Nilam Publication Sdn. Bhd. 70


MODUL • Fizik TINGKATAN 4

Terangkan pecutan yang disebabkan oleh graviti, g


Explain acceleration due to gravity, g

Pecutan disebabkan oleh graviti, g, ialah pecutan bagi objek yang disebabkan oleh kekuatan medan graviti.
Acceleration due to gravity, g, is the acceleration of an object due to the pull of the gravitational field strength.
Nilai piawai bagi pecutan graviti, g, ialah 9.81 m s-2. Nilai g yang sering digunakan ialah 10 m s-2.
Magnitud bagi pecutan yang disebabkan oleh graviti bergantung pada kekuatan medan graviti .
The standard value of the gravitational acceleration, g, is 9.81 m s-2. The value of g is often taken to be 10 m s-2
for simplicity. The magnitude of the acceleration due to gravity depends on the gravitational field strength .

Apakah jatuh bebas?


What is free fall?

Objek dikatakan 'jatuh bebas' apabila ia jatuh di bawah kekuatan medan graviti sahaja.
An object is free falls when it is falls under the gravitational field strength only.
U
Sehelai kertas tidak jatuh bebas kerana kejatuhannya dipengaruhi oleh rintangan udara. N
I
A piece of paper does not fall freely because its fall is affected by air resistance. T

Objek hanya jatuh bebas di dalam vakum . Ketiadaan udara bermaksud tiada rintangan udara yang 2
menentang pergerakan objek. / An object falls freely only in vacuum . The absence of air means there is no
air resistance to resist the motion of the object.

Di dalam vakum, kedua-dua objek yang ringan dan berat jatuh bebas. Ia jatuh dengan pecutan graviti
iaitu pecutan disebabkan oleh graviti, g.
In vacuum, both light and heavy objects fall freely. They fall with the gravitational acceleration, that is the acceleration
due to gravity , g.

Aktiviti 2: Pecutan disebabkan graviti / Activity 2: Acceleration due to gravity

Pegang dua biji batu yang berbeza saiz pada ketinggian yang sama, kemudian kedua-dua batu itu dijatuhkan
serentak daripada ketinggian yang sama.
Hold two stones of different sizes at the same height and then drop both stones simultaneously from the same height.
(a) Huraikan bagaimana halaju berubah. / Describe how the velocity changes.
Halaju meningkat dengan seragam.
The velocity increases uniformly.

(b) Bandingkan masa yang diambil untuk batu mencecah lantai.


Compare the time taken for the stones to reach the floor.
Sama / Same

(c) Adakah pecutan batu dipengaruhi oleh jisimnya? / Is the acceleration of each stone influenced by its mass?
Jisim tidak mempengaruhi pecutan.
Mass does not affect the acceleration.

71 © Nilam Publication Sdn. Bhd.


MODUL • Fizik TINGKATAN 4

Aktiviti 3: Yang mana satukah mencecah tanah dahulu?


Activity 3: Which one reaches the ground first?

Bola golf Kertas Bola golf Kertas yang direnyukkan


Golf ball Paper Golf ball Paper which is crumpled

Bola golf dan sehelai kertas dipegang pada ketinggian Ulangi dengan bola golf dan sehelai kertas yang
yang sama dan dijatuhkan serentak. / Hold a golf ball direnyukkan. / Repeat with a golf ball and a piece of
and a piece of paper at the same height and drop them paper which is crumpled.
simultaneously. (a) Objek yang manakah mencecah tanah dahulu?
(a) Objek yang manakah mencecah tanah dahulu? Which object reaches the floor first?
Which object reaches the floor first? Kedua-duanya mencecah tanah pada masa yang
Bola golf. / The golf ball.
U
sama. / Both reach the floor at the same time.
N (b) Terangkan mengapa. / Explain why.
I (b) Terangkan mengapa. / Explain why.
T Kertas mempunyai luas permukaan yang besar.
Kedua-dua objek mempunyai saiz dan luas
2 Jadi lebih banyak rintangan udara yang
permukaan yang sama. Jisim tidak memberi
bertindak ke atasnya.
kesan kepada pecutan graviti.
The paper has large surface area. As such, the air
Both objects have same size and surface area.
resistance acting on it is greater.
Mass does not affect gravitational acceleration.

Aktiviti 4: Perbezaan antara jatuh bebas di atmosfera (udara) dan jatuh bebas di dalam vakum
bagi duit syiling dan bulu pelepah.
Activity 4: The difference between free fall in atmosphere and free fall in a vacuum of a coin and a feather.

(a) Duit syiling dan bulu pelepah dilepaskan dari ketinggian yang sama
secara serentak di dalam makmal. / A coin and a feather are released from
the same height simultaneously in the laboratory.

Pemerhatian / Observation
Duit syiling jatuh lebih cepat daripada bulu pelepah.
The coin falls faster than the feather.

(b) Duit syiling dan bulu pelepah yang sama diletakkan di dalam satu tiub
vakum dan kemudian dijatuhkan serentak pada ketinggian yang sama.
The same coin and feather are put into a vacuum tube and then dropped
simultaneously from the same height.
(a) (b)
Pemerhatian / Observation
Kedua-dua objek mencecah ke bawah silinder pada masa yang sama.
Both objects reach the bottom of the cylinder at the same time.

© Nilam Publication Sdn. Bhd. 72


MODUL • Fizik TINGKATAN 4

Penjelasan (a) / Explanation (a) Penjelasan (b) / Explanation (b)

Rintangan udara yang besar bertindak ke atas Di dalam keadaan vakum, tiada rintangan udara .
Hanya terdapat satu daya yang bertindak ke atas objek
bulu pelepah kerana ia mempunyai luas permukaan
iaitu daya graviti .
yang besar.
air resistance In vacuum, there is no air resistance . The only force
A bigger acts the feather because it
surface area . acting on both objects is the force of gravity .
has a large

graviti Kedua-dua objek jatuh bebas dengan pecutan


Daya pada duit syiling mampu untuk
rintangan udara yang disebabkan graviti walaupun berbeza dari
mengatasi lebih baik daripada
segi jisim dan bentuk .
bulu pelepah.
free fall
The gravitational force on the coin is able to overcome Both objects with acceleration due to
air resistance gravity despite the differences in their mass and
better than the feather.
shapes .

U
g sebagai kekuatan medan graviti. g sebagai pecutan yang disebabkan graviti N
I
g as gravitational field strength. g as acceleration due to gravity T

1 Sebuah objek yang jatuh bebas berdekatan dengan permukaan Bumi akan memecut pada 10 m s-2.
2
An object falling freely near the Earth's surface will accelerate at 10 m s-2.

2 Setiap kilogram jisim yang berdekatan dengan permukaan Bumi mempunyai daya graviti 10 N yang bertindak
ke atasnya.
Each kilogram of mass near the Earth’s surface has a gravitational force of 10 N acting on it.
Kekuatan medan graviti, g = 10 N kg-1 / Gravitational field strength, g = 10 N kg-1
Pecutan disebabkan graviti / Acceleration due to gravity, g = 10 m s-2
Nilai g boleh ditulis sebagai 10 m s-2 atau 10 N kg-1.
The approximate value of g can be written either as 10 m s-2 or as 10 N kg-1.

Aktiviti 5: Pecutan yang disebabkan graviti


Activity 5: Acceleration due to gravity

Tujuan / Aim

Menentukan pecutan disebabkan graviti / To determine the acceleration due to gravity.

Radas / Apparatus
Jangka masa detik, bekalan kuasa 12 V, bangku, pengapit-G, pemberat, pita detik
Ticker timer, 12 V ac power supply, stool, G-clamp, slotted weight, ticker tape.

Prosedur / Procedure
1 Potong sekeping pita detik lebih kurang 2.5 m panjang dan lalukan melalui jangka masa detik yang diapit
kepada kerusi oleh pengapit-G.
Cut a piece of ticker tape about 2.5 m long and pass through the ticker timer which is clamped to a stool using G-clamp.

73 © Nilam Publication Sdn. Bhd.


MODUL • Fizik TINGKATAN 4

2 Sambungkan satu hujung pita pada pemberat 100 g. Pita detik


Ticker tape
Attach one end of the tape to the 100 g slotted weight.
Bekalan Pengapit-G
3 Hidupkan jangka masa detik dan pemberat dilepaskan supaya ia kuasa, 12 V G-clamp
A.C. Power
jatuh bebas. supply, 12 V Jangka masa
Switch on the ticker timer and release the slotted weight so that it falls detik
Ticker timer
freely.
4 Kaji pita itu untuk menentukan nilai bagi pecutan disebabkan oleh Bangku
Stool
graviti, g.
Analyse the tape to determine the value of the acceleration due to gravity, g.
Pemberat
Weight
Perbincangan / Discussion Kepingan polistirena
Polystyrene sheet
1 Mengapakah sukar untuk menentukan pergerakan objek yang jatuh
dengan hanya memerhatikannya jatuh?
Why is it difficult to describe the motion of a falling object by just observing it fall?
Objek bergerak sangat laju.
The object moves very fast.

U 2 Apakah jenis pergerakan objek jika ia jatuh di bawah tarikan graviti?


N What is the type of motion of an object falling under the pull of gravity?
I
T Pecutan seragam.
2 Constant acceleration.

3 Mengapakah pergerakan pemberat boleh diandaikan sebagai jatuh bebas?


Why is it that the motion of the slotted weight can be assumed to be a free fall?
Rintangan udara yang kecil boleh diabaikan.
The small air resistance is negligible.

4 Apakah langkah yang akan anda ambil untuk mengurangkan geseran antara pita dan jangka masa detik?
What steps did you take to minimise the friction between the ticker tape and the ticker timer?
Pegang pita detik dalam keadaan menegak dan lepaskannya. Pastikan ia jatuh melalui jangka masa
detik dengan lancar.
Hold the ticker tape vertically when releasing it. Make sure it slips through the ticker timer smoothly.

5 Terangkan mengapa perlu menjatuhkan pemberat daripada kedudukan yang tinggi.


Explain the need for the slotted weight to be dropped from a high position.
Pengiraan akan menjadi lebih tepat kerana ralat eksperimen dikurangkan.
The calculation will be more accurate because experimental errors are reduced.

6 Tunjukkan bagaimana anda mengira g daripada pita.


Show how you would calculate g from the tape.
s1 s
u= v= 2
t1 t2
v–u
a=
t

© Nilam Publication Sdn. Bhd. 74


MODUL • Fizik TINGKATAN 4

7 Bandingkan nilai pecutan disebabkan oleh graviti daripada aktiviti ini dengan nilai yang sebenar. Berikan
alasan yang munasabah bagi perbezaan di antara dua nilai tersebut.
Compare the value of the acceleration due to gravity from this activity with the actual value. Give possible reasons
for any difference in these two values.
Nilai daripada eksperimen adalah lebih rendah berbanding dengan nilai sebenar. Alasannya ialah
rintangan yang disebabkan oleh jangka masa detik.
The value from the experiment is lower than the actual value. The reason is the resistance caused by the ticker
timer.

8 Bandingkan nilai bagi g daripada eksperimen jika anda mengulangi eksperimen dengan menggunakan
pemberat 200 g dan 300 g.
Compare the values of g from the experiment if you repeat the experiment using 200 g and 300 g weights.
Keputusan sepatutnya sama.
The result should be the same.

9 Apakah yang boleh anda simpulkan tentang hubungan antara g dan jisim bagi objek yang jatuh?
U
What can you conclude about the relationship between g and the mass of the falling object? N
Jisim tidak mempengaruhi pecutan graviti, g. I
T
Mass does not affect the gravitational acceleration, g. 2

Definisi berat
Definition of weight

Berat, W, bagi sesuatu objek ialah daya graviti yang dikenakan ke atasnya.
The weight, W, of an object is the gravitational force acting on it.

Berat ialah daya dan diukur dalam unit Newton, N. Berat ialah kuantiti vektor.
Weight is a force and is measured in Newton, N. Weight is a vector quantity.

Sebiji batu yang berjisim m, dilepaskan jatuh bebas pada pecutan graviti, g.
A stone of mass, m, is released and free falls with a gravitational acceleration of g.

Daya yang bertindak ke atas batu hanyalah berat, W, di mana ianya menuju ke arah bawah.
The only force acting on the stone is its weight, W, which is downward.

Hukum gerakan Newton kedua:


Newton’s second law of motion: Pecutan = g
Acceleration = g
F = ma di mana / where F = W , a = g
Jisim / Mass = m kg

Oleh itu / Therefore:


W = mg
W = berat / weight
m = jisim / mass Berat / Weight = m × 10 N
g = pecutan graviti / acceleration due to gravity
Bumi / Earth
unit bagi g ialah = m s-2
the unit of g is = m s–2

75 © Nilam Publication Sdn. Bhd.


MODUL • Fizik TINGKATAN 4

Berat berubah, jisim tetap


Weight changes but mass is fixed
Di Bulan, berat kita lebih ringan daripada di Bumi, 1
ini kerana medan graviti di Bulan adalah lebih kecil. Nilai bagi pecutan graviti, gB, di bulan ialah
6
On the Moon, our weight is less than that on Earth because the Moon’s daripada nilai gE di bumi.
gravitational field is smaller than that of the Earth. The value of the gravitational acceleration, gB, on
1
Malahan di Bumi, berat kita berbeza sedikit dari suatu tempat ke the Moon is the value of gE on the Earth.
6
tempat yang lain, kerana kekuatan medan graviti yang berbeza.
Semakin jauh dari bumi, berat kita semakin berkurang.
Even on Earth, our weight can vary slightly from place to place,
because the Earth’s gravitational field strength varies.
Moving away from the Earth, our weight decreases.
Jika kita boleh pergi lebih jauh ke dalam ruang angkasa lepas
dan bebas daripada sebarang tarikan graviti, berat kita akan
menjadi sifar. Di Di
Di angkasa
If we could go deep into space, and be free from any gravitational
lepas permukaan permukaan
pull, our weight would be zero.
Deep in Bulan Bumi
Sama ada di atas Bumi, Bulan atau di angkasa, jisim kita tetap space On Moon’s On Earth’s
U tidak akan berubah. surface surface
N Whether on the Earth, Moon or deep in space, our mass does not
I Jisim
T change. 100 kg 100 kg 100 kg
Mass
2 Video
Berat 1
0N (1 000) N 1 000 N
Weight 6

Daya Graviti
Gravitational Force
https://goo.gl/rrdbZw
Perbezaan antara berat dan jisim
Difference between weight and mass

Berat Jisim
Weight Mass

Definisi Daya graviti yang bertindak ke atas objek. Jumlah jirim di dalam objek.
Definition The force of gravity acting on the object. The amount of matter in the object.

Berat sesuatu objek berubah dengan Jisim sesuatu objek tidak berubah
Perubahan / Tiada
perubahan kekuatan medan graviti pada sesuatu tempat. walau di mana-mana.
Change / Unchanged The weight of an object changes with the The mass of an object is unchanged
gravitational field strength at the location. anywhere.
Kuantiti asas atau
kuantiti terbitan Kuantiti terbitan Kuantiti asas
Base quantity or derived base
A quantity A quantity
derived quantity

Kuantiti vektor atau vektor skalar


Kuantiti Kuantiti
kuantiti skalar
Vector or scalar quantity A vector quantity A scalar quantity

Newton, N Kilogram, kg
Unit S.I / S.I unit
Newton, N Kilogram, kg

© Nilam Publication Sdn. Bhd. 76


MODUL • Fizik TINGKATAN 4

Untuk objek yang jatuh dengan pecutan, g, berikut adalah persamaan-persamaan yang berkaitan:
For an object falling with acceleration g, the following equations apply:
1 v = u + gt di mana / where s = sesaran / displacement
titik tertinggi, v = 0
2 s = ut + ½ gt2 u = halaju awal / initial velocity highest point, v = 0
3 s = ½ (u + v)t v = halaju akhir / final velocity
4 v = u + 2gs t = masa / time
2 2

a = g, pecutan graviti / acceleration due to gravity,


Nota / Notes:
1 Apabila suatu objek jatuh bebas: a = g = 10 m s-2 (pecutan)
When an object fall freely: a = g = 10 m s-2 (acceleration)
2 Apabila suatu objek dilambung ke atas: a = –g = –10 m s-2. (nyahpecutan)
When an object is thrown upwards: a = –g = –10 m s-2. (deceleration)
3 Pada kedudukan yang tertinggi, v = 0.
At the highest point, v = 0. Objek dilambung
4 Jatuh ke bawah, v adalah positif. / Downward direction, v is positive. ke atas
Object thrown
5 Arah ke atas, v adalah negatif. / Upward direction, v is negative. upwards

U
N
I
T
Contoh / Examples
2
Andaikan g = 10 m s-2 dan tiada rintangan udara. / Assume g = 10 m s-2 and there is no air resistance.
1 Sebiji batu jatuh daripada ketinggian 45 m. 2 Sebiji bola dilambung ke atas daripada tanah dengan
A rock falls from a height of 45 m. halaju 30 m s–1. Selepas beberapa lamakah bola itu
(a) Berapa lamakah masa yang diambil oleh batu itu akan menyentuh tanah semula?
untuk mencecah ke tanah? A ball is thrown upwards from the ground with a velocity
How long does it of 30 m s–1. After how many seconds will it strike the
take to reach the ground again?
ground? v=0

(b) Berapakah
halaju batu Pecutan g = –10 m s–2
itu semasa ia Acceleration
g
menghentam Tinggi / Height Masa
lantai? h Time
t u = 30 m s–1
What is its
velocity as it hits
the ground?
Penyelesaian / Solution
(a) u = 0, s = 45 m, Penyelesaian / Solution
g = 10 m s–2, t = ? Untuk gerakan ke atas,/For the upward motion,
s = ut + ½ gt2 u = 30 m s–1, v = 0, g = –10 m s–2,
45 m = 0 + ½ (10 m s–2)(t2) v = u + gt
t2 = 9 s2 ∴0 = 30 m s–1 + (–10 m s–2)(t)
t=3s 10t = 30 s
(b) v = u + gt t = 3 s (gerakan ke atas / upward motion)
= 0 + (10 m s–2)(3 s) Maka, masa untuk gerakan ke bawah juga
= 30 m s–1 mengambil 3 s. Oleh itu, ia mengambil masa 6 saat.
The time taken for the downward motion is also 3 s.
So it takes a total of 6 seconds.

77 © Nilam Publication Sdn. Bhd.


MODUL • Fizik TINGKATAN 4

Latihan / Exercises
Andaikan nilai g = 10 m s–2. / Assume the value of g = 10 m s–2.
1 Amir menjatuhkan batu ke dalam perigi. Jika jarak antara bahagian atas perigi dan permukaan air ialah 20 m,
Amir releases a stone into a well. If the distance between the top of the well and the water surface is 20 m,
(a) berapakah masa yang diambil oleh batu itu untuk sampai ke permukaan air?
what is the time required for the stone to reach the surface of the water?
(b) berapakah halaju batu itu apabila ia terkena permukaan air?
what is the velocity of the stone when it strikes the surface of the water?
Penyelesaian / Solution
(a) u = 0, s = 20 m , g = 10 m s–2 , t = ? Perhatian / Note:
Untuk (b), mengapa v2 = u2 + 2gs digunakan
s = ut + ½ gt2 dan bukan v = u + gt?
20 m = 0 + ½ (10 m s–2)(t2) For (b), why v2 = u2 + 2gs is used and not
v = u + gt?
t2 = 4 s2
Jawapan / Answer:
t =2s Semua nilai yang diperlukan diberi dalam
(b) v = u2 + 2gs
2 soalan.
All the values required are given in the
v2 = 0 + 2(10 m s–2)(20 m) question.
∴v = 20 m s–1
U
N
I
T
2 Suatu objek yang berjisim 5 kg dilepaskan dari sebuah bangunan setinggi 500 m. Berapakah
2 An object of mass 5 kg is released from a tall building of height 500 m. What is the
(a) berat objek itu? / weight of the object?
(b) kekuatan medan graviti? / gravitational field strength?
(c) masa yang diambil untuk sampai ke tanah? / time taken to reach the ground?
Penyelesaian / Solution
(a) W = 5 kg × 10 m s–2 (c) u = 0, s = 500 m, g = 10 m s–2
= 50 N s = ut + ½ gt2
(b) g = 10 N kg–1 atau / or 10 m s–2 500 m = 0 + ½(10 m s–2)(t2)
t2 = 100 s2
t = 10 s

MENGANALISIS KESEIMBANGAN DAYA


2.9 ANALYSING FORCES IN EQUILIBRIUM

Menerangkan situasi di mana daya berada dalam keseimbangan


Describe situations where forces are in equilibrium

Keseimbangan daya / Forces in equilibrium

1 Apabila suatu daya dikenakan terhadap objek dan ia mengekalkan keadaan pegun atau bergerak dengan
halaju seragam, maka objek itu dikatakan berada di dalam keadaan keseimbangan.
When forces act upon an object and it remains stationary or moves at a constant velocity, the object is said to be
in a state of equilibrium.

2 Apabila keadaan keseimbangan berlaku, daya paduan yang bertindak ke atas objek itu adalah sifar iaitu
tiada daya bersih bertindak ke atasnya. / When equilibrium is reached, the resultant force acting on the object is
zero, i.e. there is no net force acting upon it.

© Nilam Publication Sdn. Bhd. 78


MODUL • Fizik TINGKATAN 4

Daya paduan: daya tunggal yang menunjukkan kesan daripada gabungan dua
Definisi daya paduan atau lebih daya dalam magnitud dan arah
Definition of resultant force Resultant force: a single force that represents the combined effect of two or more forces
in magnitude and direction

Daya paduan ialah hasil tambah vektor bagi dua atau lebih daya yang bertindak ke atas objek. Dalam kes-kes
berikut, jika F ialah daya paduan, maka,
The resultant force is the vector sum of two or more forces which act on the object. In the cases below, if F is the resultant
force, hence,
F1
F2 F1
F2

Daya paduan/Resultant force = F = F1 + F2 Daya paduan/Resultant force = F = F1 – F2

Latihan / Exercises U
N
I
T
1 Hitungkan daya paduan. Ke arah manakah objek 2 Hitungkan daya paduan. Ke arah manakah objek
itu bergerak? itu bergerak? 2
Calculate the resultant force. Which direction does the Calculate the resultant force. Which direction does the
object move? object move?

5N 5N
12 N 12 N

Daya paduan, F / Resultant force, F = Daya paduan, F / Resultant force, F =


5 N + 12 N = 17 N Arah ke kanan / To the right. 12 N – 5 N = 7 N Arah ke kanan / To the right.

3 Seekor kuda menarik kereta dengan daya 500 N.


Seorang petani membantu kuda itu menolak kereta itu 200 N
500 N
dengan daya 200 N. Berapakah daya paduan?
A horse pulls a cart with a force of 500 N. A farmer helps the
horse by pushing the cart with a force of
200 N. What is the resultant force?
Daya paduan, F / Resultant force, F
= 500 N + 200 N = 700 N ke kanan / to the right

4 Seekor kuda menarik kereta dengan daya 500 N.


Seorang petani menarik kereta itu pada arah bertentangan
dengan daya 200 N. Berapakah daya paduan? 200 N 500 N
A horse pulls a cart with 500 N force. A farmer pulls the
cart with a force of 200 N but in opposite direction. What is
the resultant force?
Daya paduan, F / Resultant force, F
= 500 N – 200 N
= 300 N ke kanan / to the right

79 © Nilam Publication Sdn. Bhd.


MODUL • Fizik TINGKATAN 4

5 Rajah (a) menunjukkan sebiji ladung berjisim 0.3 kg digantung dari siling.
Diagram (a) shows a pendulum bob of mass 0.3 kg hung from the ceiling. Siling
Ceiling
Benang itu ditarik secara mengufuk dengan daya, F, supaya sudut antara benang Benang
Thread
dengan garis mencancang adalah 40° seperti ditunjukkan dalam Rajah (b).
The thread is then pulled horizontally by a force, F, so that the thread makes an angle
Ladung
of 40° with the vertical line as shown in Diagram (b). Pendulum Bob

Rajah (a) / Diagram (a)

(a) Dalam ruang di bawah, lukis sebuah segi tiga keseimbangan


daya bagi T, F dan berat ladung itu.
40° In the space below, draw a triangle of forces in equilibrium for
T T, F and the weight of the bob.

40°
F
Berat Perhatian / Note:
T Weight Arah bagi tiga daya itu
3.0 N
m = 0.3 kg
adalah berkitar.
U The directions of the three
N
I forces are cyclic.
T Berat
Weight F

2 Rajah (b) / Diagram (b)

(b) Hitung daya, F


Calculate the force, F
Dari segi tiga di atas, / From the triangle above,
F = tan 40°
3.0 N
F = 3.0 N tan 40°
= 2.52 N

(c) Hitung tegangan, T, dalam benang itu


Calculate the tension, T, in the string.
Dari segi tiga di atas, / From the triangle above,
3.0 N = kos 40°
T Trigonometri / Trigonometry
T = 3.0 N sin q =
Bertentangan / Opposite
kos 40° Hipotenus / Hypotenuse
= 3.92 N Sebelahan / Adjacent
kos q =
Hipotenus / Hypotenuse
Bertentangan / Opposite
tan q =
Sebelahan / Adjacent

Perhatian / Note: Soalan 5 di atas boleh dijawab dengan kaedah lukisan berskala.
Question 5 above can be answered by scale-drawing.

© Nilam Publication Sdn. Bhd. 80


MODUL • Fizik TINGKATAN 4

Dua daya yang betindak pada sudut tertentu antara satu sama lain
Two forces acting at an angle to each other

Daya paduan bagi dua daya yang bertindak ke atas satu objek pada dua arah berbeza boleh ditentukan dengan
menggunakan kaedah segi tiga daya atau kaedah segi empat selari daya.
The resultant force of two forces, which act on an object in two different directions, can be determined by a triangle of
forces or a parallelogram of forces.

Dua daya yang bertindak pada satu titik melalui satu sudut
[kaedah segi empat selari] F1
Two forces acting at a point at an angle [Parallelogram method]
60 °C

Skala / Scale: 1 cm = N F2

LANGKAH 1: LANGKAH 2: LANGKAH 3: U


N
STEP 1: STEP 2: STEP 3: I
Dengan menggunakan pembaris dan Lengkapkan rajah segi empat selari. Lukis pepenjuru (dari X) bagi segi T
protraktor, lukis dua daya F1 dan F2
bermula dari satu titik, X.
Complete the parallelogram. empat selari bagi menunjukkan
magnitud dan arah bagi daya
2
Using ruler and protractor, draw the paduan, F.
two forces F1 and F2 from a point, X. F1 Draw the diagonal (from X) of the
parallelogram. The diagonal represents
X the resultant force, F in magnitude and
F1
direction.
F2
X 60 °C
F1
F2 Daya paduan
X
Resultant force
F2

Contoh / Examples
Maklumat tambahan:
Rajah di bawah menunjukkan dua daya yang bertindak ke atas objek P. Additional information:
The diagram below shows two forces acting on object P.

5N

F1

60°
P 12 N
F2

Tentukan daya paduan yang terhasil.


Determine the resultant force produced.

81 © Nilam Publication Sdn. Bhd.


MODUL • Fizik TINGKATAN 4

Jawapan / Answer:
Kaedah I
Method I
Kaedah segi empat selari daya
Parrallelogram of forces method
1 Tentukan skala. Dengan menggunakan pembaris dan protraktor, lukis dua daya F1 dan F2 bermula dari titik
O.
Set a scale. Using a ruler and protractor, draw the two forces, F1 and F2 from a point O.
2 Lengkapkan rajah segi empat selari. / Complete the parallelogram.
3 Lukis pepenjuru bagi segi empat selari. Pepenjuru menunjukkan magnitud dan arah daya paduan, F.
Draw the diagonal of the parallelogram. The diagonal represents the resultant force, F in magnitude and direction.

5 cm = 5 N

U
N
I
T
60°
2 O
12 cm = 12 N

Kaedah II
Method II
Kaedah segi tiga daya
Triangle of forces method
1 Tentukan skala. Dengan menggunakan pembaris dan protraktor, lukis daya yang pertama, F1 dari titik O.
Set a scale. Using a ruler and protractor, draw the first force, F1, from a point O.
2 Lukis daya yang kedua F2 dari hujung atas F1.
Draw the second force, F2 from the head of F1.
3 Lengkapkan segi tiga daya dengan melukis garisan dari pangkal F1 ke hujung F2.
Complete a triangle of forces by drawing a line from the tail of F1 to the head of F2.
4 Pepenjuru menunjukkan magnitud dan arah daya paduan, F.
The diagonal represents the resultant force, F, in magnitude and direction.
F2 = 12 N

F1 = 5 N

F = Daya paduan
Resultant force

60°
O

© Nilam Publication Sdn. Bhd. 82


MODUL • Fizik TINGKATAN 4

Latihan / Exercises
1 Dengan menggunakan skala dan kaedah yang sesuai, tentukan daya paduan.
(Perhatian: Magnitud dan arah diperlukan).
By using suitable scale and method, determine the resultant force
(Note: Magnitude and direction are required).
(a) (b) (c)

8N 5N
6N
600 1200

8N 10 N 5N

2 Lengkapkan rajah untuk menunjukkan daya paduan.


Complete the diagram to show the resultant force.

U
N
2N I
120° T

5N 2

Jawapan / Answers:
1 Skala / Scale: 1 cm : 1 N

(a)

N
10.0
n=
d ua rce
pa fo
6N ya ant
Da sult
Re

37°
8N
10 N pada sudut 37° dengan daya 8 N
10 N at angle of 37° with the 8 N force

83 © Nilam Publication Sdn. Bhd.


MODUL • Fizik TINGKATAN 4

(b)

N
5.6
=1
an
p aduforce
a t
8N Day ultan
Res

600
260

10 N
15.6 N pada sudut 26° dengan daya 10 N
U 15.6 N at an angle of 26° with the 10 N force
N
I
T

2 (c)
N
rce 5.0
t fo =
an an
ult adu

5N
Res ya p
Da

600
600
5N

5 N pada sudut 60° dengan daya 5 N


5 N at an angle of 60° with 5 N force

2 Skala / Scale: 2 cm : 1 N
(a)

Daya
padu
an / R
esulta
nt for
ce = 6
.2 N
2N

120°
16°

5N

6.2 N pada sudut 16° dengan daya 5 N


6.2 N at an angle of 16° with 5 N force

© Nilam Publication Sdn. Bhd. 84


MODUL • Fizik TINGKATAN 4

Daya dileraikan kepada komponen berkesan


Resolve a force into the effective components

Leraian Daya / Resolution Of Forces:


Daya F boleh dileraikan kepada dua komponen yang berserenjang / bersudut tegak antara satu sama lain:
A force F can be resolved into two components which are perpendicular / at 90° to each other:
(a) Fx: komponen mengufuk / horizontal component,
θ
(b) Fy: komponen menegak / vertical component,
Fy Fx Fy F
sin θ = kos θ =
F F
θ
Fy = F sin θ Fx = F kos θ
Fx

Latihan / Exercises
1 Dapatkan komponen mengufuk dan komponen menegak daya tersebut. U
(Perhatian: Pertimbangkan magnitud daya yang positif sahaja.) N
I
Find the horizontal component and the vertical component of the force. T
(Note: Consider only the positive magnitudes of the forces.)
(a) Fy (b) Fx 2
200 N 700
700
500

75 N
500
Fx Fy

Fx = kos 50° Fy = sin 50° Fx = sin 70° Fy = kos 70°


200 N 200 N 75 N 75 N
∴ Fx = 200 N kos 50° ∴ FY = 200 N sin 50° ∴ Fx = 75 N sin 70° ∴ Fy = 75 N kos 70°
= 128.6 N = 153.2 N = 70.48 N = 25.65 N

(c) (d) 0 Fx
Fy
600
F=6N

400 F=5N
600
400

Fx
0
Fy

Fx = sin 40° Fy = kos 40° Fx = kos 60° Fy = sin 60°


5N 5N 6N 6N
∴ Fx = 5 N sin 40° ∴ Fy = 5 N kos 40° ∴ Fx = 6 N kos 60° ∴ Fy = 6 N sin 60°
= 3.21 N = 3.83 N = 3.0 N = 5.20 N

85 © Nilam Publication Sdn. Bhd.


MODUL • Fizik TINGKATAN 4

2 Rajah menunjukkan troli yang berjisim 2 kg di atas permukaan licin, 6N


ditarik oleh daya 6 N pada sudut 60° dengan garis mengufuk.
The diagram shows a trolley of mass 2 kg on a smooth surface being pulled by 2 kg 600
a force of 6 N at an angle of 60° with the horizontal. Fx
(a) Berapakah komponen mengufuk daya itu?
Permukaan licin / Smooth surface
What is the horizontal component of the force?
(b) Berapakah pecutan troli itu? / What is the acceleration of the trolley?
Penyelesaian / Solution
Fx 3N
(a) = kos 60° (b) F = ma, a = = 1.5 m s-2
6N 2 kg
∴ Fx = 6 N kos 60°
= 3.0 N

3 Rajah menunjukkan sebuah kereta sedang ditunda. Kabel itu mempunyai


daya, 5 000 N. 600
The diagram shows a car being towed. The cable has a force of 5 000 N.
(a) Tunjukkan dan labelkan / Indicate and label:
U
• daya F / the force F
N • daya komponen mengufuk, Fx / the horizontal component force Fx
I
T • daya komponen menegak, Fy / the vertical component force Fy

2
(b) Cari / Find
• daya mengufuk kabel yang menggerakkan kereta ke hadapan.
the horizontal force of the cable which moves the car forward.
• daya menegak kabel. / the vertical force of the cable.
Penyelesaian / Solution Fx
(a) (b) = kos 60°
5 000 N
F = 5 000 N ∴ Fx = 5 000 N kos 60°
Fy
60° = 2 500 N
Fy
Fx = sin 60°
5 000 N
∴ Fy = 5 000 N sin 60°
= 4 330 N

4 Seorang pelancong menarik begnya dengan daya 100 N pada sudut 55° dari garis mengufuk.
A tourist pulls his bag with a force of 100 N at an angle of 55° with the horizontal.
(a) Tunjukkan dan labelkan / Indicate and label:
• daya F / the force F
• daya komponen mengufuk, Fx / the horizontal component force Fx
• daya komponen menegak, Fy / the vertical component force Fy
(b) Cari / Find
• daya mengufuk beg yang menggerakkannya ke hadapan.
the horizontal force of the cable which moves it forward.
• daya menegak beg. / the vertical force of the bag.
Penyelesaian / Solution Fx
(a) (b) = kos 55°
100 N
F = 100 N ∴ Fx = 100 N kos 55°
Fy
55° = 57.36 N
Fy
Fx = sin 55°
100 N
∴ Fy = 100 N sin 55° = 81.92 N

© Nilam Publication Sdn. Bhd. 86


MODUL • Fizik TINGKATAN 4

Masalah yang melibatkan daya paduan dan prinsip leraian daya


Problems involving resultant force and the principle of resolution of forces

A Lif / Lift

Seorang budak lelaki berada di dalam sebuah lif. Dia berdiri di atas mesin penimbang. Berat budak lelaki, W,
bertindak ke arah bawah budak itu dan tindak balas normal, R, bertindak ke arah atas.
A boy is inside a lift. He is standing on a weighing machine. The weight of the boy, W acts downward on the boy and a
normal reaction, R, acts upwards.

Lif yang pegun (atau bergerak ke


Lif bergerak ke atas dengan Lif bergerak ke bawah dengan
bawah atau ke atas dengan halaju
pecutan a (m s-2) pecutan a (m s-2)
seragam)
The lift moves upwards with an The lift moves downwards with an
Stationary lift (or moves upwards or
acceleration of a (m s-2) acceleration of a (m s-2)
downwards with uniform velocity)

R R R

U
N
Mesin penimbang Mesin penimbang Mesin penimbang I
Weighing machine Weighing machine Weighing machine T

2
a a

W = mg W = mg W = mg

Daya paduan, F = 0 Daya paduan, F arah ke atas Daya paduan, F arah ke bawah
Resultant force, F = 0 Resultant force, F is upwards Resultant force, F is downward

F = R – mg = 0 R > mg mg > R
R = mg F = ma F = ma
F = R – mg = ma F = mg – R = ma
R = mg + ma R = mg – ma

Bacaan pada mesin penimbang = Bacaan pada mesin penimbang Bacaan mesin penimbang lebih
berat budak lelaki tersebut lebih besar daripada berat budak itu kecil daripada berat budak itu
The reading on the weighing scale = The reading on the weighing scale The reading on the weighing scale
the weight of the boy machine is larger than the weight of machine is smaller than the weight of
the boy the boy

Perhatian : Dalam setiap kes di atas, R memberi bacaan pada mesin penimbang.
Note : In each of the cases above, R gives the reading on the weighing scale.

87 © Nilam Publication Sdn. Bhd.


MODUL • Fizik TINGKATAN 4

Peta Pemikiran / Thinking Maps


Daya paduan (Banding dan Beza)
Resultant force (Comparing and Contrasting)
Kes 1 / Case 1:
Lif bergerak F1
ke atas dengan pecutan, a F2
Lift moving upward with
acceleration, a Unit: Daya paduan
R > mg Unit: Resultant force
F = R – mg N = F1 + F2
ma = R – mg
R = mg + ma
Kes 2 / Case 2:
F2 F1
Lif yang bergerak
ke atas pada halaju malar Daya paduan Daya paduan
Lift moving upward at dalam lif yang untuk gerakan Daya paduan
constant velocity bergerak mendatar Resultant force
F = R – mg Resultant force in Resultant force = F1 – F2
0 = R – mg a lift in motion for horizontal
∴ R = mg (jika / if F1 > F2)
U motion
N
I
T

2
Lif bergerak Kes 3 / Case 3:
ke bawah dengan Daya tunggal yang
pecutan, a F
menunjukkan kesan daripada θ
Lift moving downward gabungan dua atau lebih daya R
with acceleration, a dalam magnitud dan arah
R < mg Single force that shows the effects of Daya paduan
F = mg – R a combination of two or more forces
ma = mg – R Resultant force
in magnitude and directions
R = mg – ma = F cos θ – R

Latihan / Exercises
1 Seorang budak lelaki berjisim 50 kg berada di dalam sebuah lif.
A boy of mass 50 kg is inside a lift.
(a) Kirakan berat budak lelaki itu. / Calculate the weight of the boy.
(b) Kirakan bacaan yang ditunjukkan oleh mesin penimbang jika lif itu: Lif m = 50 kg
Lift
Calculate the reading on the weighing scale if the lift:
(i) pegun / is stationary
(ii) memecut ke atas dengan pecutan 2 m s-2
accelerates upwards with an acceleration of 2 m s-2
(iii) memecut ke bawah dengan pecutan 2 m s-2
accelerates downwards with an acceleration of 2 m s-2
(iv) bergerak ke atas dengan halaju seragam 1.5 m s-1
moves upwards with constant velocity of 1.5 m s-1 Mesin penimbang
Weighing scale
Penyelesaian / Solution:
(a) Berat budak = W (iii) mg – R = ma
Mass of the boy = mg R = 500 N – (50 kg)(2 m s–2)
= 50 kg × 10 m s–2 = 400 N
= 500 N (iv) R – mg = ma
(b) (i) R = W = 500 N Tetapi a = 0 (kerana halaju malar)
(ii) R – mg = ma but a = 0 (because constant velocity)
R = 500 N + (50 kg)(2 m s–2) ∴ R = mg
= 600 N = 500 N

© Nilam Publication Sdn. Bhd. 88


MODUL • Fizik TINGKATAN 4

B Takal / Pulley

1
Daya
Takal licin geseran, 2 N
Smooth Frictional T
pulley force, 2 N Takal licin
Smooth
T 4 kg pulley
T T

4 kg
3 kg
3 kg

Berat / Weight, Berat / Weight,


40 N 30 N
Berat / Weight,
30 N

Cari daya paduan, F F = 40 N – 30 N = 10 N F = 30 N – 2 N = 28 N


Find the resultant
force, F
U
N
I
Cari jisim yang m = 4 kg + 3 kg = 7 kg m = 4 kg + 3 kg = 7 kg T
bergerak, m
Find the moving
2
mass, m

Cari pecutan, a F = ma F = ma, ∴28 N = (7 kg)(a)


Find the acceleration, a,
a = 10 N = 1.43 m s-2 a = 28 N = 4 m s-2
7 kg 7 kg

Cari tegangan Kaedah I / Method I Kaedah I / Method I


benang, T Pertimbangkan jisim 4 kg sahaja (gerak Pertimbangkan jisim 3 kg sahaja (yang
Find the string ke bawah) bergerak ke bawah)
tension, T Consider only the 4 kg-mass (moving Consider only the 3 kg mass (moving
downwards) downwards)
40 N – T = m1 a 30 N – T = ma
40 N – T = (4 kg)(1.43 m s–2) 30 N – T = (3 kg)(4 m s–2)
∴ T = 34.28 N ∴ T = 30 N – 12 N
Kaedah II / Method II = 18 N
Pertimbangkan jisim 3 kg sahaja (gerak Kaedah II / Method II
ke atas) Pertimbangkan jisim 4 kg sahaja (yang
Consider only the 3 kg mass (moving bergerak ke kanan)
upwards) Consider only the 4 kg mass (moving to the
T – 30 N = m2 a right)
T = 30 N + (3 kg)(1.43 m s–2) T–2N = ma
T = 34.29 N ∴T = 2 N + (4 kg)(4 m s–2)
= 2 N + 16 N
= 18 N

89 © Nilam Publication Sdn. Bhd.


MODUL • Fizik TINGKATAN 4

2 Troli yang berjisim 2 kg disambungkan dengan tali kepada 2 kg


pemberat berjisim 3 kg. Tali diletakkan di atas takal licin. Pemberat T Takal licin
kemudiannya dilepaskan. Smooth pulley
A 2 kg-pulley is connected by a rope to a 3 kg-load. The rope passes over
a smooth pulley. The load is then released. T
Meja
(a) (i) Jika permukaan meja adalah licin, berapakah pecutan troli? Table
If the surface of the table is smooth, what is the acceleration of 3 kg
the trolley? 30 N
(ii) Berapakah tegangan dalam tali?
What is the tension in the rope?
(b) (i) Jika daya geseran antara troli dan permukaan meja ialah 10 N, berapakah pecutan troli?
If the frictional force between the trolley and the surface of the table is 10 N, what is the acceleration of the trolley?
(ii) Berapakah tegangan tali sekarang?
What is the tension in the rope now?
Penyelesaian / Solution
(a) (i) F = ma (b) (i) 30 N – 10 N = (3 + 2) kg × a
30 N = [(2 + 3) kg] [a] 20 N = 5 kg × a
a = 6 m s–2 a = 4 m s-2
(ii) dari gerakan troli (ii) dari gerakan troli
U from the motion of the trolley from the motion of the trolley
N
I T = ma T – 10 N = ma
T = (2 kg)(6 m s–2) T – 10 N = (2 kg)(4 m s-2)
2 = 12 N
= 18 N
T = 8 N + 10 N
atau dari gerakan jisim 3 kg
or from the motion of the 3 kg-mass atau dari gerakan jisim 3 kg
30 N – T = ma or from the motion of the 3 kg-mass
30 N – T = (3 kg)(6 m s–2) 30 N – T = m1 a
T = 30 N – 18 N 30 N – T = (3 kg)(4 m s-2)
= 12 N T = 30 N – 12 N
= 18 N

C Satah condong / Inclined plane

Rajah di bawah menunjukkan seorang budak lelaki yang berjisim 40 kg sedang menggelongsor turun dari sebuah
terowong luncur. Permukaan terowong luncur itu adalah kasar dan daya geseran yang bertindak ialah 120 N.
Diagram below shows a boy of mass 40 kg sliding down from a slide tunnel. The surface of tunnel is rough and the frictional
force is 120 N.
R
Tentukan / Determine 120 N
(a) daya, Wc. / the force, Wc.
(b) daya paduan yang bertindak ke atas budak itu. Terowong luncur
the resultant force acting on the boy. Slide tunnel
(c) pecutan semasa budak itu menggelongsor menuruni terowong.
the acceleration of the boy during sliding down the slide tunnel. Wc
W
Penyelesaian / Solution 30.0º

(a) Berat budak, W = 400 N (b) Daya paduan (c) F = ma


Weight of boy, W Resultant force F = (40 kg)a
= 200 + (–120) (40 kg)a = 80 N
Wc = mg sin θ = 80 N a = 80 N
Daya,Wc / Force, Wc 40 kg
= 400 sin 30º = 2 m s–2
= 200 N

© Nilam Publication Sdn. Bhd. 90


MODUL • Fizik TINGKATAN 4

MEMAHAMI KERJA, TENAGA, KUASA DAN KECEKAPAN


2.10 UNDERSTANDING WORK, ENERGY, POWER AND EFFICIENCY

Kerja dilakukan apabila daya membuatkan suatu objek bergerak. Semakin besar daya dan jarak sesuatu objek itu
digerakkan, maka semakin besar kerja dilakukan. / Work is done whenever a force makes something move. The greater the
force and the greater the distance moved, the more work is done.

Definisi kerja / Definition of work

Kerja yang dilakukan ialah hasil darab daya yang dikenakan dengan sesaran objek pada arah daya dikenakan.
Work done is the product of an applied force and the displacement of the object in the direction of the applied force.

Kerja, W = Fs , di mana F = daya, s = sesaran


Work, W = Fs , where F = force, s = displacement
s
F
Unit S.I bagi kerja ialah joule, J / The S.I unit of work is the joule, J
Kerja 1 joule dilakukan apabila daya 1 N menggerakkan objek sejauh
1 m dalam arah daya dikenakan. / 1 joule of work is done when a force of
1 N moves an object 1 m in the direction of the force.
U
N
I
T

Pengiraan kerja / Calculation of work 2


Sesaran, s, bagi objek dalam arah sama dengan daya, F Sesaran, s, bagi objek pada arah yang tidak
The displacement, s, of the object is in the direction of the force, F sama dengan arah daya, F / The displacement, s,
of the object is not in the direction of the force, F

F
θ
F1
s
s
F F F1
= kos θ
s F
F1 = F kos θ
∴W = F1 × s
W = Fs W = Fs = (F kos θ) s

Contoh / Examples 1 Contoh / Examples 2


Seorang budak menolak basikalnya dengan daya Seorang budak perempuan mengangkat sebuah pasu
25 N melalui jarak 3 m. Kira kerja yang dilakukan berjisim 3 kg pada ketinggian 0.4 m. Berapakah kerja
oleh budak itu. yang dilakukan oleh budak perempuan itu?
A boy is pushing his bicycle A girl lifts up a 3 kg flower pot steadily to
with a force of 25 N through a 25 N a height of 0.4 m. What is the work done
distance of 3 m. Calculate the by the girl?
work done by the boy. Penyelesaian / Solution
Penyelesaian / Solution W = 3 kg × 10 m s–2 × 0.4 m
W = Fs = 12 J
= 25 N × 3 m Atau / Or
= 75 J W = 3 kg × 10 N kg–1 × 0.4 m
(Perhatian / Note: 1 N m = 1 J) = 12 J 30 N

91 © Nilam Publication Sdn. Bhd.


MODUL • Fizik TINGKATAN 4

Contoh / Examples 3
Seorang pekebun menolak mesin rumput dengan daya 50 N pada sudut 60º dengan ufuk. Berapakah kerja yang
dilakukan untuk menolak mesin rumput pada jarak 100 m?
A gardener pushes a lawn mower with a force of 50 N at an angle of 60º from
horizontal. What is the work done in pushing the lawn mower through a distance of
100 m?
50 N
Penyelesaian / Solution
F
F
= kos 60° 60° 600
50 N
50 N
F = 50 N kos 60°
∴ W = F × 100 m
= (50 N kos 60°) × (100 m) 60°
= 2 500 N

U
N
I Tiada kerja dilakukan apabila: / No work is done when:
T

2 Objek berada dalam keadaan Arah gerakan objek adalah Tanpa daya luar (F = 0), satelit
pegun. Sesaran sifar (s = 0). berserenjang dengan daya yang bergerak kerana inersia.
The object is stationary. Zero dikenakan (θ = 90º). Without external force (F = 0), satellite
displacement (s = 0). The direction of motion of the object moves due to inertia.
is perpendicular to that of the applied
force (θ = 90º). Bumi Satelit
Earth Satellite

W = Fs kos θ
W = Fs kos θ Seorang pelayan membawa dulang = (0)s kos θ
= F(0) kos θ makanan dan berjalan. / A waiter is = 0
= 0 carrying a tray of food and walking.
W = Fs kos θ
= F(s) kos 90º
= 0

Latihan / Exercises

1 Berapakah kerja yang dilakukan oleh daya 80 N itu?


How much work is done by the 80 N force? F = 80 N

Penyelesaian / Solution
W = 80 N × 0.5 m
= 40 J
Sesaran / Displacement = 50 cm

© Nilam Publication Sdn. Bhd. 92


MODUL • Fizik TINGKATAN 4

2 Ali menolak sebiji batu dengan mengenakan daya 200 N.


Berapakah kerja yang dilakukannya? Pegun / Stationary
Ali pushes a big rock by applying a force of 200 N. How much
work has he done?
F = 200 N
Penyelesaian / Solution
W = sifar (pegun) / zero (stationary)

3 Seorang lelaki menarik satu beban dengan menggunakan


satah condong. Tinggi landasan condong itu ialah 80 cm.
Berapakah kerja yang dilakukan oleh lelaki itu untuk
menarik beban itu? F = 320 N
A man pulls up a load using an inclined plane. The height of
the inclined plane is 80 cm. How much work is being done by
80 cm
the man to lift the load? 30°
Penyelesaian / Solution
s U
80 cm N
30° I
T
80 cm
s
= sin 30° W = Fs
0.8 m
2
80 cm = 320 N × ( )
∴s = sin 30°
sin 30°
0.8 m = 512.0 J
=
sin 30°

4 Berapakah kerja yang dilakukan oleh Raju untuk mengangkat beban


melalui jarak 4 m?
How much work is done by Raju to lift the load through the displacement F = 150 N
of 4 m? F

Penyelesaian / Solution
W = 150 N × 4 m Sesaran
= 600 J Displacement
=4m

5 Satu daya 25 N digunakan untuk mengangkat sebuah beg. Encik Rahim berjalan
sejauh 20 m dengan memegang beg itu. Berapakah kerja yang dilakukan oleh
Encik Rahim terhadap beg itu? Terangkan jawapan anda.
A force of 25 N is used to lift a bag. Encik Rahim walks a distance of F = 25 N
20 m holding the bag. How much work is being done by Encik Rahim with respect to
the bag? Explain your answer.
Penyelesaian / Solution
Kerja dilakukan adalah sifar kerana daya adalah berserenjang dengan sesaran.
Work done is zero because the force is perpendicular to the displacement.
Sesaran / Displacement = 20 m

93 © Nilam Publication Sdn. Bhd.


MODUL • Fizik TINGKATAN 4

Menyatakan bahawa apabila kerja dilakukan, tenaga dipindahkan dari satu objek ke satu objek yang lain
State that when work is done, energy is transferred from one object to another

• Tenaga boleh ditakrifkan sebagai kebolehan untuk melakukan kerja .


Energy can be defined as the ability to do work .

• Satu objek yang boleh melakukan kerja dikatakan mempunyai tenaga.


An object that can do work has energy.

• Tenaga boleh wujud dalam pelbagai bentuk, misalnya:


Energy exists in many forms, for example:
(a) Tenaga keupayaan graviti / Gravitational potential energy
(b) Tenaga kinetik / Kinetic energy
(c) Tenaga keupayaan kenyal / Elastic potential energy
(d) Tenaga keupayaan elektrik / Electric potential energy
(e) Tenaga bunyi / Sound energy
(f) Tenaga mekanik / Mechanical energy
(g) Tenaga nuklear / Nuclear energy
U
N
I • Kerja dilakukan apabila daya dikenakan ke atas objek dan objek itu bergerak. Ini diikuti dengan pemindahan
T
tenaga dari satu objek ke objek lain. / Work is done when a force is applied on an object and the object moves.
2 This is followed by the transference of energy from one object to another.

Definisi tenaga keupayaan graviti / Definition of gravitational potential energy

Seorang budak perempuan membuat kerja apabila dia memanjat tangga sebuah papan gelongsor. Dia mempunyai
tenaga keupayaan graviti apabila dia berada pada kedudukan tertinggi papan gelongsor itu.
A girl does work when she climbs up the stairs of a sliding board. She has
gravitational potential energy when she is at the top of the sliding board.

Tenaga keupayaan graviti ialah tenaga yang tersimpan dalam objek


disebabkan ketinggian (kedudukan) nya daripada permukaan Bumi.
The gravitational potential energy is the energy stored in the object because of its
height (position) above the Earth’s surface.

Tenaga keupayaan graviti adalah sama dengan kerja dilakukan untuk menaikkan
satu objek kepada satu ketinggian tertentu. Daya diperlukan untuk menaikkan Kedudukan
akhir
objek adalah sama dengan berat objek, F = mg. Bola
Final
The gravitational potential energy is equal to the work done to raise an object to a Ball
position
particular height. The force required to raise the object is the same as the weight of the h
Kedudukan
object, F = mg. Jisim/Mass awal
= m kg Initial
Jika jarak menegak yang dilalui oleh objek ialah h, / If the distance moved by the position
object is h,

Kerja dilakukan, W = F × s Work done, W = F × s Tenaga keupayaan graviti,


= mg × h = mg × h Gravitational potential Energy,
= mgh = mgh EP = W

© Nilam Publication Sdn. Bhd. 94


MODUL • Fizik TINGKATAN 4

Peta Pemikiran / Thinking Maps


Tenaga Keupayaan Graviti (Mencirikan) / Gravitational Potential Energy (Characterising)

Dalam gerakan menegak,


In vertical movement,
Tenaga keupayaan graviti Tenaga yang tersimpan dalam objek
= Berat × Tinggi disebabkan ketinggian (kedudukan)
The energy stored in the object because
Gravitational potential energy
of its height (position)
= Weight × Height

Objek dalam
gerakan condong Bergantung pada
Object in incline motion Tenaga Depends on:
Keupayaan – Jisim / Mass
Graviti – Kekuatan medan graviti, g
h Gravitational Gravitational field strength, g
Potential Energy – Perubahan ketinggian U
N
Change in height I
T
E = (mg)h
2
Kes 1 / Case 1 Kes 2 / Case 2 Kes 3 / Case 3
Kotak Kotak
Box Box
Kotak
0.3 kg 0.3Box
kg
0.3 kg

0.3 kg 0.3 kg 1.2 m 1.2 m


2 kg 2 kg
0.2 m 0.20.3
m kg 1.2 m
0.8 m 0.8 m 2 kg
0.2 m
0.8 m Pendulum
Pendulum
Bob Bob
Pendulum
E = mgh E = mgh
Bob
E = mgh

Contoh / Examples
Kirakan tenaga keupayaan graviti bagi setiap keadaan berikut.
Calculate the gravitational potential energy for each of the following.
Kotak Kotak
(a) (b) (c) Box Box Kotak
Box
0.3 kg 0.3 kg
0.3 kg

0.3 kg 0.3 kg 1.2 m 1.2 m


2 kg 2 kg 0.3 kg 1.2 m
2 kg 0.2 m 0.2 m
0.8 m 0.8 m 0.2 m
0.8 m
Pendulum
Pendulum
Bob Bob Pendulum
Bob
Penyelesaian / Solution
E = mgh E = mgh E = mgh
= (2 kg) (10 m s ) (0.8 m) = (0.3 kg) (10 m s ) (0.2 m) = 0.3 kg × 10 m s–2 × 1.2 m
–2 –2

= 16 J = 0.6 J = 3.6 J

95 © Nilam Publication Sdn. Bhd.


MODUL • Fizik TINGKATAN 4

Nota:
Notes:
1 Tenaga keupayaan graviti bagi objek bergantung pada
The gravitational potential energy of an object depends on the

A A A

hm

B B B

(a) jisim objek, m / mass of the object, m


(b) kekuatan medan graviti, g / gravitational field strength, g
(c) perubahan ketinggian, h / change in height, h

U 2 "Kehilangan" tenaga keupayaan graviti tidak bergantung kepada kecerunan tetapi bergantung kepada jarak
N
I tegak ia bergerak.
T The "loss" of gravitational potential energy does not depend on the gradient of the slope but depends on the vertical
2 distance travelled.

Tenaga keupayaan graviti 100 J


Gravitational potential energy 100 J
Kerja dilakukan
sepanjang landasan
Work done along Kerja dilakukan
this path Work done in this path
= 100 J = mgh
= 100 J

Definisi tenaga kinetik


Definition of kinetic energy

Seorang budak lelaki yang sedang mengayuh basikal mempunyai tenaga kinetik.
Apabila dia mengayuh lebih pantas, maka dia mempunyai tenaga kinetik yang
lebih besar. Pada keadaan pegun, dia tidak mempunyai tenaga kinetik.
A boy riding a bicycle possesses kinetic energy. When he rides faster, he will have more
kinetic energy. When he is stationary, he does not have any kinetic energy.
Tenaga kinetik ialah tenaga yang diperoleh sesuatu objek disebabkan oleh
gerakannya.
Kinetic energy is the energy of an object due to its motion.

1 1
Tenaga kinetik= mv2 Kinetic energy = mv2
2 2
di mana m = jisim where m = mass
v = halaju v = velocity

© Nilam Publication Sdn. Bhd. 96


MODUL • Fizik TINGKATAN 4

Contoh / Examples

1 Sebiji bola berjisim 0.5 kg bergerak dengan halaju 4 m s-1. Hitungkan kerja yang dilakukan.
A ball of mass 0.5 kg moves with velocity of 4 m s-1. Calculate the work done.
Penyelesaian / Solution
Kerja = Tenaga kinetik Work done = Kinetic energy
1 1
= mv2 =
2
mv2
2
1 1
= (0.5 kg) (4 m s–1)2 = (0.5 kg) (4 m s–1)2
2 2
= 4.0 J = 4.0 J

2 Sebuah kereta berjisim 950 kg bergerak dengan halaju malar 20 m s-1 selama satu minit. Hitungkan kerja yang
dilakukan oleh kereta itu dalam tempoh yang tersebut.
A car of mass 950 kg moves at a constant velocity of 20 m s-1 for one minute. Calculate the work done by the car during
this period. U
m = 950 kg N
I
T
v = 20 m s–1
2
Penyelesaian / Solution
Kerja dilakukan = Tenaga kinetik
Work done = Kinetic energy
1
= mv2
2
1
= (950 kg) (20 m s–1)2
2
= 190 000 J

Prinsip keabadian tenaga / The principle of conservation of energy

Prinsip keabadian tenaga menyatakan bahawa tenaga boleh berubah dari satu bentuk ke satu bentuk yang
lain, tetapi tidak boleh dicipta atau dimusnahkan.
The principle of conservation of energy states that energy can be changed from one form to another form, but it
cannot be created or destroyed.

Jumlah tenaga di dalam sistem diabadikan.


The total energy in a system is conserved.

Jumlah tenaga sebelum berubah kepada bentuk tenaga yang lain = Jumlah tenaga selepas berubah kepada
bentuk tenaga yang lain.
Total energy before conversion to other forms of energy = Total energy after conversion to other forms of energy.

97 © Nilam Publication Sdn. Bhd.


MODUL • Fizik TINGKATAN 4

Aktiviti: Keabadian Tenaga / Activity: Conservation of Energy

1 (a) Sebiji bola dipegang pada ketinggian tertentu di atas lantai.


A ball is hold at a certain height above the floor.
Apakah jenis tenaga yang terdapat pada bola?
What is the energy gained by the ball?
Tenaga keupayaan graviti
Gravitational potential energy Bola
Ball

(b) Bola dilepaskan.


The ball is released.
(i) Apakah jenis tenaga yang diperoleh oleh bola sejurus sebelum ia menghentam Ketinggian
Height
lantai?
What is the energy gained by the ball just before it hits the floor?
Tenaga kinetik
Kinetic energy Lantai/Floor

U (ii) Dari manakah tenaga pada bola itu berasal?


N
I Where does the energy of the ball originate?
T Tenaga keupayaan graviti
2 Gravitational potential energy
(iii) Apakah hubungan antara tenaga di (a) dengan tenaga di (b)?
What is the relationship between the energy in (a) and the energy in (b)?
Sama / Equal

2 (a) Troli ditolak ke arah dinding untuk memampatkan spring.


Push a trolley against a wall to compress a spring.

Dinding
Wall
Troli
Trolley

Apakah tenaga yang tersimpan di dalam spring?


What is the energy stored in the spring?
Tenaga keupayaan kenyal
Elastic potential energy

(b) Troli dilepaskan supaya ia bergerak menjauhi dinding.


Release the trolley so that it moves away from the wall.
Dinding
Troli Wall
Trolley

Apakah yang berlaku kepada tenaga yang tersimpan di dalam spring?


What happens to the energy stored in the spring?
Tenaga keupayaan kenyal ditukarkan kepada tenaga kinetik.
The elastic potential energy is transformed to kinetic energy.

© Nilam Publication Sdn. Bhd. 98


MODUL • Fizik TINGKATAN 4

Contoh / Examples
(g = 10 m s–2)
1 Sebiji kelapa berjisim 1.2 kg jatuh dari suatu ketinggian. Sebelum kelapa jatuh
Abaikan rintangan udara. Before the coconut falls,
Tenaga keupayaan graviti = mgh
A coconut of mass 1.2 kg drops from a height. Ignore air resistance. Gravitational potential energy = mgh
(a) (i) Tentukan jarak buah kelapa itu jatuh dalam masa 2.0 s. Tenaga kinetik/Kinetic energy = 0
Determine the distance the coconut falls in 2.0 s.
(ii) Tentukan halajunya, selepas 2.0 s.
Determine its velocity after 2.0 s. h
(b) (i) Berapakah jumlah kehilangan tenaga keupayaan
graviti selepas 2.0 s?
What is the loss of its gravitational potential energy after
2.0 s?
(ii) Berapakah tenaga kinetiknya? / What is its kinetic energy then?
(c) Apakah yang boleh dikatakan tentang kehilangan tenaga keupayaannya dan tenaga kinetik yang diperoleh?
What can be said about the loss of its gravitational potential energy and the kinetic energy gained?
Penyelesaian / Solution
1 2
(a) (i) s = ut + at
2
1 U
= 0 + (10 m s–2)(2.0 s)2 N
2 I
= 20.0 m T
(ii) v = u + gt
v = 0 + (10 m s–2)(2.0 s) 2
= 20.0 m s–1
(b) (i) E = mgh = (1.2 kg) (10 m s–2) (20.0 m) = 240 J
(ii) Tenaga kinetik / Kinetic energy = 240 J
(c) Tenaga keupayaan graviti yang hilang telah ditukarkan kepada tenaga kinetik.
Lost of gravitional potential energy has been changed to kinetic energy.

2 Sebiji durian jatuh dari ketinggian 15 m. Berapakah halaju buah durian itu sejurus sebelum ia menghentam
tanah? (Andaikan bahawa g = 10 m s-2)
A durian falls from a height of 15 m. What is the velocity of the durian just before it hits the ground?
(Assume that g = 10 m s-2)
Penyelesaian / Solution
1
mgh = mv2
2
v2 = 2gh
= 2 × 10 m s–2 × 15 m
v = 17.32 m s–1

3 Sebiji bola dilepaskan pada titik A dari ketinggian 0.8 m dengan menggunakan A
landasan licin. Berapakah halaju bola itu pada titik B?
A ball is released at point A from a height of 0.8 m using a smooth inclined plane.
What is the velocity of the ball at point B?
0.8 m
Penyelesaian / Solution
Jumlah tenaga di A = Jumlah tenaga di B
Total energy at A = Total energy at B
1 B
mv2 mgh =
2
v2 = 2gh
= 2 × 10 m s–2 × 0.8 m = 16 m2 s–2
∴ v = 4 m s–1

99 © Nilam Publication Sdn. Bhd.


MODUL • Fizik TINGKATAN 4

Definisi kuasa
Definition of power

Kuasa ditakrifkan sebagai kadar kerja dilakukan atau kadar tenaga ditukarkan.
Power is defined as the rate of work done or the rate of energy transformed.
Kerja Tenaga Work Energy
Kuasa, P = Masa = Masa Power, P =
Time
=
Time

Unit S.I bagi kuasa ialah watt, W atau J s–1 .

S.I unit of power is watt, W or J s–1 .

Kuasa 1 W dihasilkan apabila 1 J kerja dilakukan dalam masa 1 saat.


A power of 1 W is generated when 1 J of work is done in 1 second.

Menerangkan Kuasa output


Kecekapan = × 100%
kecekapan peralatan Kuasa input
U
N Explain what efficiency Output power
I
of a device is Efficiency = × 100%
T Input power
2
Contoh / Examples
(g = 10 m s–2)
1 Seorang murid yang berjisim 45 kg mengambil masa 6 s untuk menaiki
tangga yang mempunyai 36 anak tangga. Jika tinggi setiap anak tangga ialah
12 cm, kirakan
A student of mass 45 kg takes 6 s to climb a flight of stairs that has 36 steps. If
each step is 12 cm high, calculate the
(a) kerja yang dilakukan oleh murid itu
work done by the student
(b) kuasa murid itu 12 cm
power of the student
Penyelesaian / Solution
(a) W = mgh = (45 kg) (10 m s–2) × (36 anak tangga × 0.12 m setiap anak tangga)
(45 kg) (10 m s–2) × (36 steps × 0.12 m each step)
= 1 944 J
1 944 J
(b) P =
6s
= 324 W

2 Sebuah motor mengangkat pemberat yang berjisim 1.5 kg pada ketinggian 1.0 m dalam masa 4.0 s. Berapakah
kuasa motor itu?
A motor lifting a weight having a mass of 1.5 kg up to a height of 1.0 m in 4.0 s. What is the power of the motor?
Penyelesaian / Solution
mgh
P =
t
= 1.5 kg × 10 m s–2 × 1 m = 3.75 W
4.0 s

© Nilam Publication Sdn. Bhd. 100


MODUL • Fizik TINGKATAN 4

3 Sebuah enjin petrol mempunyai kuasa output 96 kJ per minit. Berapakah kuasa input jika kecekapan enjin itu
ialah 20%?
A petrol engine has an output power of 96 kJ per minute. What is the input power if the engine efficiency is 20%?
Penyelesaian / Solution
3
Kuasa output / Output power = 96 × 10 J
60 s
= 1 600 W
Kuasa output Output power
Kecekapan = × 100% Efficiency = × 100%
Kuasa input Input power
∴ 20% = 1 600 W × 100% ∴ 20% =
1 600 W × 100%
Kuasa input Input power Maklumat tambahan:
Additional information:
Kuasa input = 1 600 W × 100% Input power = 1 600 W × 100%
20% 20%
= 8 000 W = 8 000 W

U
N
Latihan / Exercises (g = 10 m s–2) I
T
1 Sebuah troli dilepaskan dari keadaan rehat pada titik X. Berapakah halaju
X
2 kg
2
troli di titik Y?
A trolley is released from rest at point X. What is the velocity of the trolley at
point Y?
Penyelesaian / Solution 2.5 m
Dari X ke Y, jarak tegak = 1.5 m Y
From X to Y, the vertical distance = 1.5 m 1.5 m
∴ h = 1.5 m
1
mgh = mv2 Z
2
v2 = 2gh
= 2 × (10 m s–2)(1.5 m)
∴ v = 5.48 m s–1
vB = ?
B
2 Sebiji bola bergerak di sepanjang permukaan mengufuk yang licin dengan
halaju 6 m s-1. Bola itu kemudiannya bergerak naik ke atas satu satah condong
licin. Ketinggian satah condong itu ialah 1.5 m. Berapakah halaju bola itu di
titik B? 1.5 m
-1
A ball is moving along a smooth horizontal surface at a velocity of 6 m s . The ball vA = 6 m s–1
then moves up a smooth inclined plane. The height of the inclined plane is 1.5 m.
What is its velocity at point B? A

Penyelesaian / Solution
Jumlah tenaga di A = Jumlah tenaga di B
Total energy at A = Total energy at B
1 1
mvA2 = mgh + mvB2
2 2
1 1
(6 m s–1)2 = (10 m s–2)(1.5 m) + vB2
2 2
1 2
v = (18 – 15) m2 s–2
2 B
vB = 2.45 m s–1

101 © Nilam Publication Sdn. Bhd.


MODUL • Fizik TINGKATAN 4

3 Sebiji bola tenis dilontar ke atas dengan halaju awal 20 m s-1. Berapakah tinggi maksimum yang boleh
dicapai oleh bola tersebut?
A tennis ball is thrown upwards with an initial velocity 20 m s-1. What is the maximum height that the ball can reach?
Penyelesaian / Solution
2
1 mv2 = mgh, h = v
2 2g
–1 2
= (20 m s –2)
2(10 m s )
= 20 m

4 Nyatakan bentuk tenaga di titik


State the form(s) of energy at point

(a) P = Tenaga keupayaan graviti


Gravitational potential energy

(b) Q = Tenaga kinetik / Kinetic energy


Ketinggian
maksimum
U S Maximum height
(c) R = Tenaga kinetik + Tenaga keupayaan graviti
N P
I
T Kinetic energy + Gravitational potential energy R

2 (d) S = Tenaga keupayaan graviti


Q

Bandul ringkas
Ketinggian minimum
Minimum height

Gravitational potential energy Simple pendulum

5 Seorang budak lelaki berjisim, m, sedang berlari menaiki sebuah


tangga. Dia mengambil masa, t, untuk sampai ke puncak.
Berapakah kuasa budak itu? Beri jawapan anda dalam sebutan m,
g, Y dan t.
Y
A boy of mass, m, runs up the stairs. He takes time, t, to reach the top.
What is the power of the boy? Give your answer in terms of m, g, Y and t.
Penyelesaian / Solution
mgY mgY
Kuasa = t Power =
t

6 Sebuah kereta dengan kecekapan 25% memerlukan 3 000 J tenaga mekanikal setiap saat. Berapakah kuasa
output enjin itu?
A car engine with an efficiency of 25% needs 3 000 J of mechanical energy per second. What is the output power of
the engine?
Penyelesaian / Solution
Kuasa output Output power
Kecekapan = × 100% Efficiency = Input power × 100%
Kuasa input
Kuasa output Output power
25% = × 100% 25% = × 100%
3 000 W 3 000 W
25%
25% Output power = × 3 000 W
Kuasa output = × 3 000 W 100%
100%
= 750 W
= 750 W

© Nilam Publication Sdn. Bhd. 102


MODUL • Fizik TINGKATAN 4

7 Sebuah kren mengangkat beban 500 kg ke ketinggian 12 m dalam masa


8 s. Kuasa input ialah 45 000 W, berapakah kecekapan motor yang digunakan
oleh kren itu? / A crane lifts a load 500 kg to a height of 12 m in 8 s. The power
input is 45 000 W, what is the efficiency of the motor used in the crane?
Penyelesaian / Solution:
12 m
Kuasa output
Kecekapan = × 100% Output power
Kuasa input
Efficiency = Input power × 100%

500 kg × 108ms s 
–2
× 12 m

= × 100%
45 000 W
7 500 W
= × 100%
45 000 W
= 16.67%

8 Seorang budak lelaki berjisim 30 kg sedang duduk di atas puncak satu papan
gelongsor condong pada ketinggian 2.5 m dari tanah. Apabila budak lelaki
itu menggelongsor menuruni papan gelongsor, kerja yang dilakukan untuk U
N
mengatasi geseran ialah 510 J. Berapakah halaju pelajar itu sejurus sebelum I
2.5 m T
dia menyentuh tanah? / A boy of mass 30 kg sitting on the top end of an inclined
sliding board at a height of 2.5 m from the ground. When the boy slides down the 2
inclined board, the work done to overcome friction is 510 J. What is the velocity
of the student just before he touches the ground?
Penyelesaian / Solution:
1
mv2 + 510 J
mgh =
2
1
(30 kg)(10 m s–2)(2.5 m) = (30 kg)v2 + 510 J
2
∴ v = 4 m s–1

9 Abu dan basikalnya menuruni satu cerun bukit yang mempunyai ketinggian u = 2 m s–1
3 m pada halaju awalnya 2 m s-1 tanpa mengayuh. Di kaki bukit, halajunya
ialah 6 m s-1. Diberi bahawa jisim Abu dengan basikalnya ialah 75 kg.
Cari / Abu and his bicycle go down the slope of a hill of 3 m high at an initial
v = 6 m s–1
velocity of 2 m s-1, without pedaling. At the foot of the hill, the velocity is 6 m s-1.
Given that the mass of Abu with his bicycle is 75 kg. Find 3m
(a) tenaga kinetik awal dipunyai oleh Abu dan basikalnya.
the initial kinetic energy of Abu and his bicycle.
(b) tenaga keupayaan graviti awal yang dipunyai oleh Abu dan basikalnya.
the initial gravitational potential energy of Abu and his bicycle.
(c) kerja dilakukan menentang geseran sepanjang cerun.
the work done against friction along the slope.
Penyelesaian / Solution:
(a) Tenaga kinetik/Kinetic energy (b) mgh = (75 kg)(10 m s–2)(3 m)
1 = 2 250 J
= mv2 (c) Tenaga di puncak bukit = Tenaga di kaki bukit
2
1 Energy at the top of the hill = Energy at the foot of the hill
= (75 kg)(2 m s–1)2 1
2 2 250 J + 150 J = (75 kg)(6 m s–1)2 + geseran/friction
= 150 J 2
Geseran/Friction = 1 050 J

103 © Nilam Publication Sdn. Bhd.


MODUL • Fizik TINGKATAN 4

10 Rajah menunjukkan atlet lompat bergalah berjisim 60 kg melompat melepasi palang pada ketinggian 5.0 m.
J, K, L, M, N, O, P dan Q menunjukkan beberapa peringkat lompatan yang dibuat oleh atlit. Pada titik N,
ketinggian atlet dari paras palang ialah 0.2 m.
The diagram shows a pole vault jumper of mass 60 kg jumping over the bar of height 5.0 m. J, K, L, M, N, O, P and
Q show the different stages of the jump made by the athlete. At point N, the athlete clears the bar with 0.2 m to spare.

N
O

M
P

Arah pecutan
Direction of acceleration

L Q
Galah
U J Pole K
N
I
T Tilam getah yang tebal

2
A thick rubber maltress

(a) Mengapakah atlet perlu untuk memecut kepada halaju tertentu pada peringkat J ke K sebelum dia mula
melompat?
Why the athlete required to accelerate from J to K to a certain velocity before he begins to jump?
Untuk menambahkan tenaga kinetik. Apabila halaju bertambah, maka tenaga kinetik bertambah.
To increase the kinetic energy. Kinetic energy increases with velocity.

(b) Terangkan mengapa galah itu perlu dibengkokkan di L.


Explain why the pole has to be bent at L.
Untuk mendapatkan tenaga keupayaan kenyal yang maksimum.
To get maximum elastic potential energy.

(c) Kirakan tenaga keupayaan graviti bagi atlet pada titik N.


Calculate the gravitational potential energy of the athlete at point N.
E = mgh = (60 kg) × (10 m s–2) × (5.2 m)
= 3 120 J

(d) Berapakah pecutan menegak atlet di peringkat P?


What is the vertical acceleration of the athlete at stage P?
10 m s-2

(e) Mengapa sebuah tilam getah yang tebal diletakkan di kawasan di mana atlit mendarat?
Why is a thick rubber mattress placed in the area where the athlete lands?
Menambahkan masa hentaman untuk mengurangkan daya impuls.
Increase the time of collision to reduce impulsive force.

© Nilam Publication Sdn. Bhd. 104


MODUL • Fizik TINGKATAN 4

MEMAHAMI KEKENYALAN
2.11 UNDERSTANDING ELASTICITY

Aktiviti 1: Idea tentang kekenyalan / Activity 1: Idea of elasticity

Berikan definisi kembali ke panjang dan bentuk asal


Sifat bahan yang membolehkan objek
kekenyalan
apabila daya yang dikenakan ke atasnya dialihkan.
Define elasticity
A property of matter that enables an object to return to its original length and shape
when the force that acts on it is removed.

Tiada daya luar dikenakan. Molekul berada pada kedudukan asalnya .


Daya antara molekul adalah sifar.
No external force is applied. The molecules are at their original positions .
Intermolecular force is equal to zero.

Memampatkan pepejal menyebabkan molekulnya bergerak lebih rapat antara satu sama U
N
lain. Daya tolakan antara molekul bertindak untuk menolak molekul kembali I
T
kepada kedudukan asalnya.
Compressing a solid causes its molecules to be moved closer to each other. 2
Repulsive intermolecular forces act to push the molecules back to their original
positions.

Meregangkan pepejal menyebabkan molekulnya bergerak menjauhi antara satu sama


lain. Daya tarikan antara molekul bertindak untuk menarik kembali molekul
kepada kedudukan asalnya.
Stretching a solid causes its molecules to be moved further from each other.
Attractive intermolecular forces act to pull back the molecules to their original
positions.

Meregang wayar dengan • Molekulnya akan menjauhi antara satu sama lain.
daya luar Its molecules will move further away from one another.
Stretching a wire by an
external force:
• Daya tarikan yang kuat akan bertindak di antara molekul untuk menentang
regangan yang dikenakan.
Strong attractive forces act between the molecules to oppose the stretching.
Apabila daya luar dialih:
When the external force is removed:
• Daya tarikan antara molekul membawa molekul kembali ke kedudukan asalnya.
The attractive intermolecular forces bring the molecules back to their original positions.

• Wayar itu kembali ke kedudukan asalnya.


The wire returns to its original position.

105 © Nilam Publication Sdn. Bhd.


MODUL • Fizik TINGKATAN 4

Aktiviti 2: Hubungan antara daya dan pemanjangan spring


Activity 2: Relationship between force and extension of a spring

Inferens Pemanjangan spring bergantung kepada daya yang dikenakan.


Inference Extension of a spring depends on the force applied.

Hipotesis Pemanjang spring bertambah apabila daya yang dikenakan bertambah.


Hypothesis The extension of a spring increases when the force applied increases.

Tujuan Untuk mengkaji hubungan antara daya dan pemanjangan spring.


Aim To investigate the relationship between a force and the extension of a spring.

Pemboleh ubah Daya / Berat / Jisim


dimanipulasi
Manipulated variable Force / Weight / Mass

Pemboleh ubah Pemanjangan spring


bergerak balas
Responding variable Extension of a spring

Pemboleh ubah Diameter spring / Ketebalan spring


U
N dimalarkan
I Fixed variable Diameter of the spring / Thickness of the spring
T

2 1 Tandakan kedudukan awal pin pada pembaris meter


apabila tiada pemberat dilekatkan kepada spring, l0.
Mark the initial position of the pin on the metre rule when Pembaris
meter
no weight is attached to the spring, l0. Metre rule
2 Gantungkan pemberat berjisim 50 g di bahagian Pin / Pin
hujung spring dan bandingkan kedudukan baru pin Pemberat
Prosedur
Procedure
sekarang dengan kedudukan asalnya. Weight
Attach a slotted weight of 50 g to the end of the spring and
compare the new position of the pin with its initial position.
Spring/Spring Kaki retort
3 Ukur pemanjangan spring, x = l – l0. Retort stand
Measure the extension of the spring, x = l – l0 .
4 Ulangi eksperimen dengan jisim 100 g, 150 g, 200 g dan 250 g.
Repeat the experiment with mass 100 g, 150 g, 200 g and 250 g.

Jadualkan data bagi m, F, l dan x.


Tabulate data for m, F, l and x.
l0 = cm
Jisim / kg Daya / Force, F Pemanjangan / Extension, x
l / cm
Mass / kg F = mg / N x = l – l0 / cm

Merekodkan data
Recording data

Plotkan graf daya, F melawan pemanjangan spring, x.


Plot a graph of force, F against extension of the spring, x.

© Nilam Publication Sdn. Bhd. 106


MODUL • Fizik TINGKATAN 4

Daya / Force, F / N

Menganalisis data
Analysis data

0 Pemanjangan / Extension, x / cm

1 Daripada graf, apakah hubungan antara daya dengan pemanjangan spring, x?


From the graph, what is the relationship between a force and the extension of the spring, x?
Daya, F, berkadar langsung dengan pemanjangan spring, x
The force, F, is directly proportional to the extension of the spring, x

2 Kira kecerunan graf. Tunjukkan bagaimana anda mendapat kecerunan daripada graf.
Calculate the gradient of the graph. Show how you get the gradient from the graph.
Daya Force
Perbincangan Kecerunan = Pemanjangan spring Gradient = Extension of spring
Discussion
3 Langkah berjaga-jaga perlu diambil supaya spring itu tidak diregangkan dengan berat
yang berlebihan. Terangkan mengapa. U
Precaution should be taken so that the spring is not stretched by excessive weights. Explain why. N
I
Jika spring diregangkan dengan berat yang berlebihan, ia mungkin tidak akan kembali T

ke panjang asal kerana telah melebihi had kenyal. 2


If the spring is stretched by too large weight, it might not return to its original length due to
its exceeding the elastic limit.

Hukum Hooke Pemanjangan spring berkadar terus dengan daya yang dikenakan asalkan tidak melebihi
Hooke’s Law
had kenyalnya.
The extension of a spring is directly proportional to the applied force as long as the elastic limit is
not exceeded.
F = kx di mana / where F = daya ke atas spring / force on the spring
x = pemanjangan spring / extension of the spring
k = pemalar spring / spring constant

Graf daya- Berdasarkan graf / Based on the graph:


pemanjangan • F berkadar terus kepada x.
spring F is directly proportional to x.
Force-extension
• Kecerunan graf = pemalar spring bagi spring, k.
graph
The gradient of the graph = spring constant of the spring, k.
F (N)
• Luas di bawah graf adalah sama dengan kerja yang dilakukan untuk memanjangkan spring
= tenaga keupayaan elastik
Area under the graph is equal to the work done to extend the spring = elastic potential energy
• Kerja yang dilakukan = Tenaga keupayaan kenyal
0 x (cm) Work done = Elastic potiential energy
½ Fx = ½ kx2

107 © Nilam Publication Sdn. Bhd.


MODUL • Fizik TINGKATAN 4

Had kenyal bagi


Berat maksimum yang boleh dikenakan kepada spring supaya spring itu masih boleh kembali
spring
The elastic limit of a ke panjang asalnya apabila daya dialihkan.
spring The maximum weight that can be applied to a spring such that the spring will be able to return to
its original length when the force is removed.
F (N)
Jika daya regangan spring melebihi had kenyal, spring tidak kembali ke panjang asal
walaupun tiada lagi daya dikenakan ke atasnya.
Had
kenyal
If a force stretches a spring over its elastic limit, the spring cannot return to its original length even
Elastic though the force no longer acts on it.
limit
0 x (cm) Hukum Hooke tidak dipatuhi lagi. / The Hooke’s Law is not obeyed anymore.

Pemalar daya Pemalar daya spring, k, ialah daya yang diperlukan untuk menghasilkan satu unit pemanjangan
spring, k spring. / The force constant, k, of a spring is the force required to produce one unit of extension of
Force constant of a the spring.
spring, k F
k= unit: N m-1
Spring keras x
F (N) Stiff spring
75
k ialah pengukuran kekerasan spring.
k is a measurement of the stiffness of the spring.
U Spring
N 12.5 lembut
I Soft spring
T 0 x (m) • Spring dengan pemalar daya, k, yang tinggi adalah lebih susah untuk dipanjangkan dan

2
dikatakan lebih keras.
The spring with a high force constant, k, is harder to extend and is said to be more stiff.

• Spring dengan pemalar daya, k, yang kecil adalah lebih mudah untuk dipanjangkan dan
dikatakan lebih lembut atau kurang keras. / A spring with a small force constant, k, is easier
to extend and is said to be softer or less stiff.

Faktor-faktor yang mempengaruhi kekenyalan


Factors that affect elasticity

Faktor Bagaimana ia mempengaruhi kekenyalan


Perubahan faktor / Change in factor
Factor How it affects the elasticity

Spring lebih pendek / Shorter spring Kurang kenyal / Less elastic


Panjang
Length
Spring lebih panjang / Longer spring Lebih kenyal / More elastic

Diameter dawai Dawai lebih tebal / Thicker wire Kurang kenyal / Less elastic
spring
Diameter of
Dawai lebih nipis / Thinner wire Lebih kenyal / More elastic
spring wire

Diameter lebih kecil / Smaller diameter Kurang kenyal / Less elastic


Diameter spring
Diameter spring
Diameter lebih besar / Larger diameter Lebih kenyal / More elastic

Spring diperbuat daripada pelbagai bahan. Perubahan kekenyalan mengikut jenis bahan
Jenis bahan
Type of material
yang digunakan.
Springs are made of different materials. Elasticity changes according to the type of material.

© Nilam Publication Sdn. Bhd. 108


MODUL • Fizik TINGKATAN 4

Susunan spring yang serupa


Arrangement of identical springs

Secara bersiri / In series Secara selari / In parallel

Spring serupa
Spring serupa Identical springs
Identical springs

Beban Beban
Load Load

Beban yang sama dikenakan kepada setiap spring. Beban dikongsi bersama antara spring.
The same load is applied to each spring. The load is shared equally among the springs.
Tegangan dalam setiap spring = T T
Tegangan dalam setiap spring =
Tension in each spring = T T 2
Pemanjangan spring = x Tension in each spring =
2 x
Extension of each spring = x Pemanjangan setiap spring =
Jumlah pemanjangan = 2x x 2
Extension of each spring =
Total extension = 2x 2 U
Jika bilangan spring digunakan = n, N
Jika bilangan spring digunakan = n, If number of springs used = n, I
Jumlah pemanjangan = nx x T
Jumlah pemanjangan = n
If number of springs used = n,
The total extension = nx
x
The total extension = n 2

Contoh / Examples 1
Panjang asal spring ialah 5 cm. Dengan beban berjisim 20 g, spring memanjang kepada 7 cm.
The original length of a spring is 5 cm. With a load of mass 20 g, the length of the spring is extended to 7 cm.
Tentukan / Determine
(a) pemanjangan spring dengan beban 40 g.
the extension of the spring with a load 40 g.
(b) panjang spring dengan beban 60 g.
the length of the spring with a load of 60 g.
Penyelesaian / Solution
(a) 20 g → 7 cm – 5 cm = 2 cm
40 g → 4 cm

(b) 20 g menghasilkan pemanjangan 2 cm


20 g gives an extension of 2 cm
∴ 60 g → pemanjangan / extension 6 cm
∴ panjang spring dengan beban 60 g = 5 cm + 6 cm = 11 cm
length of spring with 60 g load = 5 cm + 6 cm = 11 cm

109 © Nilam Publication Sdn. Bhd.


MODUL • Fizik TINGKATAN 4

Contoh / Examples 2
Rajah / Diagram
(a) Panjang asal spring ialah 10.0 cm. Apabila ia diregangkan
(a)
dengan daya 6 N, ia memanjang kepada 13.0 cm. Apakah
F = 6.0 N
pemalar spring?
13.0 cm
The original length of a spring is 10.0 cm. When it is stretched by
(b)
F = 12.0 N a force of 6 N, it extends to 13.0 cm. What is the spring constant?
x = 13 cm – 10 cm = 3 cm
(c) F 6N
k= = = 2 N cm–1
F = 12.0 N x 3 cm

(b) Dua spring yang serupa disambungkan secara bersiri seperti ditunjukkan dalam rajah (b).
Two identical springs are connected in series as shown in diagram (b).
(i) Berapakah jumlah panjang spring jika diregangkan oleh daya 12.0 N?
What is the total length of the spring if stretched by 12.0 N force?
6 N → 3 cm Jumlah panjang / Total length
12 N → 6 cm = 10 cm + 10 cm + 6 cm + 6 cm
= 32 cm
U
N (ii) Berapakah pemalar spring bagi sistem spring di (b)?
I What is the spring constant of the spring system in (b)?
T
F 12 N
2 k=
x
=
12 cm
= 1 N cm–1

(c) 2 spring yang serupa disambungkan secara selari seperti ditunjukkan dalam rajah (c).
2 identical springs are connected in parallel as shown in diagram (c).
(i) Berapakah jumlah panjang sistem spring itu? / What is the total length of the spring system?
4 N → 1 cm Jumlah panjang/Total length
∴ 12 N → 3 cm = 10 cm + 3 cm = 13 cm
(ii) Berapakah pemalar spring bagi sistem spring di (c)? / What is the spring constant of the spring system in (c)?
12 N
k= = 4 N cm–1
3 cm

Contoh / Examples 3
Spring A memanjang 2 cm apabila ia digantung dengan pemberat 10 g. Spring B memanjang 4 cm apabila ia
digantung dengan pemberat 10 g. Cari jumlah regangan pada setiap sistem spring seperti ditunjukkan dalam rajah
sebelah. / Spring A extends by 2 cm when a 10 g weight is hung on it. Spring B extends by 4 cm when a 10 g weight is hung
on it. Find the total extension in each of the spring systems shown in the diagrams on the right.
Penyelesaian / Solution
(a) A: 10 g → 2 cm B: 10 g → 4 cm
20 g → 4 cm 20 g → 8 cm
Jumlah pemanjangan / Total extension = 4 cm + 8 cm = 12 cm A A A B B
(b) 10 g → 1 cm 50 g → 5 cm
Pemanjangan sistem / Extension in the system = 5 cm
(c) Sistem B/System B : 10 g → 2 cm A
B
∴ 40 g → 8 cm 50 g
A : 10 g → 2 cm
∴ 40 g → 8 cm 20 g
∴ Pemanjangan sistem / Extension in the system 40 g
= 8 cm + 8 cm
= 16 cm (a) (b) (c)

© Nilam Publication Sdn. Bhd. 110


MODUL • Fizik TINGKATAN 4

Contoh / Examples 4
Panjang asal spring ialah 12 cm. Dengan beban 20 g, panjang spring dipanjangkan kepada 15 cm. Berapakah
tenaga keupayaan kenyal yang tersimpan di dalam spring?
The original length of a spring is 12 cm. With a load of 20 g, the length of the spring is extended to 15 cm. What is the
elastic potential energy stored in the spring?
Penyelesaian / Solution
1
E = Fx
2
1
= × (0.02 kg × 10 m s–2) × 0.03 m
2
= 0.003 J

Contoh / Examples 5
Rajah menunjukkan graf daya, F, melawan pemanjangan, x, bagi spring. Berapakah F/N
tenaga keupayaan kenyal yang tersimpan apabila spring diregangkan sebanyak 0.4 m?
The diagram shows a graph of force, F, against extension, x, for a spring. What is the elastic potential 20
energy stored when the spring is extended by 0.4 m? U
N
Penyelesaian / Solution I
1 T
E = Fx
2
1
0
0.4
x/m 2
= (20 N) (0.4 m)
2
= 4.0 J

Contoh / Examples 6
Rajah menunjukkan sebiji bebola keluli berjisim 10 g ditolak pada penghujung satu spring di sepanjang permukaan
licin. Panjang asal spring ialah 14 cm dan pemalar spring ialah 200 N m-1.
The diagram shows a steel ball of mass 10 g being pushed against one end of a spring along a smooth surface. The original
length of the spring is 14 cm and its spring constant is 200 N m-1.
14 cm 10 cm

Daya
Force

Tentukan / Determine
(a) tenaga keupayaan kenyal tersimpan di dalam spring. / the elastic potential energy stored in the spring.
1
E= kx2
2
1
= (200 N m–1) (0.04 m)2
2
= 0.16 J

(b) halaju maksimum bola itu selepas daya mampatan pada spring itu dialihkan.
the maximum velocity of the ball after the force of compression on the spring is removed.
1 J Nm
mv2 = 0.16 J Perhatian / Note: =
2 kg kg
2 × 0.16 J
v2 = (kg m s–2)(m)
0.01 kg =
kg
= 32 m2 s–2 2 –2
v = 5.66 m s–1 =m s

111 © Nilam Publication Sdn. Bhd.


MODUL • Fizik TINGKATAN 4

Latihan / Exercises
Rajah di bawah menunjukkan susunan radas dalam eksperimen untuk menentukan hubungan di antara pemanjangan
spring, x, dengan berat, W. Hubungan di antara x dengan W ditunjukkan dalam graf di bawah.
The diagram shows the arrangement of an apparatus in an experiment to determine the relationship between the extension,
x, of the spring with weight W. The relationship between x and W is shown in the graph below.

Pemanjangan, x / cm
Extension, x / cm
7
Spring 6
Spring
5

4
Pemberat 3
berslot
Slotted weight 2

0
0 2 4 6 8 10 12 14
U
N Berat, W / N
I Weight, W / N
T

2 (a) (i) Nyatakan hubungan antara x dan W.


State the relationship between x and W.
x berkadar langsung dengan W, asalkan had kenyal tidak dilebihi.
x is directly proportional to W, provided the elastic limit is not exceeded.

(ii) Namakan hukum saintifik yang terlibat dalam hubungan yang dinyatakan di (a)(i).
Name the scientific law involved in the relationship stated in (a)(i).
Hukum Hooke
Hooke’s Law

(b) Tandakan dengan tanda pangkah (×) had kenyal spring di dalam graf.
Mark a cross (×) at the elastic limit of the spring on the graph.
(c) Berdasarkan graf, tentukan pemalar daya bagi spring, k (dalam unit N m–1).
Based on the graph, determine the force constant of a spring, k (in N m–1).
10 N
k= = 250 N m–1
0.04 m

(d) Tenaga keupayaan kenyal tersimpan dalam spring apabila ia diregangkan. Kirakan tenaga dalam spring ini
apabila ia memanjang sebanyak 4 cm.
Elastic potential energy is stored in the spring when it is extended. Calculate this energy in the spring when it is
extended by 4 cm.
1
E= Fx
2
Maklumat tambahan:
1 Additional information:
= (10 N) (0.04 m)
2
= 0.2 J

© Nilam Publication Sdn. Bhd. 112


MODUL • Fizik TINGKATAN 4

Latihan Pengukuhan / Enrichment Exercises


1 Rajah 1 menunjukkan troli diletakkan di atas 4 Rajah 3 menunjukkan graf halaju-masa untuk Adnan,
landasan terpampas geseran. Hamid dan Lim sepanjang perjalanan di jalan lurus
Diagram 1 shows a trolley placed on a friction- mengikut urutan.
compensated runway. Diagram 3 shows the velocity-time graphs of Adnan, Hamid
and Lim respectively travelling along a straight road.
v/m s–1 v/m s–1
Adnan Hamid
6 6
Rajah 1 / Diagram 1 4 4
Troli itu akan / The trolley will
A bergerak dengan halaju bertambah.
move with an increasing velocity. t/s t/s
0 1 2 3 4 0 1 2 3 4
B bergerak dengan halaju berkurang.
move with decreasing velocity. v/m s–1
C bergerak dengan halaju malar/tetap. 6
Lim
move with a constant velocity.
D kekal tidak bergerak.
U
remain stationary. N
I
t/s T
2 Tangki silinder minyak kereta En. Ali bocor.
2
0 1 2 3 4
Tompokan minyak kelihatan sepanjang jalan dari A
Rajah 3 / Diagram 3
ke B seperti ditunjukkan dalam Rajah 2.
Jika mereka bertiga memulakan perjalanan dari
The cylinder oil tank of En. Ali’s car is leaking. Lubricant
tempat yang sama dan masa yang sama, susun nama
spots are seen along the road from A to B as shown in
Diagram 2.
mereka dalam urutan menaik berdasarkan sesaran
mereka dari titik permulaan. / If they started their
A B
journey from the same place at the same time, arrange
their name in accordance to increasing order of their
displacements from the starting point.
10 m 15 m 20 m A Adnan, Hamid, Lim
Rajah 2 / Diagram 2 B Adnan, Lim, Hamid
Ini menunjukkan bahawa kereta Ali bergerak dengan C Hamid, Lim, Adnan
It can be concluded that En Ali’s car is moving with D Lim, Adnan, Hamid
A pecutan malar. / constant acceleration.
B nyahpecutan malar. / constant deceleration. 5 Rajah 4 menunjukkan sebiji durian gugur dari
C pecutan sifar. / zero acceleration. sebatang pokok.
D pecutan berkurang dengan malar/tetap. Diagram 4 shows a durian falling from a tree.
acceleration that is decreasing constantly.

3 Antara pernyataan berikut, yang manakah


menghuraikan perlanggaran kenyal antara dua objek
dengan betul? / Which of following statements best
describes an elastic collision between two objects?
A Hanya mementum terabadi.
Only momentum is conserved.
Rajah 4 / Diagram 4
B Hanya tenaga kinetik terabadi.
Apakah kuantiti fizik yang ditetapkan semasa durian
Only kinetic energy is conserved.
itu gugur? / What is the physical quantity that is constant
C Kedua-dua momentum dan tenaga kinetik
when the durian falls?
terabadi.
A Halaju / Velocity
Momentum and kinetic energy are conserved.
B Momentum / Momentum
D Jumlah tenaga, momentum dan tenaga kinetik
C Pecutan / Acceleration
terabadi. / Total energy, momentum and kinetic
energy are conserved. D Tenaga kinetik / Kinetic energy

113 © Nilam Publication Sdn. Bhd.


MODUL • Fizik TINGKATAN 4

6 Sebiji bola dilontar ke atas pada ketinggian h m dari 10 Rajah 6 menunjukkan alas bebola, P, yang berjisim 50
tanah. Antara berikut, yang manakah betul tentang g dilepaskan atas satu landasan licin. / Diagram 6 shows
gerakan bola itu? / A ball is thrown upwards to a height 50 g ball bearing P, being released on a smooth plane.
of h m from the ground. Which of the following is true
Alas bebola P
about the motion of the ball? Ball bearing P
A Tenaga kinetik adalah malar sepanjang gerakan. Landasan licin
Its kinetic energy is constant throughout the motion. Smooth plane
B Tarikan graviti pada ketinggian h adalah sifar.
The gravitational attraction at the height h is zero. 1.6 m
C Masa yang diambil untuk mencapai ketinggian X
h meter adalah sama dengan masa yang diambil
untuk jatuh dari ketinggian h meter ke tanah. Rajah 6 / Diagram 6
The time taken to reach a height of h metres is the Berapakah halaju alas bebola itu di X?
same as the time taken to fall from a height of h What is the velocity of the ball bearing at X?
metres to the ground. A 1.60 m s–1 C 5.66 m s–1
B 4.00 m s–1 D 8.00 m s–1
7 Seorang budak lelaki berjisim 50 kg berada di dalam
lif. Berat badannya mencapai 500 N apabila 11 Suatu beban 800 g digantung pada penghujung spring
A boy of mass 50 kg stands in a lift. His weight reaches seperti yang ditunjukkan dalam Rajah 7. Panjang asal
U 500 N when spring itu ialah 6.0 cm.
N
I (g = 10 N kg–1) A load of 800 g hangs at one end of a spring as shown in
T A lif bergerak ke atas dengan halaju malar. Diagram 7. The original length of the spring is 6.0 cm.

2 B
the lift moves upwards with a constant velocity.
lif bergerak ke bawah dengan pecutan seragam.
the lift moves downwards with a constant acceleration.
C lif itu jatuh bebas. 10.0 cm
the lift falls freely.

8 Rajah 5 menunjukkan seekor kucing berjisim 2.5 kg 800 g

menaiki tangga setinggi 3.0 m dalam masa 1.2 s. Rajah 7 / Diagram 7


Diagram 5 shows a cat with mass 2.5 kg running upstairs Tenaga keupayaan kenyal yang tersimpan dalam
to a height of 3.0 m in 1.2 s.
spring ialah / The elastic potential energy stored in the
spring is
A 0.16 J C 16.00 J
B 1.60 J D 1 600.00 J
3.0 m
12 Rajah 8 menunjukkan seorang lelaki menarik sebuah
blok dengan daya 20.0 N.
Diagram 8 shows a man pulling a block with a force of
Rajah 5 / Diagram 5 20.0 N.
Berapakah kuasa yang dijanakan oleh kucing itu? G
How much power is generated by the cat? 20.0 N
A 6.3 W C 9.0 W Blok
B 7.5 W D 62.5 W Block 30°

9 Antara berikut, yang manakah bukan ciri-ciri


keselamatan yang dipasang pada kenderaan? / Which Rajah 8 / Diagram 8
of the following is not a safety feature installed in a vehicle? Daya komponen mencancang, G, yang terhasil ialah
A Kapasiti enjin. / Engine capacity. The vertical component force, G, that is produced is
B Beg udara automatik. / Automatic air bag. A 6.7 N
C Roda stereng boleh runtuh. B 10.0 N
Collapsible steering wheels. C 17.3 N
D Cermin kaca tahan pecah. D 20.0 N
Shatterproof windscreen glass.

© Nilam Publication Sdn. Bhd. 114


MODUL • Fizik TINGKATAN 4

Soalan Struktur / Structure Questions


1 Rajah 1.1 menunjukkan sebuah bas yang bergerak dengan halaju 70 km/j dan menunjukkan kedudukan penumpang
di dalam bas apabila brek tiba-tiba digunakan. Rajah 1.2 menunjukkan tukang masak cuba untuk menggoncang sos
keluar daripada botol.
Diagram 1.1 shows a bus moving with a velocity of 70 km/h and shows the condition of a passenger standing in the bus when
the brakes are suddenly applied. Diagram 1.2 shows a cook trying to shake the sauce out of a bottle.

Rajah 1.1 / Diagram 1.1 Rajah 1.2 / Diagram 1.2


(a) Berdasarkan kedua-dua rajah di atas, nyatakan satu ciri yang dialami oleh penumpang dan sos.
Based on both of the diagrams above, state one characteristic experienced by the passengers and the sauce.
Penumpang bergerak ke hadapan dan sos cili mengalir keluar daripada botol.
U
N
The passengers move forward and the chilli sauce comes out from the bottle. I
T
(b) Apakah halaju penumpang
What is the velocity of the passenger 2
(i) sebelum pemandu bas membrek?
before the bus driver brakes?
70 km j–1/ 70 km h–1
(ii) selepas pemandu bas membrek?
after the bus driver brakes?
–1
Halaju yang sama iaitu 70 km j –1. / The same velocity that is 70 km h .

(c) Apakah halaju botol apabila sos mengalir keluar?


What is the velocity of the bottle when the sauce comes out?
Halaju sifar. / Zero velocity.
(d) (i) Berdasarkan jawapan (b), tulis satu kenyataan pergerakan bas dan penumpangnya.
Based on the answer to (b), write a statement on the motion of the bus and its passengers.
Apabila bas brek, halaju bas berkurangan dengan tiba-tiba, penumpang terus bergerak dengan halaju awalnya.

When the bus brakes, the velocity of the bus decreases suddenly, the passenger continues to move with its initial velocity.
(ii) Namakan konsep fizik yang terlibat.
Name the physics concept involved here.
Inersia / Inertia

(e) Jika anda dikejar oleh anak gajah, bagaimana anda berlari untuk menyelamatkan diri? Terangkan.
If you are chased by a baby elephant, how can you run to save yourself? Explain.
Saya perlu berlari secara zig zag. Anak gajah akan terus bergerak dalam garis lurus apabila saya menukar
arah dengan tiba-tiba. / I have to run in a zig zag manner. The baby elephant will continue to move in a straight
line when I change my direction suddenly.

115 © Nilam Publication Sdn. Bhd.


MODUL • Fizik TINGKATAN 4

Teknik Menjawab [Format Kertas 2: Perbandingan]


Answering Technique [Paper 2 Format: Comparison]

Perhatikan Rajah (a) dan Rajah (b) dengan teliti. Halaju basikal di Rajah (a) bertambah dengan kadar yang lebih cepat
daripada halaju basikal di Rajah (b). / Observed the Diagram (a) and Diagram (b) carefully. The velocity of bicycle in Diagram
(a) increases with higher rate than the bicycle in Diagram (b).

Rajah (a) / Diagram (a) Rajah (b) / Diagram (b)

(a) Lengkapkan jadual di bawah dengan membandingkan aspek-aspek berikut:


Complete the table below by comparing the following aspects:
Aspek / Aspect Rajah (a) / Diagram (a) Rajah (b) / Diagram (b)
Jumlah jisim basikal Sama Sama
U Total mass of bicycle Same Same
N
I
T Bilangan penunggang yang terlibat dalam tindakan Dua orang Satu orang
2 pengayuhan
Number of cyclist involved in action of cycling Two One

Daya tujah ke depan yang terhasil Lebih besar Lebih kecil


Thrust force produced Larger Smaller

Pecutan basikal Lebih besar Lebih kecil


Acceleration of bicycle Larger Smaller

(b) Nyatakan hubungan antara daya tujah ke depan dan pecutan basikal.
State the relationship between thrust force and acceleration of bicycle.
Semakin bertambah daya tujah ke depan, semakin bertambah pecutan basikal.
As the thrust force increases, the acceleration of bicycle increases.

(c) Berdasarkan jawapan anda di (b), namakan hukum fizik yang terlibat.
According to your answer in (b), name the physics law involved.
Hukum Newton Kedua / Newton's Second Law

(d) Kenal pasti pemboleh ubah: / Identify the variables:


(i) Dimanipulasikan / Manipulated : Daya, F / Force, F
(ii) Bergerak balas / Responding : Pecutan, a / Acceleration, a
(iii) Dimalarkan / Constant : Jisim, m / Mass, m

Teori: / Theory:
a
(a) Apabila jisim, m adalah malar, pecutan, a, bergantung pada daya, F.
As mass, m, is constant, acceleration, a, depends on force, F.
F = ma
(b) Semakin bertambah daya, F, semakin bertambah pecutan, a.
As the force, F increases, the acceleration, a increases. F#
a#=
(c) a berkadar langsung dengan F (a α F). m (malar/constant)
a is directly proportional with F. F
0

© Nilam Publication Sdn. Bhd. 116

You might also like